You are on page 1of 122

2017-18 100 &

op kers
Class 11 T
By E ran culty
-JE Fa r
IIT enior emie .
S fP r es
o titut
Ins

PHYSICS
FOR JEE MAIN & ADVANCED
SECOND
EDITION

Exhaustive Theory
(Now Revised)

Formula Sheet
9000+ Problems
based on latest JEE pattern

2500 + 1000 (New) Problems


of previous 35 years of
AIEEE (JEE Main) and IIT-JEE (JEE Adv)

5000+Illustrations and Solved Examples


Detailed Solutions
of all problems available

Topic Covered Plancess Concepts


Tips & Tricks, Facts, Notes, Misconceptions,
Rotational Mechanics Key Take Aways, Problem Solving Tactics

PlancEssential
Questions recommended for revision
7. R OTAT I O N A L
MECHANICS

1. INTRODUCTION
In this chapter we will be studying the kinematics and dynamics of a solid body in two kinds of motion. The first
kind of motion of a solid body is rotation about a stationary axis, also called pure rotation. The second kind of
motion of a solid is the plane motion wherein the center of mass of the solid body moves in a certain stationary
plane while the angular velocity of the body remains permanently perpendicular to that plane. Here the body
executes pure rotation about an axis passing through the center of mass and the center of mass itself translates
in a stationary plane in the given reference frame. The axis through the center of mass is always perpendicular to
the stationary plane. We will also learn about the inertia property in rotational motion, and the quantities torque
and angular momentum which are rotational analogue of force and linear momentum respectively. The law of
conservation of angular momentum is an important tool in the study of motion of solid bodies.

2. BASIC CONCEPT OF A RIGID BODY


A solid is considered to have structural rigidity and resists 
change in shape, size and density. A rigid body is a solid body
B
which has no deformation, i.e. the shape and size of the body
remains constant during its motion and interaction with other A
bodies. This means that the separation between any two points
of a rigid body remains constant in time regardless of the kind of
motion it executes and the forces exerted on it by surrounding 
v
bodies or a field of force.
A metal cylinder rolling on a surface is an example of a rigid
body as shown in Fig. 7.1. 

Let velocities of points P and Q of a rigid body with respect to a Figure 7.1: Metal cylinder rolling on a surface is a
reference frame be VP and VQ as shown in the Fig. 7.2. rigid body system. Relative distance between points
A and B do not change.
As the body is rigid, the length PQ should not change during
the motion of the body, i.e. the relative velocity between P and
Q along the line joining P and Q should be zero i.e. velocity of approach or separation is zero. Let x-axis be along
PQ, then

VQP = relative velocity of Q with respect to P

VQP = ( VQ cosθ2 î + VQ sinθ2 ˆj ) – ( VP cosθ1 î – VP sinθ1 ˆj )

VQP = ( VQ cosθ2 – VP cosθ1) î + ( VP sinθ1 + VQ sinθ2) ˆj
7 . 2 | Rotational Mechanics

Now VP cosθ1 = VQ cosθ2


VPsin1
(Since velocity of separation is 0) P
 P
VPcos1
VQP = ( VP sinθ1 + VQ sinθ2) ˆj (which is VP 1

perpendicular to line PQ). Q


Q VQsin2
2 VQ VQcos2
Hence, we can conclude that for each and VQP
every pair of particles in a rigid body, relative
motion between the two points in the pair will
be perpendicular to the line joining the two
points.
Figure 7.2: Relative velocity between
two points of a rigid body

PLANCESS CONCEPTS

A A 1
B B
 
1
C C

(a) (b)

Figure 7.3: (a) Angular velocity of A and B w.r.t. C is ω1 (b) Angular velocity of A and C w.r.t. B is ω1

Suppose A, B, C are points of a rigid system hence during any motion the lengths of sides AB, BC, and CA
will not change, and thus the angle between them will not change, and so they all must rotate through
the same angle. Hence all the sides rotate by the same rate. Or we can say that each point is having the
same angular velocity with respect to any other point on the rigid body.
Neeraj Toshniwal (JEE 2009 AIR 21)

3. MOTION OF A RIGID BODY 

We will study the dynamics of three kinds of motion of a rigid body. r1


m1
(a) Pure Translational motion r2
m2
(b) Pure Rotational Motion
rn mn
(c) Combined Translational and Rotational motion
Let us briefly discuss the characteristics of these three types of motion of a
rigid body. axis of rotation

3.1 Pure Translational motion Figure 7.4: Body in pure


rotational motion.
A rigid body is said to be in pure translational motion if any straight
P hysics | 7.3

line fixed to it remains parallel to its initial orientation all the time. E.g. a car
moving along a straight horizontal stretch of a road. In this kind of motion, the
displacement of each and every particle of the rigid body is the same during
any time interval. All the points of the rigid body have the same velocity and m6
acceleration at any instant. Thus to study the translational motion of a rigid m3
body, it is enough to study the motion of an individual point belonging to that m8
rigid body i.e. the dynamics of a point. m2
m5

3.2 Pure Rotational Motion


m7
m1
m4
Suppose a rigid body of any arbitrary shape rotates about an axis which is
stationary in a given reference frame. In this kind of motion every point of the
body moves in a circle whose center lies on the axis of rotation at the foot of
the perpendicular from the particle to this axis, and radius of the circle is equal
to the perpendicular distance of the point from this axis. Every point of the rigid
body moves through the same angle during a particular time interval. Such a
motion is called pure rotational motion. Each particle has same instantaneous v v
angular velocity (since the body is rigid) and different particles move in circles of
m6
different radii, the planes of all these circles are parallel to each other. Particles v
m8
moving in smaller circles have less linear velocity and those moving in bigger m3 v
circles have large linear velocity at the same instant. v
m2 m5
In the Fig. 7.4 particles of mass m1, m2, m3….. have linear velocities v1, v2, v3…. v
v m7
If ω is the instantaneous angular velocity of the rigid body, then v
m1 m4
v1 = ωr1 , v 2 = ωr2 , v3 = ωr3 ...... , vn = ωrn

Figure 7.5: Body in pure


3.3 Combined Translational and Rotational Motion translational motion.
A rigid body is said to be in combined translational and rotational motion if the body performs pure rotation about
an axis and at the same time the axis translates with respect to a reference frame. In other words there is a reference
frame K’ which is rigidly fixed to the axis of rotation, such that the body performs pure rotation in the K’ frame. The
K’ frame in turn is in pure translational motion with respect to a reference frame K. So to describe the motion of
the rigid body in the K frame, the translational motion of K’ frame is super-imposed on the pure rotational motion
of the body in the K’ frame.

Illustration 1: A body is moving down into a well through a rope passing over a fixed pulley of radius 10 cm.
Assume that there is no slipping between rope and pulley. Calculate the angular velocity and angular acceleration
of the pulley at an instant when the body is going down at a speed of 20 cm s-1 and has an acceleration of 4.0 m
s-2. (JEE MAIN)

Sol: Since the rope does not slip on the pulley, the linear speed and linear acceleration of the rim of the pulley will
be equal to the speed and acceleration of the body respectively.
Therefore, the angular velocity of the pulley is
linear velocity of rim 20 cm s-1
ω= = = 2 rad s-1
radius of rim 10 cm
And the angular acceleration of the pulley is

linear acceleration of rim 4.0 ms-2


α= = = 40 rad s-2
radius of rim 10cm
7 . 4 | Rotational Mechanics

4. ROTATIONAL KINEMATICS
Suppose a rigid body performing pure rotational motion about an axis of rotation rotates by an angle ∆θ in a time
interval ∆t. The instantaneous angular velocity ω, is defined as,
∆θ dθ
ω = Lim = …(i)
∆t→0 ∆t dt
Similarly, the instantaneous angular acceleration α is defined as,
dω d2 θ
α= = 2 …(ii)
dt dt
The relations between linear distance s, linear velocity v and linear acceleration a, and the corresponding angular
variables describing circular motion θ, ω, and α respectively are given as:
s = rθ ; v = rω ; at = rα  …(iii)
Here the subscript t along with a in the expression for acceleration signifies that this is the tangential component
of linear acceleration.
If a body rotates with uniform angular acceleration,
1 2
ω = ω0 + αt ; θ = ω0 t + αt ; ω2 = ω20 + 2αθ …(iv)
2
where ω0 is initial angular velocity.
The equations for angular displacement, angular velocity and angular acceleration are similar to the corresponding
equations of linear motion.

Illustration 2: A disc starts rotating with constant angular acceleration of π / 2 rad s-2 about a fixed axis
perpendicular to its plane and through its center. Calculate
(a) The angular velocity of the disc after 4 s
(b) The angular displacement of the disc after 4s and
(c) Number of turns accomplished by the disc in 4 s.  (JEE MAIN)

Sol:Use the first and second equations of angular motion with constant angular acceleration.
π
=
Here  α rad s−2 ;       ω0 = 0  ;       t = 4 s ;
2
π 
(a) ω(4 s) = 0 +  rad s-2  × 4 s = 2π rad s-1
2 
1π 2 2
(b) θ  rad s  × (16s ) = 4π rad
(4s) = 0 + 2 2 
(c) ⇒ n × 2π rad = 4π rad ⇒ n = 2.

PLANCESS CONCEPTS

For variable angular acceleration we should proceed with differential equation = α
dt
Akshat Kharaya (JEE 2009 AIR 235)
P hysics | 7.5

5. MOMENT OF INERTIA
Before discussing the dynamics of rigid body motion let us study about an important property of a rigid body
called Moment of Inertia which is indispensable in understanding its dynamics.
Physical Significance of Moment of Inertia: As the name suggests, moment of inertia is the measure of the
rotational inertia property of a rigid body, the rotational analog of mass in translational motion. “It is the property
of the rigid body by virtue of which it opposes any change in its state of uniform rotational motion.” The moment
of inertia of a rigid body depends on its mass, on the location and orientation of the axis of rotation and on the
shape and size of the body or in other words on the distribution of the mass of the body with respect to the axis
of rotation. SI units of moment of inertia is Kg-m2. Moment of inertia about a particular axis of rotation is a scalar
positive quantity.
Definition: Moment of inertia of a system of n particles about an axis is defined as:
n
I = m1r12 + m2r22 + .................+ mnrn2 i.e. I = ∑ mi ri2  …(i)
i=1
where, ri is the perpendicular distance of ith particle of mass mi from the axis of rotation.
For a continuous rigid body, the moment of inertia can be calculated as:
I = ∫ r2 (dm) …(ii)
where dm is the mass of an infinitesimal element of the body at a perpendicular distance r from the axis of rotation.
Moment of inertia depends on:
(a) Mass of the rigid body.
(b) Shape and size of the rigid body.
(c) Location and orientation of the axis of rotation.

PLANCESS CONCEPTS
Moment of inertia does not change if the mass:
(i) Is shifted parallel to the axis of rotation because ri does not change.
(ii) Is rotated about the axis of rotation in a circular path because ri does not change.
Chinmay S Purandare (JEE 2012 AIR 698)

Illustration 3: Two particles having masses m1 & m2 are situated in a plane A


perpendicular to line AB at a distance of r1 and r2 respectively as shown.
(i) Find the moment of inertia of the system about axis AB?
(ii) Find the moment of inertia of the system about an axis passing though m1
and perpendicular to the line joining m1 and m2 . r2 r1

(iii) Find the moment of inertia of the system about an axis passing through m2 m1
m1 and m2 .
(iv) Find moment of inertia about an axis passing though center of mass and
perpendicular to line joining m1 and m2 . (JEE MAIN)

Sol: Use the formula for moment of inertia of a system of n particles. Find the B
distance of center of mass from m1.
Figure 7.6
(i) Moment of inertia of particle on right is I1 = m1r12
7 . 6 | Rotational Mechanics

Moment of inertia of particle on left is I2 = m2r22


Moment of inertia of the system about AB is I = I1 + I2 = m1r12 + m2r22
(ii) Moment of inertia of particle on right is I1 = 0
Moment of inertia of particle on left is I2 = m2 (r1 +r2 )2
Moment of inertia of the system about axis is I = I1 + I2 = 0 + m2 (r1 +r2 )2
(iii) Moment of inertia of particle on right is I1 = 0
Moment of inertia of particle of left is I2 = 0
Moment of inertia of the system about axis is I = I1 + I2= 0 + 0
 r +r 
(iv) of system rcm = m2  1 2  = Distance of center mass from mass m1
 m1 + m2 
 r +r 
Distance of center of mass from mass m2 = m1  1 2 
 m1 + m2 
2 2
 r +r   r +r 
So moment of inertia about center of mass = Icm = m1  m2 1 2  + m2  m1 1 2 
 m1 + m2   m1 + m2 

m1m2
Icm = (r + r2 )2 .
m1 + m2 1

Illustration 4: Three particles each of mass m, are situated at the vertices of an equilateral x
triangle PQR of side a as shown in the Fig 7.7. Calculate the moment of inertia of the R
system about
(i) The line PX perpendicular to PQ in the plane of PQR.
(ii) One of the sides of the triangle PQR
(iii) About an axis passing through the centroid and perpendicular to plane of the triangle P a Q
PQR.  (JEE MAIN)
Figure 7.7
Sol: Use the formula for moment of inertia of a system of n particles.
(i) Perpendicular distance of P from PX = 0; perpendicular distance of Q from PX = a perpendicular distance of R
from PX = a/2. Thus, the moment of inertia of the particle at P is 0, that of particle Q is ma2 , and of the particle at
R is m(a/2)2 .
2
The moment of inertia of the three particle system about PX is I = 0 + ma2 + m(a/2)2 = 5ma
4
Note that the particles on the axis do not contribute to the moment of inertia.

(ii) Moment of inertia about the side PR = mass of particle Q × square of perpendicular distance of Q from side PR,
2
 3  3ma2
IPR = m  a =
 2  4
 
a
(iii) Distance of centroid from each of the particles is , so moment of inertia about an axis passing through the
3 2
 a 
centroid and perpendicular to the plane of triangle PQR = IC = 3m 
 3 
= ma2
 
P hysics | 7.7

Table 7.1: Formulae of MOI of symmetric bodies

S. No. Body, mass M Axis Figure I K(Radius of Gyration)


1. Ring or loop of Through its R
MR 2
radius R center and
perpendicular
to its plane
2. Disc, radius R Perpendicular
to its plane MR 2 R
through its 2 2
center

3. Hollow cylinder, Axis of cylinder


MR 2 R
radius R

4. Solid cylinder, Axis of cylinder


radius R MR 2 R
2 2

5. Thick walled Axis of cylinder


cylinder,
(
M R12 + R 22 ) R12 + R 22
2 2
R1
R2

6. Solid sphere, Diameter


radius R 2 2
MR 2 R
5 5

7. Spherical shell Diameter


radius, R 2MR 2 2
R
3 3

8. Thin rod, length L Perpendicular


to rod at ML2 L
middle point 12 2 3
7 . 8 | Rotational Mechanics

9. Thin rod, length L Perpendicular


to rod at one ML2 L
end 3 3

10. Solid cylinder, Through


length l center and MR 2 Ml2 R 2 l2
R + +
perpendicular 4 12 4 12
to length
l

11. Rectangular sheet, Through


length l and center and M(l2 + b2 ) l2 + b2
breadth b perpendicular 12 12
to plane
b

l l

PLANCESS CONCEPTS

While deriving the MOI of any rigid body the element chosen should be such that:
Either perpendicular distance of axis from each point of the element is same or the moment of inertia of
the element about the axis of rotation is known.
Nitin Chandrol (JEE 2012 AIR 134)

5.1 Theorems on Moment of Inertia


1. Theorem of Parallel Axes: This theorem is very useful in cases when the moment Z ZC
of inertia about an axis zC passing through the center of mass (C.O.M) of the rigid body
is known, and we sought to find the moment of inertia about any other axis z which is
parallel to the axis zC as shown in Fig. 7.8. The moment of inertia of the rigid body about
axis z is equal to the sum of the moment of inertia about axis zC and the product of the
mass m of the body by the square of perpendicular distance between the two axes. If the d
COM
moment of inertia of the rigid body about axis zC is IC , then the moment of inertia I of
this body about any parallel axis z , is given by I = IC + Md2 …(i)
where d is the perpendicular distance between the two axes.

Illustration 5: Find the moment of inertia of a uniform sphere of mass m and radius R
about a tangent if the sphere is (i) solid (ii) hollow  (JEE MAIN) Figure 7.8: Parallel axes

Sol: We know the formula for moment of inertia of sphere about an axis passing through its center. Use the parallel
axes theorem to find the moment of inertia about the tangent.
(i) Using parallel axis theorem
P hysics | 7.9

I = IC + md2 R R
For solid sphere
2 7
IC = mR 2 , d = R ; I= mR 2
5 5
(ii) Using parallel axis theorem Solid sphere Hollow sphere
2
I = IC + md
For hollow sphere
Figure 7.9
2 5
IC = mR 2 , d = R ; I = mR 2
3 3


Illustration 6: Find the moment of inertia of the two uniform joint roads having mass m each
P
X
about point P as shown in Fig 7.10. Use parallel axis theorem.  (JEE MAIN)

Sol: We know the formulae for moment of inertia of rod about the axes passing through its
center and through one of its ends and perpendicular to it. Use the parallel axes theorem to
find the moment of inertia about the point P.
Figure 7.10
m2
Moment of inertia of rod 1 about axis P, I1 = P  1
3
2 /2
m2   5/2
Moment of inertia of rod 2 about axis p, I2 = + m 5  COM
12  2

5m2 2
So moment of inertia of a system about axis p; I =
3 Figure 7.11

lz

2. Theorem of Perpendicular Axes: This theorem is applicable only in case of


two dimensional rigid body or planar lamina as shown in Fig. 7.12. Let the lamina
lie in the x-y plane and Ix and I y be the moment of inertia of the lamina about lx
x and y axes respectively then the moment of inertia about z-axis perpendicular
to the plane of the lamina and passing through the point of intersection of x and
y axes is given as:
ly
Iz = Ix + I y …(ii)
Figure 7.12: Perpendicular axes

Illustration 7: Find the moment of inertia of a half-disc about an axis perpendicular to


the plane and passing through its center of mass. Mass of this disc is M and radius is R.
 (JEE MAIN) C.M 4R
 3
Sol: We know the formula for the moment of inertia of the half disc about a perpendicular
lA
axis through the center A. Use the parallel axes theorem to find the moment of inertia
about a perpendicular axis through the center of mass.
The COM of half disc will be at distance 4R/3π from the center A. Let moment of inertia
of half disc about a perpendicular axis passing through A be IA.
Figure 7.13
First we fill the remaining half with same density to get a full disc of mass 2M.
The moment of inertia about center A of full disc will be 2IA,
2 2
2MR 2 MR 2  4R  MR 2  4R 
So, IA = = ; I A = ICM + M ×   ; ICM = − M×  
2×2 2  3π  2  3π 
7 . 1 0 | Rotational Mechanics

Illustration 8: Calculate the moment of inertia of a uniform disc of mass M and B


radius R about a diameter.
 (JEE MAIN) Z

Sol: For a uniform disc all diameters are equivalent, i.e. moment of inertia about any
diameter will be equal to that about any other diameter. We know the formula for C 0 D
moment of inertia of disc about axis perpendicular to its plane and passing through
its center. Use the perpendicular axes theorem to find the moment of inertia about
a diameter.
A
Let AB and CD be two mutually perpendicular diameters of the disc. Take them as x
and y axes and the line perpendicular to the plane of the disc through the center as Figure 7.14
1
the Z – axis. The moment of inertia of the ring about the Z – axis is I = MR 2 . As the
disc is uniform, all of its diameters are equivalent and so I = I 2
x y
I MR 2
From perpendicular an axis theorem Iz = Ix + I y ; hence Ix = z =
2 4

1
Illustration 9: In the Fig 7.15 shown find the moment of inertia of square plate having
I 2
mass m and sides a about axis 2 passing through point C (center of mass) and in the
plane of plate. (JEE MAIN) C
3
Sol: For uniform square plate axes 2 and 4 along diagonals are equivalent and axes 1
I
a
and 3 are equivalent. Suppose IC is the moment of inertia about the axis perpendicular
to the plane of plate and passing through the center C. Use perpendicular axes theorem 4
to prove that the axes 1 and 2 are also equivalent. I a
Using perpendicular axes theorem IC = I 4 + I2 = I' + I'=2I' …. (i)

Figure 7.15
Using perpendicular axes theorem IC = I3 + I1 = I + I = 2I  ….(ii)
From (i) and (ii) we get I' = I
ma2 ma2
IC = 2I = ⇒ I' =
6 12

5.2 Radius of Gyration


The radius of gyration of a rigid body about an axis z is equal to the radius of a ring whose mass is equal to
the mass of the rigid body, and the moment of inertia of the ring about an axis passing through its center and
perpendicular to its plane is equal to the moment of inertia of the rigid body about the axis z. Radius of gyration
can also be defined as the perpendicular distance from the axis of rotation where all mass of the rigid body can be
assumed to be concentrated when the rigid body is performing pure rotation to get the equation of motion of the
body. Thus, the radius of gyration is the ‘’equivalent distance’’ of the rigid body from the axis of rotation.
I = MK 2
I = Moment of inertia of the rigid body about an axis
M = Mass of the rigid body
K = Radius of gyration about the same axis
I
or K= …(iii)
M
Length K is the property of the rigid body which depends upon the shape and size of the body and on the
orientation and location of the axis of rotation. S.I. Unit of K is meter.
P hysi cs | 7.11

Illustration 10: Find the radius of gyration of a hollow uniform sphere of radius R about its tangent. (JEE MAIN)

Sol: Use the formula for radius of gyration.


5
Moment of inertia of a hollow sphere about a tangent = MR 2
3
5 5
MK 2 = MR 2 ⇒ K = R
3 3

5.3 Moment of Inertia of a Body Having a Cavity


If we know the moments of inertia of different parts of a rigid body about the same axis, then the moment of inertia
of the entire body can be calculated by simply adding the moments of inertia of the different parts (about the same
axis) i.e. moment of inertia is an additive quantity. This principle can be used to calculate the moment of inertia of
a body having hollow spaces by first assuming the hollow spaces to be filled with same density as that of the body
and evaluating the moment of inertia of the whole body about the given axis and then add the moments on inertia
of the hollow spaces about the same axis considering them to have negative mass.

Illustration 11: A uniform disc of radius R has a round disc of radius R/3 cut as shown in Fig 7.16. The mass of the
disc equals M. Find the moment of inertia of such a disc relative to the axis passing through geometrical center of
original disc and perpendicular to the plane of the disc.  (JEE ADVANCED)

Sol: Consider the whole disc without the cavity. The cavity can be thought of as a negative mass of same density as
disc. We know the formula for moment of inertia of uniform disc about axis perpendicular to its plane and passing
through its center. Find the moment of inertia of cavity (negative mass) about the perpendicular axis passing
through center of whole disc. The moment of inertia of disc with cavity is the sum of the moment of inertia of whole
disc and the moment of inertia of cavity (negative).
Let the mass per unit area of the material of disc be σ . Now the empty space can be considered as having density
–σ
Now I0 = Iσ + I −σ

Iσ = (σπR 2 )R 2 /2 = MI of σ about O = MR 2 /2

−σπ(R/3)2 (R/3)2
I-σ = + [ −σπ(R / 3)2 ](2R/ 3)2 O
2 R/3
2
R
= M.I of − σ About O = -MR /18
I0 = MR 2 /2 − MR 2 /18
4
I0 = MR 2
9
Figure 7.16
6 TORQUE
6.1 Torque About a Point
Torque of force F relative to a point O is defined as Line of action
   of force
τ = r × F …(x)
  P
where F = force applied to a point on a body

r = position vector of the point of application of force relative to the point O in r
a chosen reference frame about which we want to determine the torque (see Fig. r sin
7.17). O
Torque is a vector quantity and its direction is given by the right hand rule for cross Figure 7.17: Torque of a force
product of vectors.
7 . 1 2 | Rotational Mechanics


Magnitude of torque | τ | = r F sin θ = r⊥F = rF⊥
 
where θ is the angle between the force F and the position vector r of point of application.
r⊥ = r sin θ = perpendicular distance of line of action of force from point O.
 
F⊥ = F sin θ = component of F perpendicular to r
SI unit of torque is N-m.

Illustration 12: Find the torque about point O and A. (JEE MAIN) y
o
30
Sol: Express the position vector of A relative to O in terms of unit vectors î and (1, 1)
o
ˆj . Force is given in terms of unit vectors î and ˆj . 60
    
Torque about point O, τ = r0 × F, r0 = ˆi + ˆj , F = 5 3 ˆi + 5jˆ

τ = (iˆ + ˆj) × (5 3iˆ + 5j)
ˆ = 5(1 − 3)kˆ
     x
Torque about point A, τ = ra × F, ra = ˆj , F = 5 3 ˆi + 5jˆ O A B

τ = ˆj × (5 3iˆ + 5j)
ˆ = -5 3kˆ
Figure 7.18

Illustration 13: A particle of mass m is released in vertical plane from a point on the x0
x – axis, it falls vertically along the y – axis. Find the torque τ about origin?  O P
 (JEE MAIN) 
r
Sol: Torque is produced by the force of gravity. This will be equal to the product of
force of gravity and the perpendicular distance between the line of action of force of

gravity and the origin O.
 
τ = rF sin θ kˆ Or τ =r⊥F = x0mg mg

x0 Figure 7.19
 = r mg = mgx0kˆ
r

6.2 Torque About An Axis


  A
The torque of a force F about an axis AB is the component of the torque of F
about point A along the axis AB. F P

Alternatively to find torque of force F about axis AB we choose any point O on r
    O
the axis AB and find the torque of F about O as τ0 = r × F . Then we calculate
 
the component of τ0 along AB to get τAB (see Fig. 7.20).
B
There are a few special cases of torque of a force about an axis:
  Figure 7.20: Torque about
Case I: Applied force is parallel to the axis of rotation, i.e. F || AB an axis
  
Therefore torque r × F about any point on the axis will be perpendicular to F and hence F A
    P
perpendicular to AB . Therefore the component of r × F along AB will be zero.
r
Case II: The line of action of the applied force intersects the axis of rotation O
 
( F intersects AB )
 
If we choose the point of intersection of line of action of F and AB as the origin O then
 
the position vector r and applied force F will be collinear (see Fig. 7.21). Therefore the B
 
torque about O is r × F = 0 and thus the component of this torque along line AB will Figure 7.21: Force
also be zero. intersects axis
P hysi cs | 7.13

  
Case III: Line of action of F and axis AB are skew and F ⊥ AB A

Let O be the origin on the axis AB and P be the point of application



of force F such that OP is perpendicular to the axis AB (see Fig. 7.22).
   P  
Then torque =τ OP × F will be parallel to axis AB and the component
 r
of τ along AB will be equal to its magnitude i.e. O
F
τAB = F×(OP)sinθ = F×l
where  = (OP)sinθ is the length of the common perpendicular to
the line of action of force and the axis called the lever arm or moment B
arm of this force.
Figure 7.22: Force and axis are skew

Illustration 14: Find the torque of


weight about the axis passing through r
point P.  (JEE MAIN)
m
Sol: Required torque is equal to the r
product of force of gravity and the X
perpendicular distance between the P
line of action of force of gravity and
the point P. 
  F
F = mg Downwards
   Figure 7.23
τ = F × r= F.r sin θ

Illustration 15: A bob of mass m is suspended at point O by string of length  . Bob is moving in a horizontal circle
find out. (i) Torque of gravity and tension about point O and O’ (ii) Net torque about axis OO’. (JEE ADVANCED)

Sol: Torque of a force about an origin is equal to the product of force and the perpendicular distance between the
line of action of force and the origin.
Z
(i) Torque about point O
O
Torque of tension (T), τnet = 0 (tension is passing through point O)
Torque of gravity τmg = mg  sin θ (along negative ˆj )



Torque about point O’ y


o
τmg = mgr = mg  sin θ (along negative ˆj )
x
Torque of gravity m r
v
Torque of tension τT = Tr sin (90 + θ) (T cos θ= mg)
mg Figure 7.24
τT
= Tr cos
= θ ( sin θ)cos
= θ mg sin θ (along positive ˆj )
cos θ
(ii) Torque about axis OO’ F=T

Torque of gravity about axis OO’ τmg = 0 (force mg is parallel to axis OO’) 
Torque of tension about axis OO’ τT = 0 (force T intersects the axis OO’) 90+
O
Net torque about axis OO’ τnet = 0 
r

6.3 Force Couple Figure 7.25

A pair of forces each of same magnitude and acting in opposite directions is called a force couple.
7 . 1 4 | Rotational Mechanics

Torque due to couple = magnitude of one force x distance F


between their lines of action. d

Magnitude of torque = τ = F (d)


A couple does not exert a net force on an object even though Rigid body y2
it exerts a torque.
Net torque due to a force couple is the same about any point
(see Fig. 7.26). y1 B

Total torque about A = x1F + x2F = F(x1 + x2 ) = Fd


F
Total torque about B = y1F - y 2F = F(y1 − y 2 ) = Fd x1 x2
Figure 7.26: Force couple

6.4 Torque on a Rigid Body Executing Pure Rotation


Suppose I is the moment of inertia of a rigid body about the axis of rotation
which is stationary in a given reference frame. The body is executing pure
rotational motion about this fixed axis.
text = resultant torque about the axis of rotation due to all the external forces
acting on the body
α = instantaneous angular acceleration of the body.
ω = instantaneous angular velocity of the body.
Fixed axis of
Consider one particle of the body say ith particle of mass mi at perpendicular Rotation
distance ri from the axis.
Figure 7.27: Rigid body executing
Radial force on the particle Fr = mω2r towards the center of its circular path. pure rotation
Tangential force on the particle Ft = miat = miαri
Torque of the radial force about the axis of rotation is zero as it intersects the axis. Torque of tangential force about
the axis will be,
2
τi = rF
i t = miri α

To find the total torque on the rigid body about the axis we take summation of torques acting on all the particles
of the body. The total torque comes out to be equal to the resultant torque due to external forces only as the
torques due to internal forces cancel each other in pairs when summation is taken on all the particles of the body
(By Newton’s third law of motion internal forces form pairs of equal and opposite collinear forces. So the lever arms
of the forces of a pair with respect to the axis will be equal so their torques will have equal magnitude but opposite
directions and cancel each other in the summation). So

τext= ∑ τi= (∑ miri2 ) α = I α …(i)


i i
Remember: This formula is applicable only for pure rotational motion of a rigid body about a fixed axis.

7. KINETIC ENERGY OF BODY IN PURE ROTATION


When a rigid body performs pure rotational motion about a given axis, all of its constituent particles move in
circular paths with centers on the axis and radii r1 , r2 …… and rn (say), and with linear velocities v1 = ω r1 ,
v 2 = ω r2 ,……….. and vn = ω rn . If m1 , m2 ,….. and mn are the masses of the particles then the total kinetic
energy of the rigid body is given by
1 1 1
K= m v2 + m v 2 + ......... + m v2 …(i)
2 1 1 2 2 2 2 n n
P hysi cs | 7.15

1 1 1
= m1 ω2r12 + m2 ω2r22 + .........+ mnω2rn2
2 2 2
1
= (m1r12 + m2r22 + .......... + mnrn2 )ω2
2
Now as we have learnt the term m1r12 + m2r22 + .......... + mnrn2 is the moment of inertia of the rigid body.

Hence, the rotational kinetic energy of a body is given by


1 2
K= Iω  …(ii)
2

PLANCESS CONCEPTS

Most of the problems involving incline and a rigid body, can be solved by using the conservation of
energy. Care has to be taken in writing down the total Kinetic energy. Rotational Kinetic Energy term has
to be taken into consideration along with translational kinetic energy. And while writing the rotational
energy, the axis about which the moment of inertia is taken should be carefully chosen.
The point about which the conservation is done should be inertial to avoid calculating the work done
by pseudo forces or the point itself should be the COM so that the work done by the torque of pseudo
forces would be 0.
Shrikant Nagori (JEE 2009 AIR 30)

Illustration 16: A uniform circular disc has radius R and mass m. A particle, A
also of mass m, is fixed at a point A on the edge of the disc as shown in Fig
7.28. The disc can freely rotate about a fixed horizontal chord PQ that is at a R
distance R/4 from the center C of the disc. The line CA is ⊥ to PQ. Initially the
disc is held vertical with point A at its highest point. It is then allowed to fall C
so that it starts rotating about PQ. Find the linear speed of the particle as it R/4
reaches lowest point.  (JEE ADVANCED) P Q

Sol: Find the moment of inertia of circular disc and the particle at point A
about the chord PQ. The loss in potential energy of the system comprising the Figure 7.28
disc and the particle will be equal to the gain in its rotational kinetic energy.
2 2
1 mR 2 R   5R  15mR 2 A
I= × + m   + m  =
2 2 4  4  8
Energy equation C
O
5R mgR 1 5R mgR
mg + = Iω2 − mg −
4 4 2 4 4 C
g
ω=4
5R
5Rω A’
V= = 5gR
4 Figure 7.29

Illustrations 17: A pulley having radius r and moment of inertia I about its axis is fixed at the top of an inclined
plane of inclination θ as shown in Fig 7.30. A string is wrapped round the pulley and its free end supports a block
7 . 1 6 | Rotational Mechanics

of mass m which can slide on the plane initially. The pulley is rotated at a
speed ω0 in a direction such that the block slides up the plane. Calculate the
distance moved by the block before stopping?  (JEE ADVANCED)

Sol: Apply Newton’s second law of motion for block M along the inclined M
plane. Find the torque (about its axis) of force of tension acting on pulley.
This will be equal to the product of moment of inertia I and the angular
acceleration of pulley. 
Suppose the deceleration of the block is a. The linear deceleration of the rim
of the pulley is also a. The angular deceleration of the pulley is α = a/r. If the Figure 7.30
tension in the string is T, the equations of motion are as follows:
ma   and Tr = Iα  = Ia / r.         
mg sinθ  – T =
Eliminating T from these equations,
a mgr 2 sin θ
mg sinθ – I ma; Giving, a =
=
r2 I + mr 2
The initial velocity of the block up the incline is v = ω0r Thus, the distance moved by the block before stopping is
2
v2 (I + mr )ω20
x= =
2a 2mg sin θ

Illustration 18: A uniform rod of mass m and length  can rotate in vertical plane 
about a smooth horizontal axis hinged at point H. H X
(i) Find angular acceleration α of the rod just after it is released from initial /2 mg
A
horizontal position from rest?
Figure 7.31
(ii) Calculate the acceleration (tangential and radial) of point A at this moment.
(iii) Calculate net hinge force acting at this moment.
(iv) Find α and ω when rod becomes vertical.
(v) Find hinge force when rod become vertical. (JEE ADVANCED)
Sol: The axis of rotation passing through H is fixed. So the torque of force of gravity about axis through H is equal
to the product of moment of inertia about axis through H and angular acceleration of rod. Angular acceleration at
an instant can be found if the torque of force of gravity at the instant is known.
(i) τH = IH α

 m2 3g 
mg. = α ⇒ α=
2 3 2 X
3g 3g H A
(ii) atA = α = . =
2 2 Figure 7.32
2
aCA = ω r = 0. = 0 (∵ ω = 0 just after release)
N1
(iii) Suppose hinge exerts normal reaction in component form as shown
In vertical direction
Fext = maCM
3g N2
⇒ mg − N1 = m.
4
(We get the value of aCM from previous example) Figure 7.33

mg
⇒ N1 =
4
P hysi cs | 7.17

In horizontal direction
Fext = maCM ⇒ N2 = 0 (∵ aCM in horizontal = 0 as ω = 0 just after release)

(iv) Torque = 0 when rod becomes vertical so α = 0

Using energy conservation mg = 1 Iω2


 m2 
 I= 
2 2 
 3 

(Work done by gravity when COM moves down by (½)  = change in K.E.)
3g
ω=

(v) When rod becomes vertical

3g
α = 0, ω = (Using Fnet = MaCM )

mω2 
FH - mg = ( aCM = centripetal acceleration of COM)
2
5mg
Ans. FH =
2

Bar
Illustration 19: A bar of mass m is held as shown between 4 disks each of mass m’ and radius
r = 75 mm. Determine the acceleration of the bar immediately after it has been released
from rest, knowing that the normal forces exerted on the disks are sufficient to prevent any
slipping and assuming that. (a) m = 5 kg and m’ = 2 kg.
A
(b) The mass m’ of the disks is negligible.
(c) The mass m of the bar is negligible (JEE ADVANCED)
B
Sol: Apply Newton’s second law of motion in vertical direction for the motion of center
of mass of bar. Write the equation of torque due to force of friction acting on disc, for
rotational motion about fixed axis through center of disk. Acceleration of rod will be equal to Figure 7.34
the tangential acceleration of the disc in the case of no slipping.
(a) Equation of center of mass of rod,
mg – 4f = ma ….(i)
(where f is frictional force from one disk)
Torque acting on each disk due to frictional force is
m'r 2 a
fr =  ….(ii)
2 r
From (i) and (ii) we get
mg – 2m’ a = ma  ….(iii)
5g
5g = (5 + 2 × 2)a; a=
9
(b) Putting m’=0 in eqn. (iii) we get a = g
(c) Putting m = 0 in eqn. (iii) we get a = 0
5g
(a) ↓
9
(b) g ↓ c) 0
7 . 1 8 | Rotational Mechanics

7.1 Work Done and Power Delivered by Torque


If a torque τ rotates a body through an angle dθ, the work, dW done by it is given by
dW = τ.dθ
The total work done W in rotating a body from the initial angle θ1 to the final angle θ2 , is
θ2 ω2 ω2
dω 1 2 1 2
W= ∫ τ.dθ = ∫ I
dt
ωdt = ∫ I ω dω = Iω − Iω
2 2 2 1
θ1 ω1 ω1

So the work done by torque is equal to the change in the rotational kinetic energy.
1 
W = ∆Krot = ∆  Iω2  …(i)
2 
This is called the Work-Energy Theorem for rotation of rigid body.
The rate at which work is done is called power P, given by
dW dθ
P == τ = τω …(ii)
dt dt
Also, the power P delivered by the torque on the rigid body is equal to the rate of change of kinetic energy
1 2 dK d 1 2 
K= Iω ∴ P = =  Iω 
2 dt dt  2 
1 dω dω
∴ P = × I × 2ω= I ω=τω
2 dt dt

8. EQUILIBRIUM OF RIGID BODIES


A rigid body can be in linear equilibrium as well as in rotational equilibrium. If a rigid body is in linear equilibrium,
then the vector sum of all the forces acting on it should be zero.

i.e. Σ Fext = 0
Taking scalar components along the three axes x, y and z we get ΣFx = 0, ΣFy = 0, ΣFz = 0
If a rigid body is in rotational equilibrium then the vector sum of all the external torques acting on it with respect
to an axis in a given reference frame must be zero.
Στext = 0 ⇒ Στx = 0, Στy = 0, Στz = 0

Illustrations 20: Two boys weighing 20 kg and 25 kg are trying to balance a seesaw of total length 4 m, with the
fulcrum at the center. If one of the boys is sitting at an end, where should the other sit?  (JEE MAIN)

Figure 7.35

Sol: For rotational equilibrium, the net torque about the fulcrum of all the forces acting on the boys and the seesaw
should be zero.
It is clear that the 20 kg kid should sit at the end and the 25 kg kid should sit closer to the center. Suppose his
P hysi cs | 7.19

distance from the center is x. As the boys are in equilibrium, the normal force between a boy and the seesaw equals
the weight of that boy. Considering the rotational equilibrium of the seesaw, the torque of the forces acting on it
should add to zero. The forces are
(a) (25kg) g downward by the 25 kg boy
(b) (20kg) g downward by the 20 kg boy
(c) Weight of the seesaw and
(d) The normal force by the fulcrum.
Taking torques about the fulcrum.
(25 kg) g x = (20 kg) g (2 m) or x = 1.6 m

9. ANGULAR MOMENTUM
9.1 Angular Momentum of a Particle About a Point

If p is the linear momentum of a particle in a given reference frame, then angular momentum of the particle about
an origin O in this reference frame is defined as
  
L = r × p …(i)

where r is the position vector of the particle with respect to origin O (see Fig. 7.36).
Magnitude of angular momentum =
is L r p sinθ Pcos
or L = r⊥ p or L = p⊥ r  
  P
θ = angle between vectors r and p Psin


 r
r⊥ = component of position vector r perpendicular to vector p .
 
p⊥ = component of vector p perpendicular to position vector r . o
2
SI unit angular momentum is kg m s . -1
Figure 7.36: Angular
momentum about a point
Relation between Torque and Angular Momentum
  
∵ L = r × p
Differentiating with respect to time we get
  
dL dr   dp       
= × p+r × = v × (mv) + r × F = 0 + r × F = τ
dt dt dt

dL 
⇒ = τ  …(ii)
dt
For a single particle moving in a circle of radius r with angular velocity ω we have
v = ωr and p = mωr
So angular momentum comes out to be L = r p= mr2ω

Y
Illustration 21: A particle of mass m is projected at time t = 0 from a
point O with a speed u at an angle of 45° to the horizontal. Calculate the v
magnitude and direction of the angular momentum of the particle about
the point O at time t = u/g.  (JEE ADVANCED) r
X
Sol: Express the position and velocity of particle in Cartesian coordinates O
in terms of unit vectors î and ˆj and then calculate the cross product in Figure 7.37
7 . 2 0 | Rotational Mechanics

Cartesian coordinates.
Let us take the origin at O, X –horizontal axis and
Y – Axis along the vertical upward direction as shown in Fig 7.37 for horizontal during the time 0 to t.
u u u2
v x = u cos45°= u/ 2 and x = v x t = . =
2 g 2g
For vertical motion,
u (1 − 2 )
v y = u sin 45° - gt = − u= u
2 2
1 u2 u2 u2
and y = (u sin 45 º) t − gt2 = − = ( 2 − 1)
2 2g 2g 2g
The angular momentum of the particle at time t about the origin is
    
L r × P = m r × v = m(ix ˆ + ˆjy) × (iv
ˆ + ˆjv ) = m(kxv ˆ ˆ
= x y y − kyv x )

 u2  u u2 u2  mu3
= m kˆ   (1 − 2) − ( 2 − 1)  = − kˆ
 2g  2 2g 2  2 2g
 
mu3
Thus, the angular momentum of the particle is in the negative z – direction i.e., perpendicular to the plane
of motion, going into the plane. 2 2g

Illustration 22: A cylinder is given angular velocity ω0 and kept on a horizontal rough surface the initial velocity is
zero. Find out distance travelled by the cylinder before it performs pure rolling and work by frictional force. 
 (JEE ADVANCED)
Sol: Due to backward slipping force of friction will act forwards. The cylinder is accelerated forwards. The torque
due to friction and hence the angular acceleration is opposite to the initial angular velocity. So the angular velocity
will decrease and the linear velocity of center of mass of cylinder will increase in the forward direction, till the
slipping stops and pure rolling starts. The work done by frictional force is equal to change in the kinetic energy of
the cylinder. The kinetic energy includes both rotational kinetic energy and translational kinetic energy.
MR 2 α
µMg R =
2
2µg 0 v
α=
R  … (i) R

Initial velocity u = 0 a v
R
2 2
v = u + 2as
fk
v 2 = 2as  … (ii)
fK = ma; µMg = Ma ;
S
a = µg … (iii)
Figure 7.38
ω = ω0 − αt
2µg
From equation (i) ω = ω0 − t ; V = u + at
R
From equation (iii) v = µ g t
2v ω
ω = ω0 − ; ω = ω0 − 2ω ; ω = 0
R 3
From equation (ii)
2
 ω0R   ω 2R 2 
  = (2as) = 2µ gs ; S=  0 
 3   18µg 
 
P hysi cs | 7.21

Work done by the frictional force

µmg × ω20R 2
W = ( − fkRdθ + fk ∆s) = − µmgR∆θ + ;
18µg
  ω0R  1 2µg  ω0R  
2
 ω2R ω2R 
1 2
×
 2ω20R
∆θ = ω0 × t − αt =  0 
ω  ×    =  0 − 0  =
2   3µg  2 R  3µg    3µg 9µg  9µg
  
 2ω20R   ω2R 2   mω20R 2
W =  −µmg × R 
 +  µmg × 0   = −
 9µg 



18µg  

6

Illustration 23: A hollow sphere is projected horizontally along a rough surface with speed v and angular velocity
v
ω0 . Find out the ratio , so that the sphere stops moving after some time.  (JEE ADVANCED)
ω0
Sol: For the sphere to stop after sometime, the acceleration should be opposite to velocity, i.e. the force of friction
should be backwards (forward slipping). Also, the torque due to friction should be opposite to angular velocity, i.e.
if the torque due to friction is clockwise (see Fig. 7.39), then the initial angular velocity should be anti-clockwise.
Torque about lowest point of sphere
2 3 ωg
fK × R = Iα ; µmg × R = mR 2 α ; α= (Angular acceleration in opposite direction of angular velocity)
3 2R
0
ω = ω0 − αt (Final angular velocity ω = 0)
a=0
3 ωg ω0 × 2R a v v=0
ω0 = × t ; t=
2R 3ì g
Acceleration a = μg ft v
v t = v – at       (Final velocity v t = 0); 0R
v Figure 7.39
v = µg × t ; t =
µg
To stop the sphere, time at which v and ω are zero, should be same.

v 2ω0R v 2R
= ; ⇒ =
µg 3ì g ω0 3

Illustration 24: A rod AB of mass 2m and length  is lying m m v1=0 A


v0 A
on a horizontal frictionless surface. A particle of mass m
travelling along the surface hits the end of the rod with a
velocity v 0 in a direction perpendicular to AB. The collision /2 /2
is elastic. After the collision the particle comes to rest. Find
out after collision
2m COM COM v
(a) Velocity of center of mass of rod 
(b) Angular velocity.  (JEE ADVANCED) /2 /2

Sol: Conserve linear momentum and angular momentum of


the system constituting “the rod and the particle” before and B B
after collision. Here the linear and angular momentum of the Before collision After collision
rod before collision is zero. Angular momenta of the rod and
Figure 7.40
particle are calculated about the center of the rod.
7 . 2 2 | Rotational Mechanics

(a) Let just after collision the speed of COM of rod is v and angular velocity about COM is ω .
External force on the system (rod + mass) in horizontal plane is zero.
Apply conservation of linear momentum in x direction;
mv 0 = 2mv  ….. (i)
(b) Net torque on the system about any point is zero
Apply conservation of angular momentum about COM of rod.
  2m2
mv 0 = Iω ⇒ mv 0 = ω  …... (ii)
2 2 12
v0
From equation (i) velocity of center of mass v =
2
3v 0
From equation (ii) angular velocity ω = .

9.2 Angular Momentum of a Rigid Body Rotating About Fixed Axis


For a system of particles the total angular momentum about an origin is the sum of the angular momenta of all the
particles calculated about the same origin.
  A
L =∑ Li 
i
Differentiating with respect to time we get, r1
  m1
dL dLi     
dt
= ∑ dt = ∑ (∑ τik + τiext ) = ∑∑ τik + ∑ τiext = 0 + τext r2
i i k i k i m2
The double summation term corresponds to the sum of torques due to rn
internal forces and as explained earlier, according to Newton’s third law mn
of motion these internal torques cancel out in pairs.
So for a system of particles
 Axis of rotation
dL  B
= τext …(xvii)
dt
 
Impulse of a torque is defined as J = ∫ dL= ∫ τext dt Figure 7.41: Angular momentum
of rigid body
Angular momentum of a rigid body rotating about a fixed axis can be
calculated as below:
Angular momenta of its individual particles about the axis are

L1 = m1r12 ω, L2 = m2r22 ω, L3 = m3r32 ω, .... Ln = mnrn2 ω where ω is the instantaneous angular velocity of the
rigid body
Total angular momentum of the body
L = m1r12 ω + m2r22 ω + m3r32 ω ............+ mnrn2 ω
L= ∑ mi (ri )2ω = I ω
i

So L = I ω
Remember: This formula is applicable only for rotation of the rigid body about a fixed axis.
Again differentiating this relation with respect to time we get,
dL dω
=I = I α = τext
dt dt
P hysi cs | 7.23

Illustration 25: Two small balls of mass m each are attached to a light rod of length  , one at its center and the
other at its free end. The rod is fixed at the other end and is rotated in horizontal plane at an angular speed ω .
Calculate the angular momentum of the ball at the end with respect to the ball at the center.  (JEE MAIN)

Sol:Both the balls A and B have same angular velocity but different linear velocities.

The situation is shown in Fig 7.42. The velocity of the ball A with respect to
the fixed end O is v A = ω ( /2) and that of B with respect to O is vB = ω A
. Hence the velocity of B with respect to A is vB – v A = ω( /2) . The angular B
O 
momentum of B with respect to A is, therefore,
 1 Figure 7.42
L = mvr = mω   = mω2
22 4
along the direction perpendicular to the plane of rotation.

9.3 Conservation of Angular Momentum


In the previous article we have proved the relation
 
dL   ext
= τext where L and τ are evaluated about the same origin.
dt 

From the above equation we see that if τext =
0 then L of the system of particles remains constant.
In some situations the component of external torque about an axis is zero even if the net external torque is not
zero. So in these cases the component of the total angular momentum, about the particular axis, remains constant.

Illustration 26: A uniform rod of mass m and length  can rotate freely on a smooth horizontal plane about a
vertical axis hinged at point H. A point mass having same mass m coming with an initial speed u perpendicular to
the rod strikes the rod in-elastically at its free end. Find out the angular velocity of the rod just after collision? 
 (JEE MAIN)

Sol: After collision the rod and the particle execute pure rotational motion about vertical axis ml
through fixed point H. +
H
Angular momentum is conserved about H because no external force is present in horizontal u
plane which is producing torque about H.
 m2  3u m
mul =  + m2  ω ⇒ ω=
 3  4
  Figure 7.43

Illustration 27: A uniform rod of mass m1 and length  lies on a frictionless horizontal plane. A particle of mass
m2 moving at a speed v 0 perpendicular to the length of the rod strikes it at a distance  /3 from the center and
stops after the collision. Calculate (a) the velocity of the center of the rod and (b) the angular velocity of the rod
about its center just after the collision. (JEE ADVANCED)
w
Sol: Conserve the linear momentum of the system comprising
“the rod and the particle” before and after the collision. Conserve R

the angular momentum, about the center of the rod, of the system A  v
A
comprising “the rod and the particle” before and after the collision. /3
The situation is shown in the Fig 7.44. Consider the rod and the
particle together as the system. As there is no external resultant force,
the linear momentum of the system will remain constant. Also there (A) (B)
is no resultant external torque on the system and so the resultant
Figure 7.44
external torque on the system and the angular momentum of the
7 . 2 4 | Rotational Mechanics

system about the line will remains constant. Suppose the velocity of the center of the rod is V and the angular
velocity about the center is ω .
(a) The linear momentum before the collision is m2v0 and that after the collision is M1V.
m 
Thus m2 v 0 = m1 v, or V =  2  v 0
m 
 1
(b) Let A be the center of the rod when at rest. Let AB be the line perpendicular to the plane of the Fig 7.44.
Consider the angular momentum of N “the rod plus the particle” system about AB.
Initially the rod is at rest. The angular momentum of the particle about AB is
L = m2 v 0 ( /3)
After collision the particle comes to rest. The angular momentum of the rod about a is
   
L = LCM + m1 r0 × v
     
As r0 || v , r0 × v = 0 thus, L = LCM

Hence the angular momentum of the rod about AB is


m1 2 m2 v m1 2 4m2 v 0
L L==Iùlw= ω Thus, = ω Or ω=
12 3 12 m1 

10. RIGID BODY IN COMBINED TRANSLATIONAL AND ROTATIONAL


MOTION
As discussed earlier, in this type of motion the rigid body is performing pure rotational motion about an axis and
the axis itself is performing pure translational motion relative to a given reference frame.
Consider a car moving over a straight horizontal road with some instantaneous velocity v with respect to a reference
frame K fixed to the road. Now let us observe the motion of a wheel of the car from the K frame. This motion of the
wheel in K frame is an example of combined translational and rotational motion. Let us suppose a reference frame
K’ which is translating with respect to frame K with same instantaneous velocity v. In other words frame K’ is rigidly
fixed to the body of the car. In this frame the wheel of the car performs pure rotational motion. The body of the car
itself is performing pure translational motion.
Take another example of motion of a fan fixed inside the car.
If the fan is switched off while the car is moving on the road, the motion of fan is pure translational with respect
to K frame.
If the fan is switched on while the car is at rest, the motion of fan is pure rotational about its axis, as the axis is at
rest in the K frame.
If the fan is switched on while the car is moving on the road, the motion of the fan with respect to K frame is neither
pure translational nor pure rotational but a combination of both. Now if an observer A is sitting inside the car, as
the car moves, the motion of fan will appear to him as pure rotational while the motion of the observer A with
respect to K frame is pure translational. Hence in this case we can see that the motion of the fan can be resolved
into two components, pure rotational motion relative to observer A and pure translational motion of observer A
relative to K frame.
Such a resolution of motion of a rigid body into components of pure rotational and pure translational motion is an
important tool used in the study of their dynamics.

10.1 Kinematics of a General Rigid Body Motion


For a rigid body the value of angular displacement θ, angular velocity ω, and angular acceleration α is same for
all points on the rigid body. Also, if we choose any point of the rigid body as origin O and any other point, say A,
P hysi cs | 7.25


of the body has a position vector r relative to O, and during any 
vA

time interval the vector r rotates by an angle θ relative to its initial 
direction, then position vector of any other point, say B, relative to
  
vB = vA + vBA
any other origin, say O’, inside the rigid body will also rotate by the B
same angle θ. This means the angular variables θ, ω, and α do not
depend on the choice of origin in the rigid body.
  
vBA =  x rBA

The above concept is very important as it enables us to calculate 


vA
the velocity of each point of the rigid body if we know the velocity
A
of any one point (say A) in the rigid body with respect to a reference rigid body
frame K and angular velocity of any point in the rigid body relative
to any other point in the rigid body.
Suppose we want to calculate thevelocity of a point B in the rigid
Figure 7.45: Kinematics of rigid body
body which has a position vector rBA relative to A (see Fig. 7.45).

The velocity of point A is v A , so we have velocity of B as
     
vB = v A + vBA = v A + ω × rBA

Direction of ù is given by right hand thumb rule. If we curl the fingers of the right hand in the direction of rotation

 ω .
of the body, thumb gives the direction of  
Similarly the acceleration of point B is: aB = aA + α × rBA

Illustration 28: Consider the general motion of a wheel (radius r) which can be viewed as pure translation of its
center O (with the velocity v) and pure rotation about O (with angular velocity ù )
       
Find out v AO , vBO , v CO , vDO and v A , vB , v C ,vD  (JEE MAIN)

Sol: Express the angular velocity, linear velocity of point O and C j


position vectors of points A, B and C relative to O in Cartesian =(-k)
coordinates.
   
v AO = (ω × r AO ) = ω( −k) ( ˆ × OA
) D

V B
i

= ω( −k)

( 
)
ˆ × r( −ˆj) = −ωri

O
ˆ ; vDO = ωr(ˆj)
Similarly vBO = ωr ( −ˆj) ; v CO = ωr(i)
  
v A = v O + v AO = v ˆi − ωriˆ ; A
   Figure 7.46
vB = v O + vBO = v ˆi + ωrjˆ
     
v C = v O + v CO = v ˆi + ωr ˆi ; vD = v O + vDO = v ˆi + ωrjˆ

10.2 Dynamics of a General Rigid Body Motion


Combined rotation and translation of a rigid body is considered as combination of pure rotation in C frame about
an axis passing through the center of mass and translation of center of mass in a reference frame K. Dynamics of
combined rotational and translational motion of a rigid body in K frame is defined by two vector equations. One
of them describes the dynamics of the center of mass of the rigid body in the K frame, and the other the equation
of dynamics of pure rotation of the body about center of mass in the C frame.

So if the total mass of the rigid body is M and the net external force acting on it is Fext then we have in the K frame,

dVC 
M = Fext  …(i)
dt

If IC is the moment of inertia of the rigid body about the axis passing through center of mass and τC is the net
torque of all external forces about the axis passing through the center of mass, then we have in the C frame,

 dω 
τC = IC = IC α …(ii)
dt
7 . 2 6 | Rotational Mechanics

 
If Ptotal is the total linear momentum
 of the rigid body in the K frame, L C is angular momentum of the body in C
frame about center of mass and r C is the position vector of center of mass relative to some origin in K frame, then
we have,
 
Ptotal = MVC
Total Kinetic energy
1 1
K = MVC2 + IC ω2  …(iii)
2 2
 
LC = IC ω  …(iv)
 
Angular momentum in K frame = LC about C.O.M + L of the C.O.M about some origin in K frame
    
L = IC ω + r C × MVC …(v)

10.3 Pure Rolling (Rolling Without Slipping)


Pure rolling is a special case of combined translational and rotational motion of a rigid body with circular cross
section (e.g. wheel, disc, ring, cylinder, sphere etc.) moving on a surface. Here, 
there is no slipping between the rolling body and the surface at the point of R
contact.
V
Suppose a sphere rolls on a stationary surface and the point of contact between
the sphere and the surface is A (see Fig. 7.47). Let the velocity of the center of
sphere be v, radius be R and its angular velocity be ω. For pure rolling the relative A
velocity between the point A of the sphere and the surface must be zero. As the
surface is at rest, the velocity of point A is also zero. Figure 7.47: Sphere rolling on a
stationary surface.
∴ v A = v - ωR = 0
∴ v = ωR
If sphere is rolling on a plank moving velocity v0, then for pure rolling, v A = v - ωR = v 0 (see Fig. 7.48)
Same is true for the tangential acceleration of the point of contact in case 
of pure rolling.
Now let’s discuss the case where a rolling cylinder of mass m moves forward
V
on a rough plate of same mass with acceleration “a” and the rough plate
moves forward with an acceleration “a0” under action of force F on a smooth
surface. A
As the cylinder accelerates in the forward direction, so by Newton’s second V0
law, the friction on the cylinder at the point of contact will be in forward
direction and on the plate in backward direction by Newton’s third law (see Figure 7.48: Sphere rolling on a
Fig. 7.50). moving surface.

Equation of torque about center of cylinder:


mR 2
τ=
C fR
= α  a
2 m Rough
2f
⇒ α=  ……. (i)
mR Smooth
a0
m F
Equation of motion of center of cylinder:
f = ma  …… (ii)
From (i) and (ii) we get Figure 7.49: Cylinder rolling on an accelerating plate.
αR
a=
2
P hysi cs | 7.27

At contact point a0
3αR f
a0 = a + aR = = 3a …… (iii) a
2
 m m F
Equation of motion of plate: f
F – f = ma0
Figure 7.50: (a) FBD of Cylinder. (b) FBD of Plate.
F = m(a + a0)
F 3F
F = 4ma ; a = ; a0 =
4m 4m

Illustration 29: A wheel of radius r rolls (rolling without slipping) on a level B 


road as shown in fig 7.51.
r
Find out velocity of point A and B.  (JEE MAIN) v

Sol: Linear velocity of any point on the rim of the wheel has magnitude ωr in A
the reference frame of center of wheel (C-frame). Velocity in ground frame is
the vector sum of velocity in C-frame and the velocity of center of wheel.
Figure 7.51
Contact point at surface is in rest for pure rolling
Velocity of point is A zero.
So v = ω r
Velocity of point B = v + ω r = 2v

Illustration 30: A uniform sphere of mass 200 g rolls without slipping on a plane surface so that its center moves
at a speed of 2.00 cm s-1. Find its kinetic energy. (JEE MAIN)
Sol: The kinetic energy of sphere is the sum of the translational kinetic energy and the rotational kinetic energy.
As the sphere rolls without slipping on the plane surface its angular speed about center is

v 1 1 1 2 1
ω = cm . The kinetic energy is K = Icmω2 + Mv cm2 = . Mr 2 ω2 + Mv cm2
r 2 2 2 5 2

1 1 7 7
= Mv cm2 + Mv cm2 = Mv cm2 = (0.200 kg)(0.02 m s-1 )2 = 5.6 × 10−5 J
5 2 10 10

Illustration 31: A constant force F acts tangentially at the highest point of a uniform disc of mass m kept on a
rough horizontal surface as shown in Fig 7.52. If the disc rolls without slipping, calculate the acceleration of the
Center C and point A and B of the disc.  (JEE ADVANCED)

Sol: Apply Newton’s second law for the motion of center of mass of the disc. B
F
Find the torque of the force F and the force of friction acting on the disc at point r
A about the center of mass of the disc and thus obtain the equation relating C
the angular acceleration in the C-frame to the torques of all the external forces.
The situation is shown in Fig 7.52. As the force F rotates the disc, the point of
contact has a tendency to slip towards left so that the static friction on the A rough surface
disc will act towards right. Let r be the radius of the disc and be the linear
acceleration of the center of the disc. The angular acceleration about the center Figure 7.52
of the disc is
α = a/r, as there is no slipping.
7 . 2 8 | Rotational Mechanics

For the linear motion of the center,


F + f = ma  …….. (i)
And for the rotation motion about the center,

1  a  1
Fr – f r = I α = Fr – f r = I =  mr 2    or F – f = ma …….. (ii)
 2  r  2 
From (i) and (ii),
3 4F
2F = ma or a =
2 3m
Acceleration of point A is zero

 4F   8F 
Acceleration of point B is 2a = 2   =   .
 3m   3m 

Illustration 32: A circular rigid body of mass m, radius R and radius of gyration (k) rolls without slipping on an
inclined plane of an inclination θ. Find the linear acceleration of the rigid body and force of friction on it. What must
be the minimum value of coefficient of friction so that rigid body may roll without sliding?
 (JEE ADVANCED)

Sol: Apply Newton’s second law for the motion of center of mass of the rigid a/R
body. Find the torque of the force F and the force of friction acting on the rigid
body about the center of mass of the disc and thus obtain the equation relating
the angular acceleration in the C-frame to the torques of all the external forces. f
If a is the acceleration of the center of mass of the rigid body and f the force of mgcos
friction between sphere and the plane, the equation of translational and rotational
mg mgsin
motion of the rigid body will be
ma (Translational motion)
Mg sin θ – f =

fR = I α (Rotational motion)
Figure 7.53

f= I = m k 2 , due to pure rolling a = α R
R

Iα Iα mk 2 α R2 + k2 
mg sin θ − = m αR = mαR +=  ma + = a 
R R R  R
2


gsinθ gsin θ Iα mk 2a mg k 2 sin θ


a =  = ; f = = ⇒
 R2 + k2   k2  R R2 R2 + k2
 2   1 + 2 
 R   R 

mk 2
f ≤ mN; a ≤ µ ≤ mg cos θ
R2
k2 g sin θ tanθ tanθ
R2 × ≤ µg cos θ ; µ ≥ ; µmin ≥
R 2
(
k2 + R2 )  R 
1+ 2 
2  R2 
1+ 2 
 k   k 
P hysi cs | 7.29

PLANCESS CONCEPTS

•• From above example if rigid bodies are solid cylinder, hollow cylinder, solid sphere and hollow sphere
(having radius ‘r’ and mass ‘m’)
•• Increasing order of acceleration

a >a > a >a


solid sphere hollow sphere solid cylinder hollow cylinder

•• Increasing order of required friction force for pure rolling

f >f >f >f


hollow cylinder hollow sphere solid cylinder solid sphere
•• Increasing order of required minimum friction coefficient for pure rolling

µ >µ >µ >µ


hollow cylinder hollow sphere solid cylinder solid sphere

•• I would advise you to derive these, verify and remember!


Anand K (JEE 2011 AIR 47)

10.4 Instantaneous Axis of Rotation


The combined translational and rotational motion of a rigid body can be reduced to a purely rotational motion.
When we know the velocity VC of the center of mass and the instantaneous angular velocity ω of the body then
we can find a point whose velocity comes out to be zero at a given moment of time. The axis passing through this
point at the given moment is called instantaneous axis of rotation and the rigid body performs pure rotation about
this axis with same angular velocity at that moment.
The position of the instantaneous axis of rotation changes with time. E.g. in pure rolling the point of contact with
the surface is the instantaneous axis of rotation (see Fig. 7.54).

 (l.A.R.)
V+r=2v
V2
rig
id

v
ro
d

2v
P Instantaneous axis
V1

Figure 7.54: IAR (a) pure rolling; (b) Rod slipping down a wall

Geometrical construction of instantaneous axis of rotation (I.A.R). If we know the velocity vectors of any two
points in the rigid body then the I.A.R. is the axis passing through the point of intersection of the perpendiculars
drawn to the velocity vectors at those points.
Once location of I.A.R is known, we find the moment of inertia of the body about this axis, and then the equations
of rotation about fixed axis can be used for this axis.
0

Illustration 33: Prove that kinetic energy = 1/2 IP ω2  (JEE MAIN)


v
Sol: Kinetic energy is the sum of the translational kinetic energy and the rotational kinetic v=R
energy.
P
Figure 7.55
7 . 3 0 | Rotational Mechanics

1
K.E. =
1
2
1
2
1
Icmω2 + Mv cm2 = Icmω2 =
2
1
+ Mω2R 2
2 2
(Icm + MR 2 )ω2 =
1
I
2 contact point
ω2 ( )
Notice that in pure rolling of uniform object, equation of torque can also be applied about the contact point.

Illustration 34: A uniform bar of length  and mass m stands vertically


touching a vertical wall (y – axis). When slightly displaced, its lower end
begins to slide along the floor (x – axis). Obtain an expression for the angular
velocity ( ω ) of the bar as a function of θ . Neglect friction everywhere.
 (JEE ADVANCED)

Sol: As the rod falls, it executes pure rotational motion about the instantaneous
axis of rotation. The loss in gravitational potential energy is equal to the gain
in the rotational kinetic energy.

The position of instantaneous axis of rotation (IAOR) is shown in Fig 7.57.
Figure 7.56
   
C =  cosθ, sinθ  ; r= = half of the diagonal
2 2  2

All surfaces are smooth, therefore, mechanical energy will remain conserved.
∴ Decrease in gravitational potential energy of bar = increase in rotational
kinetic energy of bar about IAOR.
(lAOR)
 1
mg (1 – sinθ) = Iω2  ... (i) A

2 2 r
2
m
Here, I = +mr 2 (about IAOR) or I
12 C

2 2 2 O
m m m B
= + = Substituting in Eq. (i)
12 4 3 Figure 7.57

 1  m2  2 3g(1 − sin θ)


We have mg (1 – sinθ) =   ω or ω =
2 2  3  

PLANCESS CONCEPTS

Nature of friction for rigid bodies:


•• A rigid body rolling with a speed of v and angular velocity of ω at an instant. Then it falls under one of the
following cases.

Cases Rough/Smooth Diagram Inference


V<rω Rough Surface 1. There is relative motion at point of contact.
 With respect to the body the surface moves
v slower than itself. So the surface tries to
decrease its angular velocity by a frictional force
fk in forward direction. And this friction is kinetic
Rough surface
friction.

2. It increases v and decreases ω So, after


sometime, v =r ω and pure rolling will resume.
P hysi cs | 7.31

PLANCESS CONCEPTS

Cases Rough/Smooth Diagram Inference


Smooth Surface No friction is possible and it is not pure rolling.

v

Smooth surface
Rough surface With respect to the COM of the cylinder, the
surface moves at a higher speed than itself. So
v > rω 
v the surface tries to increase its angular velocity
by exerting a frictional force in backward
Rough surface direction. And this friction would be kinetic
friction.

2. The friction tries to reduce V and increase ω


V>rω Smooth Surface No friction and no pure rolling.

v

Smooth surface

V=rω Rough Surface This is pure rolling. However there might be


 static friction acting on the body.
v

Smooth surface

Smooth Surface No friction is possible and it is pure rolling



v

Smooth surface

Rohit Kumar (JEE 2012 AIR 79)

Illustration 35: A rigid body of mass m and radius r rolls without slipping on a surface. A force is acting on the rigid
body at x distance from the center as shown in Fig 7.58. Find the value of x so that static friction is zero. 
 (JEE MAIN)
Sol: For static friction to be zero, the linear and angular accelerations a and α
caused by the force F should be related as a = α R, for rolling without slipping.

P F
x
Torque about center of mass Fx = Icm α  … (i)
a
o
a=R
F = ma  … (ii) f
Rough surface
From equation (i) & (ii) max = Icmα (a = α R) ;
I Figure 7.58
x = cm
mR
7 . 3 2 | Rotational Mechanics

Illustration 36: There are two cylinders of radii R₁ and R2 having moments of
inertia I₁ and I2 about their respective axes as shown in Fig 7.59. Initially, the 1 2
cylinders rotate about their axes with angular speed w1 and w2 as shown in the R1
11 R2 12
Fig 7.59. The cylinders are moved close to touch each other keeping the axes
parallel. The cylinders first slip over each other at the contact but the slipping
finally ceases due to the friction between them. Calculate the angular speeds Figure 7.59
of the cylinders after the slipping ceases.  (JEE ADVANCED)

Sol: The force of friction acting on the cylinder moving faster will be such that its angular velocity decreases. The
force of friction acting on the cylinder moving slower will be such that its angular velocity increases. When slipping
ceases, the linear speeds of the points of contact of the two cylinders will be equal.
If w'1 and w'2 be the respective angular speeds at the instant slipping ceases, we have

w'1 R1 = w'2 R2  …(i)

The change in the angular speed is brought about by the frictional force which acts as long as the slipping exists. If
this force f acts for a time t. the torque on the first cylinder is fR1 and that on the second is fR2. Assuming w1 > w2.
The corresponding angular impulses are – fR1 t and fR2 t,
We therefore, have

– f R1 t = I1 (ω`1 −ω1 ) and f R 2 t = I2 (ω`2 −ω2 )

I1 I2
or − (ω`1 −ω1 ) = (ω`2 −ω2 )  …(ii)
R1 R2

I ω R + I2 ω2R1 I ω R + I2 ω2R1
Solving (i) and (ii) ω '1 =1 1 2 R 2 and ω '2 =1 1 2 R1 .
2 2
I2R1 + I1R 2 I2R12 + I1R 22

10.5 Energy Method in Solving Problems of Rolling Body


We can conserve energy in case of pure rolling of a rigid body because the point of
contact between the surfaces remains at rest and so the frictional forces acting at
the point of contact do not do any work. Thus only conservative force do work on
the body. S
h
Thus Potential energy + total K.E. = constant
As shown in the Fig. 7.60, a disc is rolling down on an inclined plane. Then we can 
conserve total mechanical energy. If the disc falls a height h then loss in potential
energy is equal to gain in kinetic energy. Figure 7.60

1 1
Mgh = Iω2 + MV 2  … (i)
2 2 C
1  K2 
1 1 2  1+  
Its total kinetic energy = MV 2 + Iω2 = MVC …(ii)
2 C 2 2  R2 
 

where K is the radius of gyration of the disc and VC the velocity of center of mass.

1  K2  2gh
So MVC2  1 + 2 Mgh;
 = VC2 =
2  R   K2 
 1 + 2 
 R 

Thus the velocity of center of mass of a body rolling down an inclined plane is given by
P hysi cs | 7.33

2gh
VC = 1/2
 K2 
1 + 2 
 R 

If aC is linear acceleration of center of mass down this plane, and distance covered on the plane is s, then if the body
starts from rest we have

2 VC2 2 gh g sin θ
VC = 2aCs ∴ aC = = or aC =
2s  K 
2
h K2
 1 + 2  × 2 × 1+ 2
 R  sin θ R

PLANCESS CONCEPTS

Rather than going in a conventional way, using this method greatly simplifies our effort. But take care
while writing the kinetic energy!
Nitin Chandrol (JEE 2012 AIR 134)

Illustration 37: A solid sphere is released from rest from the top of an incline of inclination θ and length  . If the
sphere rolls without slipping. What will be its speed when it reaches the bottom?  (JEE MAIN)
Sol: The loss in the gravitational potential energy of the solid sphere is equal to the gain in the kinetic energy.
The kinetic energy of the sphere comprises the rotational kinetic energy as well as the translational kinetic energy.
Let the mass of the sphere be m and its radius be r. Suppose the linear speed of the sphere when it reaches the
bottom is v. As the sphere rolls without slipping, its angular speed ω about its axis is v/r. The kinetic energy at the
bottom will be
1 2 1 12  1 1 1 7
K= Iω + mv 2 =  mr 2  ω2+ mv 2 = mv 2 + mv 2 = mv 2
2 2 25  2 5 2 10
This should be equal to the loss of potential energy mg  sinθ. Thus

7 10
mv 2 = mg sinθ Or v= g sinθ .
10 7

11. TOPPLING
When an external force is applied to the upper edge of a body Q R
with a flat base to cause it to slide along a surface, the body may Ftopple
topple before sliding starts. Toppling is more likely to happen
when the width of the base of the body is small.
Toppling occurs due to the turning effect of torques of applied
force at the upper edge and frictional force at the base. mg N
Let the surface be quite rough and the force F is applied at f P S
h
height h above the base of the block as shown in Fig. 7.61. Width
of the base is b. The static friction at the base is f = F. The normal Rough surface
reaction is N = mg. The couple of forces F and f try to topple
b
the block about point S. To cancel the effect of this unbalanced
torque the normal reaction N shifts towards S by a distance x so
that torque of N counter balances torques of F and f. Figure 7.61: Block toppling on rough surface
7 . 3 4 | Rotational Mechanics

Fh
Fh = (mg)x or x =
mg
If F or h or both increase, distance x also increases, but it cannot go beyond the maximum value of xmax = b/2 i.e in
extreme case N passes through edge S. If F is further increased block will topple.
mgb
So, Ftopple =
2h

Here we assumed that the surface is sufficiently rough so that sliding starts only when
b
F = fmax = µmg > Ftopple or µ > (toppling before sliding)
2h
If surface is not sufficiently rough, the body slides before F is increased to Ftopple i.e. the body will slide before
toppling. This is the case when
b
F = fmax = µmg < Ftopple or µ <
2h

Illustration 38: A uniform cube of side ‘a’ and mass m rests on a rough horizontal table. A horizontal force F is
a
applied normal to one of the faces at a point directly below the center of the face, at a height above the base.
4
(i) What is the minimum value of F for which the cube begins to tip about an edge?
(ii) What is the minimum value of µs so that toppling occurs?

(iii) If f1= µmin , find minimum force for toppling.
(iv) Minimum µs so that Fmin can cause toppling.  (JEE ADVANCED)

Sol: For part (i) we consider toppling before sliding. The normal reaction will pass through the edge. In part (ii) it
is not mentioned whether the toppling occurs before sliding or sliding occurs before toppling. So the toppling will
occur for any value of µs , sliding or no sliding. Part (iii) is same as part (i). Part (iv) is the case of toppling before
sliding.
(i) In the limiting case normal reaction will pass through O. The cube will tip about O if torque of F about O
exceeds the torque of mg.
a a
Hence, F   > mg   or F > 2 mg
4 2
Therefore, minimum value of F is 2 mg.
(ii) In this case since it is not acting at COM, toppling can occur even after body started sliding even if there is no
friction by increasing the torque of F about COM. Hence µmin = 0.
(iii) Now body is sliding before toppling. O is not I.A.R., torque equation cannot be applied across it. It can be
applied about COM.
a a
F× =N×  … (i) N
4 2
N = mg  ... (ii)
From (i) and (ii) -> F = 2 mg
F

(iv) F > 2 mg ………………… (i) (From sol. (i)) a/2 a/4


fr
N = mg  … (ii)
O
F = µs N = µs mg  … (iii)
mg
From (i) and (iii) µs = 2
Figure 7.62
P hysi cs | 7.35

Illustration 39: Find minimum value of  so that truck can avoid the dead end, without toppling the block kept
on it. (JEE ADVANCED)

b
v
h m
Sufficiently

rough surface
power breaks

Figure 7.63

Sol: The block kept on truck will experience pseudo force in forward direction and friction
b
force due to the floor of the truck in backward direction. We assume the case of toppling
before sliding. In extreme case the normal reaction N = mg will pass through the edge.
h ma
h b b
ma ≤ mg ⇒ a ≤ g
2 2 h
f
N
b
Final velocity of truck is zero. So that 0 = v 2 − 2( g) mg
h
Figure 7.64
h v2
=
2b g

PROBLEM-SOLVING TACTICS

•• Most of the problems involving incline and a rigid body can be solved by using conservation of energy during
pure rolling. In case of non-conservative forces, work done by them also has to be taken into consideration in
the equation. Care has to be taken in writing down the Kinetic energy. Rotational Kinetic Energy term has to be
taken into consideration. And while writing the rotational energy, the axis about which the moment of inertia
is taken should pass through the COM.
•• The motion of a body in pure rolling can be viewed as pure rotation about the bottommost point of the body
or the point of contact with the ground. Hence an axis passing through the point of contact and tangential
to the point would be the Instantaneous axis of rotation. So problems on pure rolling can be solved easily by
using the concept of instantaneous axis of rotation.
•• Problems on toppling can be easily solved by writing the moments on the body and visualizing them as forces
acting on the body. If the net moment is tending to stabilize the body, then the body doesn’t topple. For any
condition else it may get toppled.
•• Problems which include the concept of sliding and rolling can be solved easily by using the concept of
conservation of angular momentum. But care has to be taken in selecting the proper axis so that net moment
about that axis vanishes.
7 . 3 6 | Rotational Mechanics

FORMULAE SHEET

S. Term Description Linear Motion Rotational motion &


No relation
1 Displacement Displacement (linear or angular) is the physical s
θ
change in the position of the body when a body
moves linearly or angular in position. (s = rθ)
(a) The linear displacement ∆s is difference
between final and initial position measured in
linear direction.

S.I. unit: meter m

(b) The angular displacement of the body while


rotating about a fixed axis is the displacement
∆θ it swept out with respect to its initial
position in sense of rotation. It can be positive
(anti clockwise) or negative (clockwise)

S.I. unit: radians rad,


2 Velocity Velocity of any moving object is the time
rate of change of position. The velocity is the ds dθ
vector quantity. Linear velocity is in the plane v= ω= (v = rω)
dt dt
of motion. Angular velocity can be positive or
negative & its direction is perpendicular to the
plane of rotation

Linear velocity is categorized as

- Average velocity= ∆s / ∆t

- Instantaneous velocity= ds/dt.

S.I. unit: m/s

Angular velocity is categorized as

- Average angular velocity ∆θ / ∆t

- Instantaneous angular velocity ω = dθ / dt

S.I. unit: rad/s


3 Acceleration Acceleration is the time rate change of
velocity of a body. It’s a vector quantity. Linear dv dω
acceleration can be positive or negative and a= α= (a = rα )
dt dt
related to direction of motion.

Linear acceleration is categorized as

- Average acceleration= ∆v / ∆t

- Instantaneous acceleration = dv/dt.


P hysi cs | 7.37

S. Term Description Linear Motion Rotational motion &


No relation
S.I. unit: m/s–2

Angular acceleration is categorized as

- Average angular acceleration ∆ω / ∆t

- Instantaneous angular acceleration


α = dω / dt
S.I. unit: rad/s–2
4 Mass Mass is the basic entity of any body by virtue of M I ( I = ∑mr2)
which the body gains weight.

In linear kinematics the mass of whole body is


constant. S.I. unit: kilogram kg

In angular kinematics mass of body is


distributed among various tiny rigid points
so mass is measured about inertia of rotating
body- moment of inertia I
5 Momentum Momentum of body is product of mass and its p = mv

L =I
velocity of motion. It’s a vector quantity.
  
Linear momentum= mv
L= r × p

S.I. unit: kg m/s

Angular momentum of body is a vector in


direction perpendicular to plane of rotation

given by L

S.I. unit: kg m2/s


6 Impulse Impulse is the product of force and time period

And it is categorized as ∫ F dt ∫ τdt


-Linear impulse

-Angular impulse
  
7 Force From the newton second law of motion, force is F = ma τ= r×F = I×α
time rate of change of momentum. It’s a vector 
(Newton’s If = 0 the body is in
quantity. dL
second law of equilibrium with its =
dp
motion) Linear force F= = ma surrounding dt
dt
If = 0 the body is in
S.I. unit: Newton N
equilibrium with its
 
Angular force τ = I × α surrounding

Laws of conservation of momentum

- Linear momentum is said to be conserved if


dp
= 0, than P remains constant
dt
- Angular momentum is said to be conserved if

dL
= 0 than L remains constant
dt
7 . 3 8 | Rotational Mechanics

S. Term Description Linear Motion Rotational motion &


No relation
8 Work Work is the product of displacement of body
under action of external applied force. W= ∫ F ds W= ∫ τdθ
9 Power Power is the time rate change of work done P =F P=τω

10 Kinetic energy The phenomenon associated with the moving


1 1 2
bodies K.E. tran = mv 2 K.E. rot = Iω
2 2

11 Kinematics of Kinematical equation are the interrelation of v = u + at ω = ω0 + αt


Motion displacement, velocity, acceleration and time 1
s = ut + at2
and are categorized as follows: 2 1 2
θ = ω0 t + αt
v 2 = u2 + 2as 2
-Linear kinematical equation

-Angular kinematical equation ω2 = ω20 + 2αθ

12 Parallel Axis
I xx = Icc +Md2 where Icc is the moment of
Theorem
inertia about the center of mass

13 Perpendicular I xx + I yy = I zz It is valid for plane laminas


Axis Theorem
only.

14 Work energy Work energy principle is used to determine the


1 1 1 1
principle change in the kinetic energy of moving body W= mv 2 − mu2 W= Iω2 − Iω20
2 2 2 2

Solved Examples

JEE Main/Boards
X v
R
The first five Examples discussed below show us the cm
strategy to tackle down any problem in the rigid body L
motion. Hence follow them up properly! They may be
lengthy but are very learner friendly!!
Sol: The frictional forces acting on the feet of man will
Example 1: A person of mass M is standing on a railroad provide the necessary centripetal acceleration to move
car, which is rounding an unbanked turn of radius at in a circular path. Apply the Newton’s second law of
speed v. His center of mass is at a height of L above motion at the center of mass of the man to get the
the car midway between his feet, which are separated equation of motion along the circular path. In the
by a distance of d. The man is facing the direction of vertical plane the man is in rotational and translational
motion. What is the magnitude of the normal force on equilibrium under the action of its weight acting
each foot? vertically downwards and the normal reactions at its
feet acting vertically upwards. Get one equation each
P hysi cs | 7.39

for rotational and translational equilibrium in vertical d


τcm2 = − N2 + Lf2 (clockwise)
plane. 2

X v NL
R
N1 N2 L Fcm
f1 f2
f

d
We draw the free body diagram of the man, as shown 2
in figure.
 
Static friction f1 and a normal reaction N1 is acting on Both these torques about the center of mass must add

the inner foot. Static friction f 2 and normal reaction up to zero.

N2 is acting on the outer foot. We do not assume the Therefore
limiting value of frictional forces. The weight of the man
d d
acts at its center of mass. ( N1 + Lf1 )+ ( − N2 + Lf2 )=0
2 2
As the man is moving in a circular path with speed, by
d
Newton’s Second Law the forces of friction should act (N − N2 ) + L(f1 + f2 )= 0  ….(iii)
2 1
towards the center of the circular path.
Putting (i) in (iii) we get,
cm
d v2
(N1 − N2 ) + Lm = 0
N1 2 R
Fcm 2Lmv 2
or
L
N2 − N1 =  ….(iv)
Rd
F1 Solving (ii) and (iv) we get
d
2 1 2Lmv 2 
N1=  mg −   ….(v)
 2 R d 
v2
f1 + f2 =m ….(i)
R  1  2Lmv 2 

= N2  + mg   …. (vi)
For vertical equilibrium we should have 2  R d 

N1 + N2 − mg =
0
or mg 
N1 + N2 = ….(ii) Example 2: A Yo-Yo of mass m has an axle of radius b
and a spool of radius R. Moment of inertia about the
For rotational equilibrium of the man about its center center can be taken to be Icm = (1/2) MR² and the
 total thickness of the string can be neglected. The Yo-Yo is
of mass we have τcm 0
=
released from rest. You will need to assume that the
The gravitational force does not contribute to the center of mass of the Yo-Yo descends vertically, and
torque about center of mass because it is acting at the that the string is vertical as it unwinds.
center of mass itself. We draw a torque diagram in the
figure showing the line of action of the forces at the
inner foot.
The torque on the inner foot about COM is given by
d
τcm1 = N + Lf1 (clockwise) b
2 1
We draw a similar torque diagram for the forces at the
outer foot. R

The torque on the outer foot about COM is given by


7 . 4 0 | Rotational Mechanics

(a) What is the tension in the cord as the Yo-Yo or


descends? Mg Mg Mg
T= = = ….(v)
(b) What is the magnitude of the angular acceleration  Mb2   Mb2   2b2 
as the Yo-Yo descends and the magnitude of the linear  1+   1+   1+ 2 
 Icm   (1 / 2)MR 
2  R 
acceleration?    

(c) Find the angular velocity of the Yo-Yo when it (b) Substitute Eq. (v) into Eq. (i) to determine the angular
reaches the bottom of the string when a length L of the acceleration
string has unwound. bT 2bg
α= = 2  ….(vi)
Icm (R +2b2 )
Sol: Apply the Newton’s second law of motion at the
center of mass of Yo-Yo to get the equation of motion From (iii) and (vi) we get
along the vertical direction. Get the relation between
net torque, of all the external forces acting on Yo- 2b2g g
acm = bα = =  ….(vii)
2 2
Yo, and its moment of inertia, both these quantities (R + 2b ) 1 + (R / 2b2 )
2

calculated about the axis passing through the center of For a typical Yo-Yo, the acceleration is much less than
mass of Yo-Yo. As the Yo-Yo descends, the loss in the that of an object in free fall.
gravitational potential energy is equal to the gain in the
(c) Use conservation of energy to determine the angular
translational and rotational kinetic energy.
velocity of the Yo-Yo when it reaches the bottom of
the string (Tension force does not perform any work
>

k
because point of contact between string and Yo-Yo is
>

X i always at rest).
Loss in gravitational potential energy = Gain in kinetic
>

j
T energy MgL
b
1 1 1 1
= MgL
= Mv 2cm + = Icmω2 2
M( v cm + R 2 ω2 ) ….(viii)
R 2 2 2 2
Linear velocity of COM and angular velocity are related
mg by the constraint condition,
(a) The torque of tension in the cord about the center of v cm - bω= 0 ⇒ v cm= bω ….(ix)
mass of the Yo-Yo is in the clockwise direction. So as the
Yo-Yo descends with linear acceleration acm, it rotates in Solving (viii) and (ix) for ω, we get ω =
4gL
the clockwise direction with angular acceleration α. (2b2 + R 2 )
τcm = b T = Icmα (clockwise)  ….(i)
Example 3: A uniform cylinder of radius R and mass
Applying Newton’s Second Law for the motion of COM
M with moment of inertia about the center of mass
in the vertical direction,
Icm = (1/2) MR² starts rolling due to the mass of the
Mg - T = Macm  ….(ii) cylinder, and has dropped a vertical distance h when
As the string is stationary, and the Yo-Yo does not slip it reaches the bottom of the incline. Let g denote the
on the string, the angular acceleration and the linear gravitational constant. The coefficient of static friction
acceleration of COM are related by the constraint between the cylinder and the surface is µ . The cylinder
condition, rolls without slipping down the incline. The goal of this
problem is to find the magnitude of the velocity of
acm - bα = 0 ⇒ acm= bα  ….(iii) the center of mass of the cylinder when it reaches the
From (ii) and (iii) we get, bottom of the incline.

Mg – T = M bα  ….(iv)
Eliminating α from (i) and (iv) we get
Mb2 T
Mg – T =
Icm
P hysi cs | 7.41


R
cm

>
cm Fs X k
h
fs
mg
N

 

Sol: This problem can be solved either by applying law


Then the only force exerting a torque about the center of
of conservation of mechanical energy, or by applying
mass is the friction force, and so we have fsR = Icmα z
Newton’s laws of motion.
…(iii)
We shall solve this problem in three different ways.
Use Icm = (1/2) M R² and the kinematic constraint
1. Applying the torque equation about the center of for the no-slipping condition α z = ax /R in Eq. (xxxiv)
mass and the force equation for the center of mass to solve for the magnitude of the static friction force
motion. yielding
2. Applying the energy equation. fx =(1/2)Max …(iv)
3. Using torque about a fixed point that lies along the Substituting Eq. (iv) into Eq. (v)
line of contact between the cylinder and the surface.
Mg sin θ = (1/2) M ax =M ax  …(v)
Applying the torque equation about the center of mass
and the force equation for the center of mass motion. Which we can solve for the acceleration

We will find the acceleration and hence the speed at 2


ax = sin β  …(vi)
the bottom of the incline using kinematics. A figure 3
showing the force is shown below.
The displacement of the cylinder is x = h/ sin β in
f
the time it takes the bottom, t . The x – component
e f
of the velocity v x at the bottom is v x.f = ax.f . The
displacement in the time interval t satisfies x = (1/2)
mg f f
 ax t2f . After eliminating tf , we have x f = v x .f 2 /2 ax ,
fs
so the magnitude of the velocity when the cylinder
N j i reaches the bottom of the inclined plane is

v x .f = 1ax xf
 
=          2((2 / 3)g sin=
β)(h / sinβ) (4 / 3)gh
Choose x = 0 as the point where the cylinder just …(viii)
starts to roll. With the unit vectors shown in the figure
above, Newton’s second Law, applied in the x – and y –
Note that if we substitute Eq. (vi) into Eq. (iv) the
directions in turn, yields
magnitude of the friction force is
Mg sin β − fs = Max  …(i)
fs = (1/3) Mg sinb  …(ix)
−N + Mg cos β =0  …(ii)
In order for the cylinder to roll without slipping.
Choose the center of the cylinder to compute the
torque about (see figure below). fs ≤ µsMg cos β  …(x)

So combining Eq. (ix) and Eq. (x) we have the condition


that

(1/3) Mg sin β ≤ µs Mg cosβ  …(xi)


7 . 4 2 | Rotational Mechanics

Thus in order to roll without slipping the coefficient of 


static friction must satisfy

>
1 P1 X k
µs ≥ tanβ  …(xii) mg
3
P
Applying the energy equation mg
We shall use the fact that the energy of the cylinder- f i
N
earth system is constant since the static frictional j
d2
force does no work. Choose a zero reference point
for potential energy at the center of mass when the 
cylinder reaches the bottom of the incline plane. 
The gravitational force M g = Mg sin β ˆj acts at the
Then the initial potential energy is Ut = Mgh …(xiii) center of mass. The vector from the point P to the
Mg – N = 0 center of mass is given by rp.mg = dp ˆi − Rjˆ , so the
torque due to the gravitational force about the point
For the given moment of inertia, the final kinetic energy
P is given by
is
  
1 1 τp.mg= rp.mg × Mg= (dP ˆi − Rj)
ˆ × (Mg sin β ˆi + Mg cos βˆj)
Kf = Mv x.f 2 + Icmωz.f 2
2 2
1 1            = (dpMgcos β + RMgsin β)kˆ  …(xvii)
= Mv x.f 2 + (1 / 2)MR 2 (v x.f /R)2  … (xiv)
2 2
The normal force acts at the point of contact between
3
= Mv x.f 2 the cylinder and the surface and is given by
4 
N = −Njˆ . The vector from the point P to the point
 of
Setting the final kinetic energy equal to the initial
contact between the cylinder and the surface is τP.N =
gravitational potential energy leads to
dpˆi . So the torque due to the normal force about the
point P is given by
  
vxy=0 τp.N = τp.N × N =(dpˆi) × ( −Nj)
ˆ =−d kˆ 
p …(xviii)
h
Substituting Eq. (xxxiii) for the normal force into Eq.
v=0 (xviii) yields
x
vxy 
 τp.N = −dPMg cosβ kˆ  …(xix)

3 Therefore the sum of the torques about the point P is


Mgh= Mv x.f 2  …(xv)
4   
τp = τp.Mg + τp.N = ( Mg cos β + RMg sinβ) k̂ – dP
The magnitude of the velocity of the center of mass of Mg cosβ k̂ = Rmg sinβ k̂  …(xx)
the cylinder when it reaches the bottom of the incline
is v x.f = (4/3)gh  …(xvi) The angular momentum about the point P is given by
   
LP = Lcm + rp.cm × MVcm
In agreement with Eq. (viii)
= Icmωzkˆ +(dpˆi − Rj)
ˆ × (Mv )iˆ
x
Using torque about a fixed point that lies along the line
of contact between the cylinder and the surface = (Icmωz + RMv x )kˆ  …(xxi)
Choose a fixed point that lies along the line of contact The time derivative of the angular momentum about
between the cylinder and the surface. Then the torque the point P is then
diagram, is shown below. 
dLP
= Icmα z +RMaxkˆ  …(xxii)
dt
Therefore the torque equation

 dLP
τ=  …(xxiii)
dt
P hysi cs | 7.43

Becomes RMg sin βk̂ = (Icmα z +RMax )kˆ  …(xxiv) in direction and have same line of action and will also
pass through the point of contact S. (Point of contact is
Using the fact that Icm = (1 / 2 ) MR² and α x = vertically below the COM of the ball).
ax / R, we can conclude that RM ax = ( 3 / 2 ) MR ax  Thus we choose the initial point of contact S as the
…(xxv) origin and the net torque of all the forces about the
origin S comes out to be zero at all times. So we can
We can now solve Eq. (xxv) for the x – component of conserve the angular momentum of the ball about the
the acceleration initial point of contact (origin S).

ax  = ( 2 / 3) g sin β  …(xxvi) The initial angular momentum about the origin S is


only due to the translation of the center of mass.
In agreement with Eq. (vi). Li= mv0R
The final angular momentum about the origin S has
Example 4: A bowling ball of mass m and radius R is
both translational and rotational contribution.
initially thrown down an alley with an initial speed v 0
and it slides without rolling but due to friction it begins Lf= m vcm,f R + Icmω
to roll. The moment of inertia of the ball about its Finally the ball rolls without slipping, so we have vcm,f=
center of mass is Icm = (2/5) mR². Using conservation Rω
of angular momentum about a point (you need to find
that point), find the speed v f of the bowling ball when Now Icm = (2/5)mR²
it just start to roll without slipping? Therefore the final angular momentum about the origin
S is

VC WC VF


rs,cm,f mvcm,f

S W2,f
Sol: The angular momentum of any rigid body about
a fixed point in ground reference frame is the sum of L f =(mR + (2/5)mR)v cm,f = (7/5)mRv cm,f
the angular momentum in the C-frame and the angular Now equating Li = Lf we get
momentum corresponding to the translation of the
center of mass relative to the fixed point in ground mR v0 = (7/5) m R v cm,f
frame. or v cm,f = (5/7) v 0

 Example 5: A long narrow uniform stick of length 


and mass m lies motionless on ice (assume the ice
>

X k
cm provides a frictionless surface). The center of mass of
the stick is the same as the geometric center (at the
mg i midpoint of the stick). The moment of inertia of the
stick about its center of mass is Icm . A puck (with
fk N j putty on one side) has same mass m as the stick. The
puck slides without spinning on the ice with a speed
of v 0 towards the stick, hits one end of the stick, and
At t = 0, when the ball is released v cm,i = v 0 towards attaches to it. You may assume that the radius of the
right and ωi =0 , so the ball slips towards right on the puck is much less than the length of the stick so that
surface and hence the frictional force on the ball, will moment of inertia of the puck about its center of mass
be towards left. is negligible compared to Icm .
The frictional force will pass through the point of
contact S with the surface.
The weight of the ball as well as the normal reaction
from the surface are equal in magnitude and opposite
7 . 4 4 | Rotational Mechanics

m (c) As the collision is perfectly inelastic the mechanical


energy of the system is not conserved.
(d) Choose the center of mass of the stick-puck
combination, as found in part (a) as the point about
which we find the angular momentum before and after
OVC the collision. This choice is advantageous as there will
m be no angular momentum due to the translation of
before after the center of mass just after the collision about the
(a) How far from the midpoint of the stick is the center of center of mass itself. Before the collision, the angular
mass of the stick–puck combination after the collision? momentum was entirely due to the motion of the puck,

(b) What is the linear velocity of the stick plus puck after L0 = ( / 4)(mv 0 )
the collision? After the collision, the angular momentum is L= Icmωf
f
(c) Is mechanical energy conserved during the collision? where Icm is the moment of inertia about the center of
Explain your reasoning. mass of the stick-puck combination.
(d) What is the angular velocity of the stick plus puck This moment of inertia of the stick about the new center
after the collision? of mass is found from the parallel axis theorem, and the
(e) How far does the stick’s center of mass move during moment of inertia of the puck is m ( / 4)2 , and so
one rotation of the stick?
Icm = m(2 / 12) + m( / 4)2 + m( / 4)2 = 5m2 / 24

Sol: Apply the law of conservation of linear momentum 2


and law of conservation of angular momentum before Equating L0 = Lf we get ( / 4)(mv
= 0 ) 5m ωf / 24
and after collision. The angular momentum is to be
6v 0
calculated about the center of mass of “stick-puck This gives ωf =
system”. 5
(e) Time taken by stick-puck system for one rotation is
(a) From the midpoint of the stick the center of mass
T = 2π/ωf
of the stick–puck combination after the collision is at a
distance dcm. Distance travelled by COM during this time is

mstick dstick + mpuck dpuck m × 0 + m( / 2)  2π 2π


(b) dcm
= = = xcm = v cmT = v cm = (v / 2)
mstick + mpuck m+m 4 ωf 6v 0 / 5 0
5π
This gives xcm =
There are no external forces acting on this system 6
comprising “stick and puck” so the momentum of the
system before and after the collision is conserved. Example 6: Two small kids of masses 10 kg and 15 kg
are trying to balance a seesaw of total length 5.0 m with
>

m j the fulcrum at the center. If one of the kids is sitting at


>

i
ends, where should the other sit?
>

cm Wf Sol: For rotational equilibrium, the net torque about


the fulcrum of all the forces acting on the boys and the
Vcm
OV0
m seesaw should be zero.
before after
It is clear that the 10 kg kid should sit at the end and
After the collision, suppose the center of mass of the the 15 kg kid should sit closer to the center. Suppose
system is moving with speed v f this distance from the center is X. As the kids are in
equilibrium, the normal force between a kid and the
Equating initial and final linear momentum we get, see-saw equals the weight of that kid. Considering the
v0 rotational equilibrium of the seesaw, the torques of the
mv= (2m)v f ⇒ =
vf
0
2 forces acting on it should add to zero. The forces are.
The direction of the velocity is the same as the initial
direction of the puck’s velocity.
P hysi cs | 7.45

2 2
T= N = (392N) × = 450 N.
3 3

Example 8: A solid cylinder of mass m and radius r


starts rolling down an inclined plane of inclination θ .
Friction is enough to prevent slipping. Find the speed
of its center of mass when, its center of mass has fallen
a height h.
(a) (15kg) g downward by the 15kg kid,
(b) (10 kg) g downward by the 10kg kid,
(c) Weight of the seesaw and
(d) The normal force by the fulcrum. 
Taking torques about by the fulcrum
Sol: Loss in the gravitational potential energy of
(15kg) g x = (10 kg) g (2.5m) or x = 1.7 m. the cylinder is equal to the gain in rotational plus
translational kinetic energy.
Example 7: The ladder shown in figure has negligible Consider the two shown positions of the cylinder. As it
mass and rests on a frictionless floor. The crossbar does not slip, total mechanical energy will be conserved.
connects the two legs of the ladder at the middle. The
angle between the two legs is 60°.
The fat person sitting on the ladder has a mass of 80 kg.
Find the contact force exerted by the floor on each leg
and the tension in the crossbar. h

Sol: The forces of normal reaction at the feet of the 


ladders balance the weight of the person. For rotational
equilibrium of ladder the toque due to normal reaction
at the foot is balanced by the torque due to tension in Energy at position 1 is E1 = mgh
the crossbar. Both the torques are calculated about the 1 1
Energy at position 2 is E2 = mv 2 + I ω2
upper end of the ladder. 2 cm 2 cm
The forces acting on the different parts are shown in v mr 2 3
 cm = ω and Ic.m = ⇒ E2 =mv 2c.m
figure. Consider the vertical equilibrium of “the ladder r 2 4
plus the person” system. The forces acting on this 4
system are its weight (80kg)g and the contact force N + From COE, E1 = E2 ; v c.m = gh
3
N = 2N due to the floor. Thus
2N = (80kg) g or N = (40 kg) (9.8m/s²) = 392 N Example 9: A uniform disc of radius R and mass M is
rotated to an angular speed ω0 in its own plane about
its center and then placed on a rough horizontal surface
such that plane of the disc is parallel to the horizontal
w surface. If co-efficient of friction between the disc and
1m the surface is µ then how long will it take for the disc
0 to come to stop.
60
N N
1m T T
300
dx

Next consider the equilibrium of the left leg of the C x


ladder. Taking torque of the forces acting on it about
the upper end.
N (2 m) tan 30°= T (1 m) or R
7 . 4 6 | Rotational Mechanics

Sol: The disc can be thought of made-up of elementary The slipping will stop when the condition of pure rolling
rings of infinitesimal thickness. The torque about the is satisfied.
center of disk due to friction force on each ring will be v
Velocity about the center = 0 . Thus v 0 > ω0r . The
different from the other rings in the disc as the radii 2r
of rings are different, varying from 0 to R. So use the sphere slips forward and thus the friction by the plane
method of integration to find the torque on the entire on the sphere will act backward. As the friction is kinetic
disc. its value of N is given by µN = µMg and sphere will be
Consider a differential circular strip of the disc of radius decelerated by acm = f/M. Hence.
x and thickness dx. Mass of this strip is dm = 2ρ πx dx , This friction will also have a torque T = fr about the
M center. This torque is clockwise and in the direction of
where ρ = . Frictional force on this strip is along
πR 2 ω0 . Hence the angular acceleration about the center
the tangent and is equal to dF = 2µρπgx dx will be
r 5f
Torque on the strip due to frictional force is equal to α=f =
(2 / 5)Mr 2 2Mr
dt = mρg2px2dx
The disc is supposed to be the combination of Number and the clockwise angular velocity at time t will be
of such strips hence torque on the disc is given by v 0 5f
5f
ω(t) = ω0 + t= + t
R 2Mr 2r 2Mr
2 R3
τ= ∫ dτ = µρ g2πx ∫ x dx = µρg2π 3
Pure rolling starts when
0
⇒ τ = µMg(2/3)R Eliminating t from (i) and (ii)
5 5 v
2µMgR 4 µg v(t) + v(t) = v 0 + 0 Or
⇒ α= = 2 2 2
 MR 2  3 R
3 Thus the sphere rolls with translational velocity 6 v 0 /7
 2  in the forward direction.
 
The α is opposite to the ù

 ω(t) − ω0 + αt ⇒ 0 = ω0
4µg
t
JEE Advanced/Boards
3R
3ω0R Example 1: A carpet of mass M made of inextensible
⇒t= . material is rolled along its length in the form of a
4µg
cylinder of radius R and is kept on a rough floor. The
carpet starts unrolling without sliding on the floor
Example 10: A sphere of mass M and radius r shown
when a negligibly small push is given to it. Calculate
in figure slips on a rough horizontal plane. At some
the horizontal velocity of the axis of the cylindrical part
instant it has translational velocity v 0 and rotational
of the carpet when its radius decreases to (R/2).
v
velocity about the center is 0 . Find the translational
2r 50N
velocity after the sphere starts pure rolling. P

30o
=V0/2r
r V0
Sol: As the carpet unrolls, the radius and mass of
A cylindrical part decreases and center of mass descends.
F Thus loss in the gravitational potential energy is equal
to gain in rotational plus translational kinetic energy.
Sol: Due to forward slipping the friction will act
If ρ is the density of material of the carpet, initial mass
backwards. So the sphere will decelerate. The torque
of the carpet (cylinder) M will be πR 2Lρ . When its radius
due to friction will be in the direction of initial angular
becomes half, the mass of cylindrical part will be
velocity. So the angular velocity will increase.
π(R / 2)2 Lρ = M/4
MF =
P hysi cs | 7.47

So initial PE of the carpet is MgR while final energy. At the edge the COM moves in circular arc during
the time interval when the vertical radius through the
(M/4) g(R/2) = MgR/8
point of contact turns by angle α to become normal to
So loss in potential energy when due to unrolling radius the inclined plane. During this interval normal reaction
changes from R to R/2 from edge should always be greater than zero.
MgR (1–(1/8)) = (7/8)MgR  … (i)
1 1
Initial energy E1= mv 20 + Ic.mω2+mgR
2 2
v0
For rolling =ω
R


R R/2
(A) (B) 

This loss in potential energy is equal to increase in 1 1 1 v


⇒ E1= mv 20 + . mR 2 0 + mgR
rotation KE which is 2 2 2 R2
1 1 3
K = K T +KR = Mv 2 + Iω2 = mv 02 +mgR
2 2 4
If v is the velocity when half the carpet has unrolled, 1 1
E2= mv 2 + Ic.mω`2 + mgR cosα
then as 2 2
3
R M 1 M  R 
2 = mv 2 + mgR cosα
v = ω, M → and I =     4
2 4 2  4  2
From COE
2
1  M  2 1  MR 2   2v 
K =  v +    3 3
24 2  32   R  mv 2 + mgR cosα = mv 20 + mgR
4 4
1 1 3 4
i. e., K = Mv 2+ Mv 2= Mv 2  … (i) mv 2 = mv 20 +
3
mgR(1 − cos α )  …. (i)
8 16 16
F.B.D. of the cylinder when it is at the edge.
So from equation (i) and (ii) Center of mass of the cylinder describes circular motion
 3  2 7 about P.
  Mv =   MvR
 16  8 Hence mg cos α - N = mv 2 /R
i.e., v = (14gR) / 3 ⇒ N = mgcos α − mv 2 /R
mv 20 4 4
= mg cos α - − mg + mgcos α
Example 2: A uniform solid cylinder of radius R = 15 R 3 3
cm rolls over a horizontal plane passing into an inclined For no jumping, N ≥ 0
plane forming an angle α = 30° with the horizontal. 7 4 mv 0 2

Find the maximum value of the velocity v 0 which still ⇒ mgcos α − mg − ≥0


3 3 R
permits the cylinder to roll onto the inclined plane
section without a jump. The sliding is assumed to be 7gR 4
absent.
⇒ v0 ≤ cos α − g
3 3

V0 Example 3: Two thin circular disc of the mass 2 kg each


R and radius 10 cm each are joined by a rigid massless
 rod of length 20 cm. The axis of the rod is perpendicular
to the plane of the disc through their centers as
Sol: As the cylinder rolls into the inclined plane section, shown in the figure. The object is kept at the center
its center of mass descends, thus loss in gravitational as shown in the figure. The object is kept on a truck in
potential energy will be equal to increase in kinetic such a way that the axis of the object is horizontal and
7 . 4 8 | Rotational Mechanics

perpendicular to the direction of motion of the truck. Here a’ = acceleration of the center of mass of the
Its friction with the floor of the truck is large enough object.
to prevent slipping. If the truck has an acceleration of
For rotational motion
9 m/s² calculate.
fR = Ia
mR 2 a'
= 2. . for no slipping α = a/R
2 R
f
⇒ a' =  ….. (ii)
m
From (i) and (ii) we get
(a) The force of friction on each disc.
2
(b) The magnitude and direction of the frictional torque F= ma î ⇒ Force of friction on each disc is
3
acting on each disc about the center of mass ‘O’ of the
f ma ˆ
object. Take x-axis along the direction of the motion of = i = 6iˆ N
the truck, and z-axis along vertically upwards direction. 2 3
1 2
Express the torque in the vector form in terms of unit 2
vectors î , ˆj and k̂ in the x, y and z directions. z

(c) Find the minimum value of the co- efficient of r1 O


friction µ between the object and the floor of the truck r2
y
which makes rolling of the object possible.
 ma ˆ
f1 = i
Sol: This problem is best solved in the reference frame 3
of truck. Each disc will experience pseudo force as well 
as frictional force. Get two equations, one by applying r1 =  ˆj − R kˆ
Newton’s second law at the center of mass of the object,
  
and the other relating the torques of forces about the τ f = r1 × f 1 = ˆ × ma ˆi
−(ˆj + Rk)
center of mass and the moment of inertia about axis 1 3
passing through the center of mass. maR ˆ ma ˆ
= − j+ k
F.B.D. of the object with respect to truck. 3 3
= 6 × 0.1 î + 6 × 0.1 k̂
N
= –0.6 ˆj + 0.6 k̂

z τ(f ) = –0.6 ˆj – 0.6 k̂
2ma 2
(c) Maximum value of frictional force is 2 µ mg
f x 2 a
mg Side view ⇒ ma ≤ 2µmg ⇒ µ> .
3 3g
In the reference frame of truck it experiences a pseudo
Example 4: A uniform disc of mass m and radius r
force F = -2ma î is projected horizontal with velocity v 0 on a rough
where a = acceleration of the truck. horizontal floor so that it starts of with a purely sliding
motion at t = 0. At t = t0 second it acquires a purely
rolling motion.
V0 V (a) Calculate the velocity of the center of mass of the
disc at t = t0 .

Pseudo force does not provide torque about the center


of the disc. Because of this force object has tendency
to slide along – Ve x –axis, hence frictional force will act
along + Ve x – axis. For translational motion. P
2ma – f = ma’  ……… (i) (b) Assuming coefficient of friction to be µ calculate t0 .
P hysi cs | 7.49

(c) The work done by the frictional force as a function 2(v 0 − v)


of time. ⇒ω=  … (iii)
r
(d) Total work done by the friction over a time t much (Using (i)) Vc.m = ωr
longer then t0 .
⇒ v = 2 ( v 0 – v) (using (iii))
Sol: This problem can be solved either by applying
Newton’s laws of motion or by law of conservation of 2
⇒ v= v
angular momentum about the point of contact of the 3 0
disc with the floor. The force of friction will act opposite
to the direction of motion, and the work done by friction Alternative method: Since frictional force passes
will be equal to loss in kinetic energy. The friction will through the point of contact, hence about this point no
stop doing any work once pure rolling starts. external torque is acting.
F.B.D. of the disc. Therefore angular momentum of the disc about point
of contact does not change.
N
Initial angular momentum about p is given by

L=1 0 + mv 0 r (Using LP = L c.m + r × Pc.m )
When it starts pure rolling its angular momentum
about P is given by
L2 = Lc.m + ω + mvr
f
For rolling v = ωr
mg
mr 2 v 3
⇒ L2 = + mvr = mvr
When the disc is projected it starts sliding and hence 2 r 2
there is a relative motion between the points of From COAM
contact. Therefore frictional force acts on the disc in
2
the direction opposite to the motion. L1 = L2 ⇒ v = v 0
3
(a) Now for translational motion
(b) Putting the value of v in equation (i)
f v
ac.m = We get t0 = 0
m 3µg
f = µN (as it slides) = µmg
(c) Work done by the frictional force is equal to change
⇒ ac.m =
−µg , negative sign indicates that in K.E.
ac.m is opposite to v c.m ⇒ Wfriction
2
v 0 − µgt0
⇒ v c.m(t) = 1 1  mr 2   2µgt  1
= m(v 0 − µgt)2 +   2
 − mv 0
2 2  2   r  2
(v 0 − v)
⇒ t0 =  …(i) 3 
µg = m  µ2g2 t2 − v 0 µgt  , For t ≤ t0
 2 
where v c.m(t ) = v (d) For time t ≥ t0 work done by the friction is zero.
0
For rotational motion about center For longer time total work done is same as that in part

mr 2  2
v 0 
τf +τmg = Ic.mα ⇒ µmgr = α  3 2 2 2  v0 
2 (c) ⇒ W = m µ g t   − v 0 µgt .
2  3µg  3µg 
2µg  
⇒ α= …(ii) mv 20
r = −
2µg 6
Therefore ω(t ) = 0 + t0
0 r

Using ωt = ω0 + αt
7 . 5 0 | Rotational Mechanics

Example 5: In the shown figure a mass m slides down  ML2 


a frictionless surface from height h and collides with Where I=  + mL2 
 3 
a uniform vertical rod of length L and mass M. After  
collision the mass m sticks to the rod. The rod is free Gain in potential energy of the system
to rotate in a vertical plane about fixed axis through O.
find the maximum angular deflection of the rod from  From conservation of energy
its initial position.
1 2  M
Iω =  m +  gL(1 − cosθ)
o 2  2

m 1  ML2  m2 v 20
⇒  + mL2  ×
2  3 
 M
2
 2
 + m L
h
3 
 M
=  m +  gL(1 − cos θ)
 2
Sol: During the collision between rod and the mass,
the linear momentum is not conserved because of the 1 m v0
2 2
 M
reaction force acting on the rod due to the hinge at =  m +  gL(1 − cos θ)
2 M   2
the fixed point O. The torque of reaction force at the  + m
hinge will be zero about the hinge itself, i.e. about point 3 
O. So we can conserve the angular momentum of the 1 m v0 2 2

“rod and mass system” before and after collision. As the cos θ
cos è= .
2 M  M 
rod rotates, the gain in gravitational potential energy is 3 + m 2 + m  gL
  
equal to the loss in the kinetic energy.
Just before collision, velocity of the mass m is along the Example 6: A billiard ball, initially at rest, is given a
horizontal and is equal to v 0 = 2gh . In the process of sharp impulse by a cue. The cue is held horizontally
a distance h above the center line as shown in figure.
collision only angular momentum of the system will be
The ball leaves the cue with a speed v 0 and because
conserved about the point O.
of its forward rotation (backward slipping) eventually
If L1 and L2 are the angular momentum of the system 9
just before and just after the collision then L1 = mv 0L acquires a final speed v 0 show that.
7
 ML2 
And L2 = Iω =  + mL2  ω F
 3 
  h
From Conservation of Angular Momentum

o
4
(L/2)(1-cos) m h= R where R is the radius of the ball.
5
h
Sol: Initial linear and angular velocity of ball is found
L(1-cos)

by calculating the linear and angular impulse delivered


M  by the cue. The angular momentum of the ball about
 + m  L2 ω = mv 0L
3  the point of contact with ground surface, during its
combined translational and rotational motion remains
mv 0 conserved.
⇒ω=
M 
 + m L
hF
 3  h 9
V= V0
7
Let the rod deflect through an angle θ.
1 2
Initial energy of rod and mass system = Iω fmax
2
P hysi cs | 7.51

Let ω0 be the angular speed of the ball just after it be limiting. Write the equations of Newton’s second
leaves the cue. The maximum friction acts in forward law and torque about center of mass for the sphere
direction till the slipping continues. Let v be linear and the equation of Newton’s second law for the plank.
speed and ω the speed when slipping is ceased. The free body diagram of the sphere and the plank are
v as shown below:
∴ v = Rω or ω =
R Writing equation of motion:
9 µsMg
Given, v = v 0  ….. (i)
7 For sphere: Linear acceleration a1 = =µsg  ….(i)
M
9 v0
∴ ω=  ….. (ii) (µSMg)R
7 R 5 µS g
Angular acceleration: α = =  ….(ii)
Applying Linear impulse = change in linear momentum 2 2 R
MR 2
5
For plank: Linear acceleration
F dt = V0  …. (iii)
Applying Angular impulse = change in angular 
momentum
2 B1
or =
Fh dt mR 2 ω0  …. (iv)
5 s Mg
Angular momentum about bottommost point will
remain conserved.
i.e., Li = L f s Mg a2
or Iω0 + mRv 0 = Iω + mRv F
2
∴ mR 2 ω0 + mRv 0
5
F − µSMg
2  9 v0  9 α2 =
= mR 2   + mRv 0 …... (v) m
5 7 R  7
For no slipping α2 = a1 + Rα
4
Solving Eqs. (iii), (iv) and (v), we get h = R Solving the above four equations, we get
5
 7 
F = µS g  M + m 
Example 7: Determine the maximum horizontal force F  2 
that may be applied to the plank of mass m for which  7 
Thus, maximum value of F can be µS g  M + m 
the solid does not slip as it begins to roll on the plank.  2 
The sphere has a mass M and radius R (see figure). The
coefficient of static and kinetic friction between the Example 8: A uniform disc of radius r0 lies on a
sphere and the plank are µs and µk respectively. smooth horizontal plane. A similar disc spinning with
the angular velocity ω0 is carefully lowered onto the
M first disc. How soon do both discs spin with the same
angular velocity if the friction coefficient between them
is equal to µ ?

m F Sol: The initial angular momentum about its center of


the disc being lowered will be equal to the combined
angular momentum of both the discs about their
centers once they start rotating together. Each disc
Sol: As the plank moves forward, the sphere, due to its can be thought of made-up of elementary rings of
inertia, has a tendency to slip backwards relative to the infinitesimal thickness. The torque about the center of
plank. So the force of friction acts on the sphere in the disk due to friction force on each ring will be different
forward direction. For maximum force F, the friction will from that on the other rings in the disc as the radii
7 . 5 2 | Rotational Mechanics

of rings are different, varying from 0 to r0. So use the Let m be the mass of the sphere.
method of integration to find the torque on the entire
Since, it is a case of backward slipping, force of friction
disc.
is in forward direction. Limiting friction will act in this
From the law of conservation of angular momentum. case.
Iω0 = 2Iω
Here, I = moment of inertia of each disc relative to
common rotation axis
ω0 fmax
∴ ω= = steady state angular velocity
2
The angular velocity of each disc varies due to the
torque τ of the frictional forces. To calculate τ , let us f µmg
Linear acceleration
= a = =µg
take an elementary ring with inner and outer radii r and m m
r + dr. The torque of the friction acting on the given is τ f.r 5 µg
equal to. Angular retardation α= = =
I 2 2 2 r
mr
 mg   2µmg  5
dτ = µr   2πr dr =   r 2 dr
 πr 2   r2  Slipping ceases when v = rω
 0   0 
Or (at) = r (ω0 − αt)
where m is the mass each disc. Integrating this respect
to r between 0 and r0 , we get  5 µg 
Or µgt = r  ω0 − 
2  2 r 
τ= µmgr0
3
7 2 rω0
The angular velocity of the lower disc increases by d ω r 0; t =
µgt =ω
2 7 µg
over the time interval
2
 3r  v= at =µgt =rω0
=  0  dω 7
 4µg  v 2
And ω= = rω
Integrating this equation with respect to ω between 0 r 7 0
ω0 3r0 ω0
and , we find the desired time t = Alternative solution: Net torque on the sphere about
2 8µg
the bottommost point is zero. Therefore, angular
momentum of the sphere will remain conserved about
Example 9: A solid sphere of radius r is gently placed
the bottommost point.
on a rough horizontal ground with an initial angular
speed ω0 and no linear velocity. If the coefficient of Lt = L f
friction is µ , find the time when the slipping stops. In
∴ Iω0 = Iω + mrv
addition, state the linear velocity v and angular velocity
ω at the end of slipping. 2 2 2 2
Or mr= ω0 mr ω + mr(ωr)
5 5
2
∴ ω = ω0
7
2
And v = rω = rω0
7

Example 10: A thin massless thread is wound on reel


Sol: Due to backward slipping the force of friction of mass 3 kg and moment of inertia 0.6 kg-m². The hub
will act forwards and torque due to friction will be radius is R = 10 cm and peripheral radius is 2R = 20 cm.
anti-clockwise. This problem can be solved either by The reel is placed on a rough table and the friction is
Newton’s second law and torque about center of mass enough to prevent slipping. Find the acceleration of the
method or by applying the law of conservation of center of reel and of hanging mass of 1 kg (see figure).
angular momentum about the point of contact of the
sphere with the ground.
P hysi cs | 7.53

Sol: Apply Newton’s second law at the center of mass τ f(2R) − T.R 0.2f − 0.1T
of reel in horizontal direction. Find relation between net α = τ = f(2R) − T.R = 0.2f − 0.1T
α= I = I = 0.6
torque about center of mass of reel and moment of I I 0.6
f T
inertia about axis passing through the center of mass. = f − T…(ii)
…….(ii)
= 3 − 6 …….(ii)
Apply Newton’s second law for hanging mass in vertical 3 6
direction.
T

2R
R
a2

10N

Free body diagram of mass is


Let, a1 = acceleration of center of mass of reel Equation of motion is,
a2 = acceleration of 1 kg block 10 − T = a2 .....(iii) …(iii)

α = angular acceleration of reel (clockwise) For no slipping condition,
T = tension in the string a1 = 2Rα or a1 = 0.2α  …(iv)
and f = force of friction And a2 = a1 − Rα or a2 = a1 − 0.1 α .....(v)
Free body diagram of reel is as shown below; (only Solving the above five equations, we get
horizontal forces are shown.).
a1 = 0.27 m/s²; a2 = 0.135 m/s²
Equations of motion are: T – f = 3a1  … (i)

a1
T

JEE Main/Boards

Exercise 1 Q.4 What are two theorems of moment of inertia?

Q.5 What is moment of inertia of a solid sphere about


Q.1 What are the units and dimensions of moment of
its diameter?
inertia? Is it a vector?

Q.6 What is moment of inertia of a hollow sphere about


Q.2 What is rotational analogue of force?
an axis passing through its center.

Q.3 What is rotational analogue of mass of a body?


7 . 5 4 | Rotational Mechanics

Q.7 What are the factors on which moment of inertia of Q.20 A uniform circular disc and a uniform circular ring
a body depends? each has mass 10kg and diameter 1m. Calculate their
moment of inertia about a transverse axis through their
Q.8 Is radius of gyration of a body a constant quantity? center.

Q.9 There are two spheres of same mass and same Q.21 Calculate moment of inertia of earth about its
radius, one is solid and other is hollow. Which of them diameter, taking it to be a sphere of radius 6400 km
has a larger moment of inertia about its diameter? and mass 6 ×1024 kg.

Q.10 Two circular discs A and B of the same mass and Q.22 Calculate moment of inertia of a uniform circular
same thickness are made of two different metals whose disc of mass 700 g and diameter 20 cm about
densities are dA and dB ( dA > dB ). Their moments of (i) An axis through the center of disc and perpendicular
inertia about the axes passing through their centers of to its plane,.
gravity and perpendicular to their planes are I A and IB .
(ii) A diameter of disc
Which is greater, I A or IB ?
(iii) A tangent in the plane of the disc,
Q.11 The moments of inertia of two rotating bodies (iv) A tangent perpendicular to the plane of the disc.
A and B are I A and I B ( I A > I B ) and their angular
moments are equal. Which one has a greater kinetic
Q.23 Three particles, each of mass m, are situated at
energy?
the vertices of an equilateral triangle ABC of side L. Find
the moment of inertia of the system about the line AX
Q.12 Explain the physical significance of moment of perpendicular to AB in the plane of ABC, in the given figure.
inertia and radius of gyration.

x C
Q.13 Obtain expression of K.E. for rolling motion.

Q.14 State the laws of rotational motion.

Q.15 Establish a relation between torque and moment


of inertia of a rigid body. A B
L

Q.16 State and explain the principle of conservation of


Q.24 Calculate K.E. of rotation of a circular disc of mass
angular momentum. Give at least two examples.
1 kg and radius 0.2 m rotating about an axis passing
through its center and perpendicular to its plane. The
Q.17 Derive an expression for moment of inertia of disc is making 30/π rpm.
a thin circular ring about an axis passing through its
center and perpendicular to the plane of the ring.
Q.25 A circular disc of mass M and radius r is set into
pure rolling on a table. If w be its angular velocity,
Q.18 The moment of inertia of a circular ring about an show that it’s total K.E. is given by (3/4) Mv², where
axis passing through the center and perpendicular to v is its linear velocity. M.I. of circular disc = (1/2)
its plane is 200 g cm². If radius of ring is 5 cm, calculate mass × (radius)².
the mass of the ring.
Q.26 The sun rotates around itself once in 27 days. If
Q.19 Calculate moment of inertia of a circular disc it were to expand to twice its present diameter, what
about a transverse axis through the center of the disc. would be its new period of revolution?
Given, diameter of disc is 40 cm, thickness = 7 cm and
density of material of disc = 9 g cm−3 Q.27 A 40 kg flywheel in the form of a uniform circular
disc 1 meter in radius is making 120 r.p..m. Calculate
its angular momentum about transverse axis passing
through center of fly wheel.
P hysi cs | 7.55

Q.28 A body is seated in a revolving chair revolving at Q.5 A rigid body can be hinged about any point on the
an angular speed of 120 r.p.m. By some arrangement, x-axis. When it is hinged such that the hinge is at x, the
the body decreases the moment of inertia of the system moment of inertia is given by I = 2x² - 12x + 27
from 6 kg m² to 2 kg m². What will be the new angular
The x-coordinate of center of mass is
speed?
(A) x = 2 (B) x = 0 (C) x = 1 (D) x = 3

Q6. A weightless rod is acted upon by upward parallel


Exercise 2 forces of 2N and 4N at ends A and B respectively. The
total length of the rod AB=3m. To keep the rod in
Single Correct Choice Type equilibrium, a force of 6N should act in the following
manner:
Q.1 Thee bodies have equal masses m. Body A is solid (A) Downwards at any point between A and B.
cylinder of radius R, body B is square lamina of side
R, and body C is a solid sphere of radius R. Which (B) Downwards at mid-point of AB
body has the smallest moment of inertia about an axis (C) Downwards at a point C such that AC=1m
passing through their center of mass and perpendicular
to the plane (in case of lamina) (D) Downwards at a point D such that BD= 1m.

(A) A (B) B (C) C (D) A and C both


Q.7 A heavy rod of length L and weight W is suspended
horizontally by two vertical ropes as shown. The first
Q.2 For the same total mass which of the following rope is attached to the left end of rod while the second
will have the largest moment of inertia about an axis rope is attached a distance L/4 from right end. The
passing through its center of mass and perpendicular tension in the second rope is:
to the plane of the body.
(A) A disc of radius a 1 2
(B) A ring of radius a L/4
(C) A square lamina of side 2a
(D) Four rods forming a square of side 2a L

Q.3 A thin uniform rod of mass M and length L has its (A) (1/2) W (B) (1/4) W
moment of inertia I1 about its perpendicular bisector. (C) (1/3) W (D) (2/3) W (E) W
The rod is bend in the form of semicircular arc. Now its
moment of inertia perpendicular to its plane is I2 . The
ratio of I1 : I2 will be Q.8 A right triangular plate ABC of mass m is free to
rotate in the vertical plane about a fixed horizontal
(A) < 1 (B) >1 (C) =1 (D) Can’t be said axis though A. It is supported by a string such that the
side AB is horizontal. The reaction at the support A in
Q.4 Moment of inertia of a thin semicircular disc (mass equilibrium is:
= M & radius = R) about an axis through point O and
perpendicular to plane of disc, is given by:

O l
A B
R
l

1 1 1
(A) MR 2 (B) MR 2 (C) MR 2 (D) MR 2 C
4 2 8
mg 2mg mg
(A) (B) (C) (D) mg
3 3 2
7 . 5 6 | Rotational Mechanics

Q.9 A rod is hinged at its center and rotated by applying


(I + mR 2 )ω2 − mv 2
a constant torque from rest. The power developed by B)
the external torque as a function of time is: I

Pext Pext Iω − mvR


(C)
(A) (B) I
time time
(I + mR 2 )ω − mvR
(D)
Pext Pext I
(C) (D)
time time Q.13 A uniform rod of length l and mass M is rotating
about a fixed vertical axis on a smooth horizontal table.
It elastically strikes a particle placed at a distance l/3
Q.10 Two uniform spheres of mass M have radii R and from its axis and stops. Mass of the particle is
2R. Each sphere is rotating about a fixed axis through its
diameter. The rotational kinetic energies of the spheres
are identical. What is the ratio of the angular moments
L2 2R l
of these sphere? That is, = l/3
LR
(A) 4 (B) 2 2 (C) 2 (D) 2 (E) 1
3M 3M 4M
(A) 3M (B) (C) (D)
Q.11 A spinning ice skater can increase his rate of 4 2 3
rotation by bringing his arms and free leg closer to
his body. How does this procedure affect the skater’s Q.14 a disc of radius R is rolling purely on a flat
momentum and kinetic energy? horizontal surface, with a constant angular velocity. The
angle between the velocity and acceleration vectors of
(A) Angular momentum remains the same while kinetic
point P is
energy increases.
(B) Angular momentum remains the same while kinetic
energy decreases
C
(C) Both angular momentum and kinetic energy remains P
the same.
(D) Both angular momentum and kinetic energy
increase.
(A) Zero (B) 45°
Q.12 A child with mass m is standing at the edge of (C) 135° (D) tan−1 (1 / 2)
a disc with moment of inertia I, radius R, and initial
angular velocity w. See figure given below. The child
Q.15 A particle starts from the point (0m, 8m) and
jumps off the edge of the disc with tangential velocity
moves with uniform velocity of 3m/s. After 5 second,
v with respect to the ground. The new angular velocity
the angular velocity of the particle about the origin will
of the disc is
be:
V y
3 m/s

8m

0 x

8 3
(A) rad/s (B) rad/s
2 2 289 8
(A) Iω − mv
I 24 8
(C) rad/s (D) rad/s
289 17
P hysi cs | 7.57

Q.16 Two points of a rigid body are moving as shown.


The angular velocity of the body is:

B
V

A
R (a) (b)
9 10
(A) 1 (B) (C) (D) None
10 9
v v 2v 2v
(A) (B) (C) (D)
2R R R 3R Q.20 A body kept on a smooth horizontal surface is
pulled by a constant horizontal force applied at the
Q.17 A yo-yo is released from hand with the string top point of the body. If the body rolls purely on the
wrapped around your finger. If you hold your hand still, surface, its shape can be:
the acceleration of the yo-yo is (A) Thin pipe (B) Uniform cylinder
(A) Downward, much greater than g (C) Uniform sphere (D) Thin spherical shell
(B) Downward much greater than g
(C) Upward, much less than g Q.21 A uniform rod AB of mass m and length l is at rest
on a smooth horizontal surface. An impulse j is applied
(D) Upward, much greater than g to the end B, perpendicular to the rod in the horizontal
(E) Downward, at g l
direction. Speed of point P at A distance from the
6
Q.18 Inner and outer radii of N a spool are r and R πml
center towards a of the rod after time t = is
respectively. A thread is wound over its inner surface 12J
J J
and placed over a rough horizontal surface. Thread is (A) 2 (B)
pulled by a force F as shown in figure. Then in case of m 2m
pure rolling. J J
(C) (D) 2
m m

Q.22 The moment of inertia of a solid cylinder about its


axis is given by (1/2) MR². If this cylinder rolls without
F slipping, the ratio of its rotational kinetic energy to its
translational kinetic energy is
(A) Thread unwinds, spool rotates anticlockwise and (A) 1: 1 (B) 2: 2 (C) 1: 2 (D) 1: 3
friction acts leftwards
(B) Thread unwinds, spool rotates clockwise and friction Q.23 A force F is applied to a dumbbell for a time
acts leftwards interval t, first as in (i) and then as in (ii). In which case
(C) Thread wind, spool moves to the right and friction does the dumbbell acquire the greater center-of-mass
acts rightwards. speed?

(D) Thread winds, spool moves to the right and friction m m


does not come into existence. F
F
Q.19 A sphere is placed rotating with its center initially
at rest in a corner as shown in figure (a) & figure (b). m m
Coefficient of friction between all surfaces and the (i) (ii)
1 f
sphere is . Find the ratio of the frictional force a by (A) (i)
3 fb
ground in situations (a) & (b). (B) (ii)
7 . 5 8 | Rotational Mechanics

(C) There is no difference Q.27 Choose the correct statement(s)


(D) The answer depends on the rotation inertia of the (A) The momentum of the ring is conserved
dumbbell
(B) The angular momentum of the ring is conserved
about its center of mass
Q.24 A hoop and a solid cylinder have the same
(C) The angular momentum of the ring is conserved
mass and radius. They both roll, without slipping on a
about a point on the horizontal surface.
horizontal surface. If their kinetic energies are equal
(D) The mechanical energy of the ring is conserved.
(A) The hoop has a greater translational speed then the
cylinder
(B) The cylinder has a greater translational speed then
Previous Years’ Questions
the hoop
(C) The hoop and the cylinder have the same Q.1 Let I be moment of inertia of a uniform square plate
translational speed about an axis AB that passes through its center and is
parallel to two of its sides. CD is a line in the plane of
(D) The hoop has a greater rotational speed then the the plate that passes through the center of the plate
cylinder. and makes an angle è with AB. The moment of inertia
of the plate about the axis CD is then equal to. (1998)
Q.25 A ball rolls down an inclined plane, as shown in (A) I (B) I sin2 θ
figure. The ball is first released from rest from P and
then later from Q. Which of the following statement (C) I cos2 θ (D) I cos2 (θ / 2)
is /are correct?
Q.2 A smooth sphere is moving on a frictionless
horizontal plane with angular velocity ω and center
of mass velocity v. It collides elastically and head on
P with an identical sphere B at rest, Neglect friction
2h everywhere. After the collision their angular speeds are
ωA and ωB respectively. Then, (1999)
h
(A) ωA < ωB (B) ωA = ωB
O
(C) ωA = ω (D) ωB = ω
(i) The ball takes twice as much time to roll from Q to O
as it does to roll from P to O.
Q.3 A particle of mass m is projected with a velocity
(ii) The acceleration of the ball at Q is twice as large as v making an angle of 45 ° with the horizontal. The
the acceleration at P. magnitude of the angular momentum of the projectile
(iii) The ball has twice as much K.E.at O when rolling about the point of projection when the particle is at its
from Q as it does when rolling from P. maximum height h is  (1990)

(A) i, ii only (B) ii, iii only (A) Zero (B) mv 3 /(4 2g)

(C) i only (D) iii only (C) mv 3 /( 2g) (D) m 2gh3

Q.26 If a person is sitting on a rotating stool with his Q.4 Consider a body, shown in figure, consisting of two
hands outstretched, suddenly lowers his hands, then identical balls, each of mass M connected by a light rigid
his rod. If an impulse J = Mv is imparted to the body at one
(A) Kinetic energy will decrease of its end, what would be its angular velocity? (2003)

(B) Moment of inertia will decrease


L
(C) Angular momentum will increase M M

(D) Angular velocity will remain constant


J=Mv

(A) v/L (B) 2v/L (C) v/3L (D) v/4L


P hysi cs | 7.59

Q.5 A tube of length L is filled completely with an Q.9 A cubical block of side L rests on a rough horizontal
incompressible liquid of mass M and closed at both the surface with coefficient of friction µ . A horizontal force
ends. The tube is then rotated in a horizontal plane about F is applied sufficient high, so that the block does not
one of its ends with a uniform angular velocity ω . The slide before toppling, the minimum force required to
force exerted by the liquid at the other end is (1992) topple the block is (2000)
N
Mω2L Mω2L Mω2L2
(A) (B) Mω2L (C) (D)
2 4 2
F
Q.6 A cylinder rolls up an inclined plane, reaches some
height and then rolls down (without slipping throughout G
these motions.) The directions of the frictional force L
acting on the cylinder are  (2002) L
2
(A) Up the incline while ascending and down the incline f1
while descending.
mg
(B) Up the incline while ascending as well as descending
(A) Infinitesimal (B) mg/4
(C) Down the incline while ascending and up the inline
while descending. (C) mg/2 (D) mg (1 – µ )

(D) Down the incline while ascending as well as


descending. Q.10 An equilateral triangle ABC formed from a uniform
wire has two small identical beads initially located at A.
The triangle is set rotating about the vertical axis AO.
Q.7 Two point masses of 0.3 kg and 0.7 kg fixed at the Then the beads are released from rest simultaneously
ends of a rod of length 1.4 m and of negligible mass. and allowed to slide down, one along AB and other
The rod is set rotating about an axis perpendicular to along AC as shown. Neglecting frictional effects, the
its length with a uniform angular speed. The point on quantities that are conserved as beads slide down are
the rod through which rotation of the rod is minimum,  (2000)
is located at a distance of  (1995)
A
(A) 0.42 m from mass of 0.3kg
g
(B) 0.70 m from mass of 0.7 kg
(C) 0.98 m form mass of 0.3 kg
(D) 0.98 m form mass of 0.7 kg
B C
O
Q.8 A disc of mass M and radius R is rolling with
angular speed ω on a horizontal plane as shown. The
(A) Angular velocity and total energy (kinetic and
magnitude of angular momentum of the about the
potential)
origin O is  (1999)
y (B) Total angular momentum and total energy
(C) Angular velocity and moment of inertia about the
axis of rotation
(D) Total angular momentum and moment of inertia
about the axis of rotation.


M
Q.11 One quarter section is cut from a O uniform
x circular disc of radius R. This section has a mass M. It is
O
made to rotate about a line perpendicular to its plane
1 and passing through the center of the original disc. Its
(A)   MR 2 ω (B) MR 2 ω
2 moment of inertia about the axis of rotation is  (2001)

3
(C)   MR 2 ω (D) 2MR 2 ω
2
7 . 6 0 | Rotational Mechanics

a
v
M

O
(A) 3v / 4a (B) 3v / 2a

1 1 1 (C) 3 / 2a (D) Zero


(A) MR 2 (B) MR 2 (C) MR 2 (D) 2MR 2
2 4 8
Q.15 A thin wire of length L and uniform linear mass
Q.12 A circular platform is free to rotate in a horizontal density ρ is bent into a circular loop with center at O
plane about a vertical axis passing through its center. A as shown. The moment of inertia of the loop about the
tortoise is sitting at the edge of the platform. Now the axis XX’ is (2000)
platform is given an angular velocity ω0
x x’
When the tortoise moves along a chord of the platform o
90
with a constant velocity (with respect to the platform).
The angular velocity of the platform ω (t) will vary with
time t as  (2002) O

(t) (t)

(A) 0 (B) 0
ρL3 ρL3 5ρL3 3ρL3
(A) (B) (C) (D)
8π2 16π2 16π2 8π2
t t
Q.16 A diatomic molecule is made of two masses m1
and m2 which are separated by a distance r. If we
calculate its rotational energy by applying Bohr’ s rule
(t) (t)

of angular momentum quantization, its energy will be


(C) 0 (D) 0 given by (n is an integer) (2012)

(m1 + m2 )
2
n2 2 n2 2
(A) (B)
2 (m1 + m2 ) r 2
t t
2m12 m22 r 2
(A)

2n2 2 (m1 + m2 ) n2 2
Q.13 A thin circular ring of mass M and radius r is (C) (D)
rotating about its axis with a constant angular velocity
(m1 + m2 ) r2 2m1 m2 r 2
ω . Two objects, each of mass m2 are attached gently
to the opposite ends of a diameter of the ring. The Q.17 A hoop of radius r and mass m rotating with an
wheel now rotates with an angular velocity  (2006) angular velocity ω0 is placed on a rough horizontal
(A) wM/(M + m) surface. The initial velocity of the centre of the hoop is
zero. What will be the velocity of the centre of thehoop
(B) w(M – 2m)/(M + 2m) when it ceases to slip? (2013)
(C) wM/(M + 2m)
r ω0 r ω0 r ω0
(D) w(M + 2m)/M (A) (B) (C) r ω0 (D)
3 2 4

Q.14 A cubical block of side a moving with velocity v on Q.18 A bob of mass m attached to an inextensible
a horizontal smooth plane as shown. It hits at point O. string of length  is suspended from a vertical support.
The angular speed of the block after it hits O is  (1999) The bob rotates in a horizontal circle with an angular
speed w rad/s about the vertical. About the point of
suspension: (2014)
P hysi cs | 7.61

(A) Angular momentum changes in direction but not in Which of the following
 statements is false for the
magnitude. angular momentum L about the origin? (2016)
(B) Angular momentum changes both in direction and  R 
magnitude. (A) L mv 
= − a kˆ when the particle is moving from
 2 
(C) Angular momentum is conserved. C to D.
(D) Angular momentum changes in magnitude but not  R 
(B) L mv 
= + a kˆ when the particle is moving from
in direction.
 2 
B to C.
Q.19 The current voltage relation of diode is given  mv
R kˆ when the particle is moving from D to A.
by I e
= (
1000 V /T
)
− 1 mA , where the applied voltage V
(C) L =
2
is in volts and the temperature T is in degree Kelvin.  mv ˆ
If a student makes an error measuring ± 0.01 V while (D) L = − R k when the particle is moving from
2
measuring the current of 5 mA at 300 K, what will be A to B.
the error in the value of current in mA? (2014)
(A) 0.5 mA (B) 0.05 mA Q.22 A roller is made by joining together two cones
(C) 0.2 mA (D) 0.02 mA at their vertices O. It is kept on two rails AB and CD
which are placed asymmetrically (see figure), with its
axis perpendicular to CD and its centre O at the centre
Q.20 From a solid sphere of mass M and radius R a
of line joining AB and CD (see figure). It is given a light
cube of maximum possible volume is cut. Moment of
push so that it starts rolling with its centre O moving
inertia of cube about an axis passing through its center
parallel to CD in the direction shown. As it moves, the
and perpendicular to one of its faces is: (2015)
roller will tend to: (2016)
MR 2 4MR 2 4MR 2 MR 2
(A) (B) (C) (D) B D
16 2 π 9 3π 3 3π 32 2 π

Q.21 A particle of mass m is moving along the side of


O
a square of side ‘a’, with a uniform speed v in the x-y
plane as shown in the figure:
A C
y a
D
v
C (A) Turn right
a v v a (B) Go straight
A v B
(C) Turn left and right alternately
a
o
R (D) Turn left
45
O x
7 . 6 2 | Rotational Mechanics

JEE Advanced/Boards

Exercise 1
0.3m C
Q.1 A thin uniform rod of mass M and length L is
0.1m
hinged at its upper end, and released from rest in a String
horizontal position. Find the tension at a point located B
at a distance L/3 from the hinge point, when the rod A
becomes vertical.

Q.6 A uniform wood door has mass m, height h, and


Q.2 A rigid body in shape of a triangle has VA = 5 m/s width w. It is hanging from two hinges attached to one
downwords, VB = 10 m/s downwords. Find velocity of side; the hinges are located h/3 and 2h/3 from the
point C. bottom of the door.
C

W
a

A a B
com
Hinges h
Q.3 A rigid horizontal smooth rod AB of mass 0.75 kg
and length 40 cm can rotate freely about a fixed vertical
axis through its mid-point O. Two rings each of mass 1
kg are initially at rest at a distance of 10 cm from O on
either side of the rod. The rod is set in rotation with an
Suppose that m = 20.0 kg, h = 2.20m, and W = 1.00 m
angular velocity of 30 rad per second. Find the velocity
and the bottom smooth hinge is not screwed into the
of each ring along the length of the rod in m/s when
door frame, find the forces acting on the door.
they reach the ends of the rod.

Q.7 A thin rod AB of length a has variable mass per unit



 x
length ρ0  1 +  where x is the distance measured
A C D B  a 
O from a and ρ0 is a constant
(a) Find the mass M of the rod.
(b) Find the position of center of mass of the rod.
Q.4 A straight rod AB of mass M and length L is placed
(c) Find moment of inertia of the rod about an axis
on a frictionless horizontal surface. A horizontal force
passing through A and perpendicular to AB. Rod is
having constant magnitude F and a fixed direction starts
freely pivoted at A and is hanging in equilibrium when
acting at the end A. The rod is initially perpendicular to
it is struck by a horizontal impulse of magnitude P at
the force. Find the initial acceleration of end B.
the point B.
(d) Find the angular velocity with which the rod begins
Q.5 A wheel is made to roll without slipping, towards
to rotate.
right, by pulling a string wrapped around a coaxial
spool as shown in figure. With what velocity the string (e) Find minimum value of impulse P if B passes through
should be pulled so that the center of wheel moves a point vertically above A.
with a velocity of 3 m/s?
P hysi cs | 7.63

Q.8 Two separate cylinders of masses m (=1 kg) and 4m


r
and radii R (=10cm) and 2R are rotating in clockwise
direction with ω1 = 100rad/sec and ω2 = 200 rad/
sec. Now they are held in contact with each other as
in figure Determine their angular velocity after the
slipping between the cylinders stops.

Q.12 A thin wire of length L and uniform linear mass


density ρ is bent into a circular loop with center at O as
Q.9 A spool of inner radius R and outer radius 3R shown in the figure. The moment of inertia of the loop
has a moment of inertia = MR2 about an axis passing about the axis XX` is
through its geometric center, where M is the mass of
the spool. A thread wound on the inner surface of the X X'
spool is pulled horizontally with a constant force = O
Mg. Find the acceleration of the point on the thread
which is being pulled assuming that the spool rolls
purely on the floor.

Q.13 A block X of mass 0.5 kg is held by a long massless


string on a frictionless inclined plane of inclination 30°
to the horizontal. The string is wound on a uniform
solid cylindrical drum Y of mass 2 kg and of radius
0.2 m as shown in the figure. The drum is given an initial
angular velocity such that the block X stands moving
up the plane. (g = 9.8 m/s²)

Q.10 A sphere of mass m and radius r is pushed onto


Y
a fixed horizontal surface such that it rolls without
slipping from the beginning. Determine the minimum
X
speed v of its mass center at the bottom so that it rolls
completely around the loop of radius (R + r) without
leaving the track in between.

(i) Find the tension in the string during the motion.


(R+r) (ii) At a certain instant of time the magnitude of the
Sphere angular velocity of Y is 10rad/sec. Calculate the distance
travelled by X form that instant of time unit it comes to
rest.

Q.11 Two uniform cylinders each of mass m = 10 kg Q.14 A uniform rod AB of length L and mass M is lying
and radius r = 150 mm, are connected by a rough belt on a smooth table. A small particle of mass m strikes
as shown. If the system is released from rest, determine the rod with a velocity v 0 at point at distance from the
center O. The particle comes to rest after collision. Find
(a) The tension in the portion of the belt connecting the
the value of x, so that of the rod remains stationary just
two cylinder.
after collision.
(b) The velocity of the center of cylinder a after it has
moved through 1.2 m.
7 . 6 4 | Rotational Mechanics

B
m V0 m V0

R R
m v0
C
x (i) Calculate the velocity of the center of mass of the
O disk at t0
(ii) Assuming the coefficient of friction to be µ calculate
A t0 . Also calculate the work done by the frictional force
as a function of time and the total work done by it over
Q.15 A uniform plate of mass m is suspended in each of a time t much longer then t0 .
the ways shown. For each case determine immediately
after the connection at B has been released:
Q.18 A circular disc of mass 300 gm and radius 20 cm
can rotate freely about a vertical axis passing through
Pin support its center of mass o. A small insect of mass 100 gm
is initially at a point A on the disc (which is initially
stationary). The insect starts walking from rest along the
A B
rim of the disc with such a time varying relative velocity
that the disc rotates in the opposite direction with a
(1/2)C constant angular acceleration = 2 π rad/s². After some
time T, the insect is back at the point A. By what angle
has the disc rotated till now, as seen by a stationary
C earth observer? Also find the time T.
(I) y

Q.19 A uniform disc of mass m and radius R rotates


x about a fixed vertical axis passing through its center
with angular velocity ω . A particle of same mass m and
(a) The angular acceleration of the plate. having velocity 2 ω R towards center of the disc collides
(b) The acceleration of its mass center. with the disc moving horizontally and stick to its rim.
Find

Q.16 A carpet of mass ‘M’ made of inextensible (a) The angular velocity of the disc
material is rolled along its length in the form of cylinder (b) The impulse on the particle due to disc.
of radius ‘R’ and is kept on a rough floor. The carpet
starts unrolling without standing on the floor when (c) The impulse on the disc due to hinge.
a negligibly small push is given to it. The horizontal
velocity of the axis of the cylindrical parts of the carpet Q.20 The door of an automobile is open and
when its radius decreases to R/2 will be: perpendicular to the body. The automobile starts with
an acceleration of 2 ft/sec², and the width of the door
is 30 inches. Treat the door as a uniform rectangle, and
neglect friction to find the speed of its outside edge as
seen by the driver when the door closes.

R/2 Q.21 A 20 kg cabinet is mounted on small casters that


R allow it to move freely ( µ = 0) on the floor. If a 100 N
force is applied as shown, determine.
Q.17 A uniform disk of mass m and radius R is projected
horizontally with velocity v 0 on a rough horizontal floor
so that it starts off with a purely sliding motion at t = 0.
After t0 seconds it acquires a purely rolling motion as
shown in figure.
P hysi cs | 7.65

Exercise 2
G Single Correct Choice Type
F
0.9m Q.1 Let I1 and I2 be the moment of inertia of a uniform
h square plate about axes APC and OPO` respectively as
I
shown in the figure. P is center of square. The ratio 1
0.6m of moment of inertia is I2
(a) The acceleration of the cabinet,
O
(b) The range of values of h for which the cabinet will A B
not tip.
P
Q.22 Two thin circular disks of mass 2 kg and radius
10 cm each are joined by a rigid massless rod of length
20 cm. The axis of the rod is along the perpendicular to D C
the planes of the disk through their center. The object O’
is kept on a truck in such a way that the axis of the
1 1
object is horizontal and perpendicular to the direction (A) (B) 2 (C) (D) 1
of motion of the truck. Its friction with the floor of the 2 2
truck is large enough so that the object can roll on
the truck without slipping. Take x-axis as the vertically Q.2 Moment of inertia of a rectangular plate about an
upwards direction. If the truck has an acceleration of axis passing through P and perpendicular to the plate
9m/s² calculate. is I. Then moment of PQR about an axis perpendicular
to the plane.

P Q
O

20cm
S R
(a) The force of friction on each disk.
(A) About P = I/2 (B) About R = I/2
(b) The magnitude and the direction of the frictional
torque acting on each disk about the center of mass O (C) About P > I/2 (D) About R > I/2
of the object. Express the torque in the vector form of
unit vectors in the x-y and z direction. Q.3 Find the moment of inertia of a plate cut in shape
of a right angled triangle of mass M, AC=BC=a about
Q.23 Three particles A, B, C of mass m each are joined an axis perpendicular to plane, side the plane of the
to each other by mass less rigid rods to form an plate and passing through the mid-point of side AB.
equilateral triangle of side a. Another particle of mass
m hits B with a velocity v 0 directed along BC as shown. A
The colliding particle stops immediately after impact.
O
(i) Calculate the time required by the triangle ABC to
complete half-revolution in its subsequent motion. (ii)
What is the net displacement of point B during this
interval? C B

Ma2 Ma2 Ma2 2Ma2


(A) (B) (C) (D)
12 6 3 3

Q.4 Let I be the moment of inertia of a uniform square


plate about an axis AB that passes through its center
7 . 6 6 | Rotational Mechanics

and is parallel to two of its sides. CD is a line in the


plane of the plate that passes through the center of the P
plate and makes an angle θ with AB. The moment of L
inertia of the plate about the axis CD is then equal to
(A) I (B) I sin² θ 

(C) I cos² θ (D) I cos² ( θ /2)


(A)  (2/3) gcos  θ (B) (2 / 3)g

Q.5 A heavy seesaw (i.e., not mass less) is out of balance. 3
(C) g (D)   gcos θ
A light girl sits on the end that is tilted downward, and 2
a heavy body sits on the other side so that the seesaw
now balances. If they both move forward so that they (E) (3 / 2)g
are one-half of their original distance from the pivot
point (the fulcrum) what will happen to the seesaw?
Q.8 A mass m is moving at speed v perpendicular to a
(A) The side the body is sitting on will tilt downward rod of length d and mass M = 6m which pivots around
(B) The side the girl is sitting on will once again tilt a frictionless axle running through its center. It strikes
downward and sticks to the end of the rod. The moment of inertia
of the rod about its center is Md² /12. Then the angular
(C) Nothing; the seesaw will still be balanced speed of the system right after the collision is.
(D) It is impossible to say without knowing the masses (A) 2v / d (B) 2v / (3d)
and the distances.
(C) v / d (D) 3v / (2d)
Q.6 A pulley is hinged at the center and a mass less
thread is wrapped around it. The thread is pulled with Q.9 A sphere of mass M and radius R is attached by a
a constant force F starting from rest,. As the time light rod of length l to a point P. The sphere rolls without
increases, slipping on a circular track as shown. It is released from
the horizontal position. The angular momentum of the
F system about P when the rod becomes vertical is:

l
P

(A) Its angular velocity increases, but force on hinge


remains constant
(B) Its angular velocity remains same, but force on
hinge increases
10 10 2
(C) Its angular velocity increases and force of hinge (A) M gl[l + R] (B) M gl[l + R]
7 7 5
increases
(D) Its angular velocity remains same and force on 10 7
(C) M gl[l + R] (D) None of the above
hinge is constant 7 5

Q.7 A uniform flag pole of length L and mass M is


Q.10 A ladder of length L is slipping with its ends
pivoted on the ground with a frictionless hinge. The
against a vertical wall and a horizontal floor. At a
flag pole makes an angle θ with the horizontal. The
certain moment, the speed of the end in contact with
moment of inertia of the flag pole about one end is
the horizontal floor is v and the ladder makes an angle
(1/3) ML². If it starts falling from the position shown in
α = 30 ° with the horizontal. Then the speed of ladder’s
the accompanying figure, the linear acceleration of the
center must be
free end of the flag pole – labeled P – would be:
(A) 2v 3 (B) v/2 (C) v (D) None
P hysi cs | 7.67

Q.11 In the previous question, if dv/dt = 0, then the (C) x  = vt +  0.5b sin ( 3vt / b ) ,  y =
 0.5c =  0.5b cos ( 3vt / b )
angular acceleration of the ladder when α = 45 ° is
 0.5vt +  0.5b sin ( 3vt / b ) ,  y =
(D) x  =  0.5b cos ( 3vt / b )
(A) 2v 2 /L2 (B) v 2 /2L2
Q.14 A yo-yo is resting on a perfectly rough horizontal
(C) 2[v 2 L2 ] (D) None table. Forces F1 , F2 and F3 are applied separately as
shown. The correct statement is
Q.12 A uniform circular disc placed on a rough horizontal
surface has initially a velocity v 0 and an angular velocity F3 F2
ω0 as shown in the figure. The disc comes to rest after
moving some distance in the direction of motion.
v0
Then is
rω0
F1
0

(A) When F3 is applied the center of mass will move to


V0 the right.
(B) When F2 is applied the center of mass will move to
the right.
(A) 1/2 (B) 1 (C) 3/2 (D) 2 (C) When F1 is applied the center of mass will move to
the right.
Q.13 An ice skater of mass m moves with speed 2v to
(D) When F2 is applied the center of mass will move to
the right, while another of the same mass m moves
the right.
with speed v toward the left, as shown in figure I. Their
paths are separated by a distance b. At t = 0, when they
are both at x = 0, they grasp a pole of length Multiple Correct Choice Type

y Q.15 A rod of weight w is supported by two parallel


knife edges and B and is in equilibrium in a horizontal
m 2v position. The knives are at a distance d from each other.
The center of mass of the rod is at a distance x from A.
x wx
b (A) The normal reaction at a is
d
m w(d − x)
t<0 v (B) The normal reaction at a is
d
wx
(C) The normal reaction at B is
b and negligible mass. For r > 0 consider the system d
as a rigid body of two masses m separated by distance w(d − x)
(D) The normal reaction at B is
b, as shown in figure II. Which of the following is the d
correct formula for motion after t = 0 of the skater
initially at y = b/2? Q.16 A block with a square base measuring and height
h, is placed on an inclined plane. The coefficient of
y
friction is µ . The angle of inclination (α) of the plane is
gradually increased. The block will
b/2 a
>x (A) Topple before sliding if µ >
t=0 h
a
(A) x   2=
= vt,  y   b / 2  (B) Topple before sliding if µ <
h
vt +  0.5 b sin ( 3vt / b ) ,  y =
(B) x  =  0.5b cos ( 3vt / b )
7 . 6 8 | Rotational Mechanics

a Q.20 A hollow sphere of radius R and mass m is fully


(C) Slide before toppling if µ >
h filled with non-viscous liquid of mass m. It is rolled
a down a horizontal plane such that its center of mass
(D) Slide before toppling if µ < moves with a velocity v. If it purely rolls
h
5
(A) Kinetic energy of the sphere is mv 2
Q.17 A particle falls freely near the surface of the earth. 6
Consider a fixed point O (not vertically below the 4
particle) on the ground. (B) Kinetic energy of the sphere is mv 2
3
(A) Angular momentum of the particle about O is
(C) Angular momentum of the sphere about a fixed
increasing.
8
(B) Torque of the gravitational force on the particle point on ground is mvR
3
about O is decreasing.
(C) The moment of inertia of the particle about O is (D) Angular momentum of the sphere about a fixed
decreasing. 14
point on ground is mvR
5
(D) The angular velocity of the particle about O is
increasing.
Q.21 In the figure shown, the plank is being pulled to
the right with a constant speed v. If the cylinder does
Q.18 The torque τ on a body about a given point is not slip then:
found to be equal to a × L where a constant vector is
and L is the angular momentum of the body about that
point. From this it follows that
(A) dL/dt is perpendicular to L at all instants of time R

(B) The components of l in the direction of a does not


change with time. V
(C) The magnitude of l does not change with time.
(D) L does not change with time.
(A) The speed of the center of mass of the cylinder is 2v.
Q.19 In the given figure. a ball strikes a uniform rod of (B) The speed of the center of mass of the cylinder is
same mass elastically and rod is hinged at point A. Then zero.
which of the statement (S) is /are correct?
(C) The angular velocity of the cylinder is v/R.
A
(D) The angular velocity of the cylinder is zero.

Q.22 A disc of circumference s is at rest at a point A on


u a horizontal surface when a constant horizontal force
begins to act on its center. Between A and B there is
sufficient friction to prevent slipping and the surface is
(A) Linear momentum of system (ball + rod) is smooth to the right of B. AB = s. The disc moves from A
conserved. to B in time T. To the right of B,

(B) Angular momentum of system (ball + rod) about


the hinged point A is conserved. Force
(C) Kinetic energy of system (ball + rod) before the
collision is equal to kinetic energy of system just after A B
the collision
(A) The angular acceleration of disc will disappear,
(D) Linear momentum of ball is conserved. linear acceleration will remain unchanged
(B) Linear acceleration of the disc will increase
P hysi cs | 7.69

(C) The disc will make one rotation in time T/2. Q.27 A man spinning in free space changes the shape
of his body, eg. By spreading his arms or curling up. By
(D) The disc will cover a distance greater then s in
doing this, he can change his
further time T.
(A) Moment of inertia
Q.23 A rigid object is rotating in a counterclockwise (B) Angular momentum
sense around a fixed axis. If the rigid object rotates
(C) Angular velocity
though more than180 ° but less than 360 ° , which of
the following pairs of quantities can represent an initial (D) Rotational kinetic energy
angular position and a final angular position of the
rigid object. Q.28 A ring rolls without slipping on the ground. Its
(A) 3 rad, 6 rad (B) –1 rad, 1 rad center C moves with a constant speed u. P is any point
on the ring. The speed of P with respect to the ground
(C) 1 rad, 5 rad (D –1rad, 2.5 rad
is v.
(A) 0 ≤ v ≤ 2u
Q.24 ABCD is a square plate with center O. The moments
of inertia of the plate about the perpendicular axis (B) v = u, if CP is horizontal
through O is I and about the axes 1, 2, 3 & 4 are I1 , I2 ,
(C) v = u is CP makes an angle of 30 ° with the horizontal
I3 , & I 4 respectively. If follows that:
and P is below the horizontal level of c.
1 (D) v = 2 u, if CP is horizontal
A 2
B
Q.29 A small ball of mass m suspended from the ceiling
at a point O by a thread of length  moves along a
3
O horizontal circle with a constant angular velocity ω .

C
D 4
(A) I2 = I3 (B) I = I1 + I 4

(C) I = I2 + I 4 (D) I1 = I3

Q.25 A body is in equilibrium under the influence of m C


a number of forces. Each force has a different line of
action. The minimum number of forces required is
(A) Angular momentum is constant about O
(A) 2, if their lines of action pass through the center of
(B) Angular momentum is constant about C
mass of the body.
(C) Vertical component of angular momentum about O
(B) 3, if their lines of action are not parallel.
is constant
(C) 3, if their lines of action are parallel.
(D) Magnitude of angular momentum about O is
(D) 4, if their lines of action are parallel and all the forces constant.
have the same magnitude.
Q. 30 If a cylinder is rolling down the incline with sliding.
Q.26 A block of mass m moves on a horizontal rough
(A) After some time it may start pure rolling
surface with initial velocity v. The height of the center
of mass of the block is h from the surface. Consider a (B) After sometime it will start pure rolling
point a on the surface in line with the center of mass. (C) It may be possible that it will never start pure rolling
(A) Angular momentum about a is mvh initially (D) None of these.
(B) The velocity of the block decreases as time passes.
(C) Torque of the forces acting on block is zero about a. Q.31 Which of the following statements are correct.

(D) Angular momentum is not conserved about A. (A) Friction acting on a cylinder without sliding on an
7 . 7 0 | Rotational Mechanics

inclined surface is always upward along the incline Q. 34 A wheel of radius r is rolling on a straight line,
irrespective of any external force acting on it the velocity of its center being v. At a certain instant
the point of contact of the wheel with the grounds is M
(B) Friction acting on a cylinder without sliding on an
and N is the highest point on the wheel (diametrically
inclined surface may be upward may be downwards
opposite to M). The incorrect statement is:
depending on the external force acting on it
(A) The velocity of any point P of the wheel is
(C) Friction acting on a cylinder rolling without sliding
proportional to MP
may be zero depending on the external force acting
on it (B) Points of the wheel moving with velocity greater
than v form a larger area of the wheel than points
(D) Nothing can be said exactly about it as it depends
moving with velocity less than v
on the frictional coefficient on inclined plane.
(C) The point of contact M is instantaneously at rest
Q. 32 A plank with a uniform sphere placed on it rests (D) The velocities of any two parts of the wheel which
on a smooth horizontal plane. Plank is pulled to right are equidistant from center are equal.
by a constant force F. If sphere does not slip over the
plank. Which of the following is correct?
Q.35 A ring of mass M and radius R sliding with a
velocity v 0 suddenly enters into a rough surface where
e the coefficient of friction is µ , as shown in figure.

> V0
(A) Acceleration of the center of sphere is less than that
of the plank Rough ()

(B) Work done by friction acting on the sphere is equal Choose the correct statement(s)
to its total kinetic energy
(A) As the ring enters on the rough surface, the limiting
(C) Total kinetic energy of the system is equal to work frictional force acts on it
done by the force F
(B) The direction of friction is opposite to the direction
(D) None of the above. of motion.
(C) The frictional force accelerates the ring in the
Q. 33 a uniform disc is rolling on a horizontal surface. clockwise sense about its center of mass
At a certain instant B is the point of contact and A is at
(D) As the ring enters on the rough surface it starts
height 2R from ground, where R is radius of disc.
rolling.

Q.36 Choose the correct statement (s)


(A) The ring starts its rolling motion when the center of
mass is stationary
B
(B) The ring starts rolling motion when the point of
contact becomes stationary
(A) The magnitude of the angular momentum of the v0
disc about B is thrice that about A (C) The time after which the ring starts rolling is
2µg
(B) The angular momentum of the disc about A is v0
anticlockwise (D) The rolling velocity is
2
(C) The angular momentum of the disc about B is
clockwise Q.37 Choose the correct alternative (s)
(D) The angular momentum of the disc about A is equal
(A) The linear distance moved by the center of mass
to that about B.
3v 20
before the ring starts rolling is
8µg
P hysi cs | 7.71

3
(B) The net work done by friction force is − mv 20
8 A
mv 20
(C) The loss is kinetic energy of the ring is
4
X Y
mv 20
(D) The gain in rotational kinetic energy is +
8

Q.38 A tightrope walker in a circus holds a long flexible


pole to help stay balanced on the rope. Holding the
pole horizontally and perpendicular to the rope helps Statement-II: About any point in the plane, the torque
the performer. of gravity force and normal contact force by ground
balance each other

Q.41 Statement-I: The angular velocity of all the points


on the laminar rigid body lying in the plane of a body
as seen from any other point on it is the same.
Statement-II: The distance between any 2 points on
the rigid body remains constant.

Q.42 Consider the following statements:-


Statement-I: The moment of inertia of a rigid body
(A) By lowering the overall center-of- gravity
reduces to its minimum value as compared to any other
(B) By increasing the rotation inertia parallel axis when the axis of rotation passes through its
center of mass.
(C) In the ability to adjust the center- of -gravity to be
over the rope. Statement-II: The weight of a rigid body always acts
through its center of mass in uniform gravitational field.
(D) In achieving the center of gravity to be under the
rope.
Q.43 Statement-I: The moment of inertia of any rigid
body is minimum about axis which passes through its
Assertion Reasoning Type
center of mass as compared to any other parallel axis.
(A) Statement-I is true, statement-II is true and
Statement-II: The entire mass of a body can be
statement-II is correct explanation for statement-I.
assumed to be concentrated at its center of mass for
(B) Statement-I is true, statement-II is true and statement- applying Newton’s force Law.
II is NOT the correct explanation for statement-I.
(C) Statement-I is true, statement-II is false. Q.44 A uniform thin rod of length L is hinged about
one of its ends and is free to rotate about the hinge
(D) Statement-I is false, statement-II is true.
without friction, Neglect the effect of gravity. A force
F is applied at a distance x from the hinge on the rod
Q.39 Consider the following statements such that force is always perpendicular to rod. As the
Statement-I: a cyclist always bends inwards while value of x is increased from zero to L,
negotiating a curve Statement-I: The component of reaction force by hinge
Statement-II: By bending he lowers his center of on the rod perpendicular to length of rod increases.
gravity of these statements. Statement-II: The angular acceleration of rod increases.

Q.40 Statement-I A disc A moves on a smooth Q.45 Statement-I: For a round shape body of radius R
horizontal plane and rebounds elastically from a rolling on a fixed ground, the magnitude of velocity of
smooth vertical wall (Top view is shown in Fig 7.166), its center is given by ωR , where ω is its angular speed.
in this case about any point on line XY the angular
momentum of the disc remains conserved. Statement-II: When distribution of mass is symmetrical
then center of round shape body is its center of mass.
7 . 7 2 | Rotational Mechanics

Paragraph 2:
F
A uniform rod is fixed to a rotating turntable so that its
x lower end is on the axis of the turntable and it makes
an angle of N20°to the vertical. (The rod is thus rotating
with uniform angular velocity about a vertical axis
Q.46 Statement-I: a body cannot roll on a smooth passing through one end.) If the turntable is rotating
horizontal Surface. clockwise as seen from above.
Statement-II: when a body rolls purely, the point of
contact should be at rest with respect to surface. Q.51 What is the direction of the rod’s angular
momentum vector (calculated about its lower end)?
Comprehension Type (A) Vertically downwards
Paragraph 1: (B) Down at 20°to the horizontal
The figure shows an isosceles triangular plate of mass (C) Up at 20° to the horizontal
M and base L. The angle at the apex is 90°. The apex lies (D) Vertically upwards
at the origin and base is parallel to X – axis

 Q.52 Is there torque acting on it, and if so in what


direction?
M
(A) Yes, vertically
(B) Yes, horizontal
x
(C) Yes at 20°to the horizontal

Q.47 The moment of inertia of the plate about the z – (D) No


axis is
ML2 ML2 ML2 Paragraph 3:
(A) (B) (C) (D) None of these
12 24 6 In the following problems, indicate the correct direction
of friction force acting on the cylinder, which is pulled
Q.48 The moment of inertia of the plate about the x on a rough surface by a constant force F.
axis is
ML2 ML2 ML2 ML2 Q.53 A cylinder of mass M and radius R is pulled
(A) (B) (C) (D)
8 32 24 6 horizontal by a force F. The frictional force can be given
by which of the following diagrams
Q.49 The moment of inertia of the plate about its base
parallel to the x – axis is C F

ML2 ML2 ML2


(A) (B) (C) (D) None of these
18 36 24 FF FF
(A)
(A) C
C (B) C
(B) C
 
Q.50 The moment of inertia of the plate about the y –
axis is
20
o
(C) C FF (D)
(D) Cannot
Cannot be
be interpret
interpret
(C) C
=0
=0
Q.54 A cylinder is pulled horizontally by a force F acting
at a point below the center of mass of the cylinder, as
shown in figure. The frictional force can be given by
ML2
which of the following diagrams?
ML2 ML2
(A) (B) (C) (D) None of these
6 8 24
P hysi cs | 7.73

F Previous Years’ Questions


C

Q.1 A thin uniform angular disc (See figure) of mass


M has outer radius 4R and inner radius 3R. The work
(A) C
F (B) C F required to take a unit mass from point P on its axis to
  infinity is  (2010)
P
F (D) Cannot be interpret 4R
(C) C
=0 3R 4R

Q.55 A cylinder is pulled horizontally by a force F acting


at a point above the center of mass of the cylinder, as 2GM 2GM
(A) (4 2 − 5) (B) − (4 2 − 5)
shown in figure. The frictional force can be given by 7R 7R
which of the following diagrams 2GM
GM
(C) (D) ( 2 − 1)
4R 5R
F
C
Q.2 A solid sphere of radius R has moment of inertia
I about its geometrical axis. If it’s moment of inertia
F F about the tangential axis (which is perpendicular to
(A) C (B) C
plane of the disc), is also equal to I, then the value of r

=0 is equal to  (2006)
l
(C) F (D) Cannot be interpret
C

r

Q.56 A cylinder is placed on a rough plank which in


turn is placed on a smooth surface. The plank is pulled
with a constant force F. The frictional force can be given 2 2 3 3
(A) R (B) R (C) R (D) R
by which of the following diagrams. 15 5 15 15

Q.3 A block of base 10 cm × 10 cm and height 15 cm


C
F is kept on an inclined plane. The coefficient of friction
between them is 3 . The inclination θ of this inclined
plane from the horizontal plane is gradually increased
(A) (A)
C C (B) C(B) C (C) C(C) C from 0°. Then,  (2009)
(A) At θ = 30°, the block will start sliding down the plane
   
=0 =0
(B) The block will remains at rest on the plane up to
C (C) C (D) Cannot be interpreted.
 certain θ and then it will topple
(C) At θ = 60°, the block will start sliding down the plane
=0
and continue to do so at higher angles
(D) At θ = 60°, the block will start sliding down the plane
and on further increasing θ, it will topple at certain θ.
7 . 7 4 | Rotational Mechanics

Q.4 From a circular disc of radius R and mass 9M, a (C) hA = hC ; KB = K C


small disc of radius R/3 is removed from the disc. The
(D) hA < hC ; KB > K C
moment of inertia of the remaining disc about axis
perpendicular to the plane of the disc and passing
through O is  (2010) Q.8 A child is standing with folded hands at the center
of platform rotating about its central axis. The kinetic
energy of the system is K. The child now stretches
R/3
his arms so that the moment of inertia of the system
doubles. The kinetic energy of system now is  (2004)
K K
(A) 2K (B) (C) (D) 4 K
2 4

O Q.9 Consider a body, shown in figure, consisting of two


R
identical balls, each of mass M connected by a light rigid
rod. If an impulse J = Mv is imparted to the body at one
of its end, what would be its angular velocity?  (2003)
L
40 37
(A) 4 MR² (B) MR 2 (C) 10MR² (D) MR 2 M
9 9

Q.5 Let I be moment of inertia of a uniform square plate J=Mv


about an axis AB that passes through its center and is
(A) v / L (B) 2v / L (C) v / 3L (D) v / 4L
parallel to two of its sides. CD is a line in the plane and
makes an angle θ with AB. The moment of inertia of the
plate about the axis CD is then equal to  (1998) Q.10 A disc is rolling (without slipping)
on a horizontal surface. C is its center Q
(A) I (B) I sin² θ
and Q and P are two points equidistant C
(C) I cos² θ (D) I cos² (θ/2) from C. Let vP , v Q and v C be the P
magnitude of velocity of points P, Q,
Q.6 A solid sphere is in pure rolling motion on an
and C respectively, then  (2004)
inclined surface having inclination θ (2006)
(A) Frictional force acting on sphere is f = µ (A) v Q > v C> vP

(B) f Is dissipative force (B) v Q < v C< vP


1
(C) Friction will increase its angular velocity and (C) v Q = vP , v C = v
2 P
decrease its linear velocity (D) v Q < v C > vP
(D) If θ decreases, friction will decrease
Paragraph 1: Two discs A and B are mounted coaxially
Q.7 A ball moves over a fixed track as shown in figure. on a vertical axle. The discs have moments of inertia I
From A to B the ball rolls without slipping. If surface BC and 2I respectively about the common axis. Disc A is
is frictionless and K A , KB and K C are kinetic energies imparted an initial angular velocity 2 ω using the entire
of the ball at A, B and C respectively, then  (2006) potential energy of a spring compressed by a distance
x1. Disc B is imparted an angular velocity ω by a spring
C having the same spring constant and compressed by
a distance x2. Both the discs rotate in the clockwise
hA r direction.  (2007)

x1
B Q.11 The ratio is
x2
(A) hA > hC ; KB > K C
1 1
(B) hA > hC ; K C > K A (A) 2 (B) (C) 2 (D)
2 2
P hysi cs | 7.75

Q.12 When disc B is brought in contact with disc A, Q.16 The center of mass of the disk undergoes simple
they acquire a common angular velocity in time t. The harmonic motion with angular frequency ω equal to
average frictional torque on one disc by the other
during this period is. (A)
k
(B)
2k
(C)
2k
(D)
4k
M M 3M 3M
2Iω 9Iω 9Iω 3Iω
(A) (B) (C) (D)
3t 2t 4t 2t
Q.17 The maximum value of v 0 for which the disk will
Q.13 The loss of kinetic energy during the above roll without slipping is
process is
Iω2 Iω2 Iω2 Iω2 M M 3M 5M
(A) (B) (C) (D) (A) µg (B) µg (C) µg (D) µg
2 3 4 6 k 2k k 2k

Q.14 A small object of uniform Q.18 A thin uniform rod, pivoted at O, is rotating in
density rolls up a curved the horizontal plane with constant angular speed ω , as
surface with an initial velocity shown in the figure. At time t = 0, a small insect starts
V
v. If reaches up to a maximum from O and moves with constant speed v, with respect
height of 3v /4g with respect
2
to the rod towards the other end. It reaches the end
to the initial position. The object is of the rod at t = T and stops. The angular speed of
(A) Ring (B) Solid sphere the system remains ω throughout. The magnitude of

(C) Hollow sphere (D) Disc ( )
the torque τ about O, as a function of time is best

represented by which plot?  (2012)


Paragraph 2: A uniform thin cylindrical disk of mass
M and radius R is attached to two identical mass less
springs of spring constant k which are fixed to the wall
as shown in the figure. The springs are attached to the (A) l l (B) l l
disk diametrically on either side at a distance d from its
center. The axle is mass less and both the springs and the r r
axle are in a horizontal plane. The un-stretched length O T O T
of each spring is L. The disk is initially at its equilibrium
position with its center of mass (CM) at a distance L from
the wall. The disk rolls without slipping with velocity
 (C) l l (D) l l
v 0 = v 0 î . The coefficient of friction is µ . (2008)

r r
O T O T

y
Q.19 A small mass m is attached to a massless string
whose other end is fixed at P as shown in the figure.
The mass is undergoing circular motion in the x-y
2d

planewith centre at O and constant angular speed ω .


If the angular momentum of the system, calculated
 
about O and P are denoted by LO and LP respectively,
then  (2012)
R z
v0

P
x

Q.15 The net external force acting on the disk when its O m
center of mass is at displacement x with respect to its
equilibrium position is

2kx 4kx  
(A) −kx (B) −2kx (C) − (D) − (A) LO and LP do not vary with time.
3 3
7 . 7 6 | Rotational Mechanics

 
(B) LO varies with time while LP remains constant. Q.22 Consider a disc rotating in the horizontal plane
with a constant angular speed ω about its centre O. The
 
(C) LO remains constant while LP varies with time. disc has a shaded region on one side of the diameter
  and an unshaded region on the other side as shown in
(D) LO and LP both vary with time. the figure. When the disc is in the orientation as shown,
two pebbles P and Q are simultaneously projected at an
angle towards R. The velocity of projection is in the y-z
Q.20 A lamina is made by removing a small disc of
plane and is same for both pebbles with respect to the
diameter 2R from a bigger disc of uniform mass density
disc. Assume that (i) they land back on the disc before
and radius 2R, as shown in the figure. The moment of
inertia of this lamina about axes passing though O 1
the disc has completed rotation (ii) their range is less
and P is I0 and IP respectively. Both these axes are 8
perpendicular to the plane of the lamina. The ratio than half the disc radius and (iii) ω remains constant
IP / IO to the the nearest integer is  (2012) throughout. Then  (2012)

R 
y
2R
x Q
2R P O
O

(A) P lands in the shaded region and Q in the unshaded


Q.21 Two identical discs of same radius R are rotating region.
abouttheir axes in opposite directions with the same
(B) P lands in the unshaded region and Q in the shaded
constant angular speed ω . The discs are in the same
region.
horizontal plane. At time t = 0, the points P and
(C) Both P and Q land in the unshaded region.

  (D) Both P and Q land in the shaded region.

P Q Paragraph for Questions 23 and 24


R R
The general motion of a rigid body can be considered
to be a combination of (i) a motion of its centre of mass
about an axis, and (ii) its motion about an instantaneous
Q are facing each other as shown in the figure. The
axis passing through the centre of mass. These axes
relative speed between the two points P and Q is vr
need not be stationary. Consider, for example, a thin
In one time period (T) of rotation of the discs, vr as a
uniform disc welded (rigidly fixed) horizontally at its
function of time is best represented by - (2012)
rim to a massless stick, as shown in the figure. When
vr vr
the disc-stick system is rotated about the origin on a
horizontal frictionless plane with angular speed ω , the
(A)
(A) (B)
(B)
motion at any instant can be taken as a combination of
O T
t
O T
t
(i) a rotation of the centre of mass of the disc about
vr vr the z-axis, and (ii) a rotation of the disc through an
(C)
(C) (D)
(D) instantaneous vertical axis passing through its centre of
mass (as is seen from the changed orientation of points
O T
t
O T
t P and Q). Both these motions have the same angular
speed ω in this chase.
P hysi cs | 7.77

z with the horizontal. Then with respect to the horizontal


 surface, (2012)
z

Q P P Q
y 

x /2
Now consider two similar systems as shown in the P
3R
figure: Case (a) the disc with its face vertical and parallel R
30o x
to x-y plane; Case (b) the disc with its face making an O
angle of 45° with x-y plane and its horizontal diameter
2R
parallel to x-axis. In both the cases, the disc is welded
at point P, and the systems are rotated with constant
angular speed ω about the z-axis.
z z
Q
(A) The point O has a linear velocity 3Rωˆi
 
Q

45o 11 ˆ 3
(B) The point P has a linear velocity Rω kˆ
y y
P P Rω i +
4 4
x x
13 ˆ 3
Case (a) Case (b) (C) The point P has a linear velocity Rω i − Rω kˆ
4 4
Q.23 Which of the following statements about the  3 1
instantaneous axis (passing through the centre of mass) (D) The point P has a linear velocity  3 −  Rω ˆi + Rω kˆ
 4  4
is correct?  (2012)  

(A) It is vertical for both the cases (a) and (b) (B) It is
Q.26 Two solid cylinders P and Q of same mass and
vertical for case (a); and is at 45° to the x-z plane and
same radius start rolling down a flixed inclined plane
lies in the plane of the disc for case (b)
from the same height at the same time. Cylinder P has
(C) It is horizontal for case (a); and is at 45° to the x-z most of its mass concentrated near its surface, while Q
plane and is normal to the plane of the disc for case (b) has most of its mass concentrated near the axis. Which
statement(s) is (are) correct? (2012)
(D) It is vertical for case (a); and is at 45° to the x-z
plane and is normal to the plane of the disc for case (b) (A) Both cylinders P and Q reach the ground at the
same time.
Q.24 Which of the following statements regarding the (B) Cylinder P has larger linear acceleration than
angular speed about the instantaneous axis (passing cylinder Q.
through the centre of mass) is correct? (2012)
(C) Both cylinders reach the ground with same
(A) It is 2 ω for both the cases translational kinetic energy
ω (D) Cylinder Q reaches the ground with larger angular
(B) It is ω for case (a); and for case (b)
2 speed

(C) It is ω for case (a); and 2 ω for case (b)


Q.27 A uniform circular disc of mass 50 kg and radius
(D) It is ω for both the cases 0.4 m is rotating with an angular velocity of 10 rad
s-1 about its own axis, which is vertical. Two uniform
circular rings, each of mass 6.25 kg and radius 0.2 m,
Q.25 The figure shows a system consisting of (i) a ring
are gently placed symmetrically on the disc in such a
of outer radius 3R rolling clockwise without slipping
manner that they are touching each other along the
on a horizontal surface with angular speed ω and (ii)
axis of the disc and are horizontal. Assume that the
an inner disc of radius 2R rotating anti-clockwise with
friction is large enough such that the rings are at rest
angular speed ω / 2 . The ring and disc are separated
relative to the disc and the system rotates about the
by frictionless ball bearings. The system is in the x-z
original axis. The new angular velocity (in rad s-1) of the
plane. The point P on the inner disc is at a distance
system is (2013)
R from the origin, where OP makes an angle of 30°
7 . 7 8 | Rotational Mechanics

Q.28 In the figure, a ladder of mass m is shown leaning


against a wall. It is in static equilibrium making an angle
θ with the horizontal floor. The coefficient of friction
between the wall and the ladder is µ1 and that between
the floor and the ladder is µ2 . The normal reaction of
the wall on the ladder is N1 and that of the floor is N2 .
If the ladder is about to slip, then  (2014)
Q.31 Two identical uniform discs roll without slipping on
1 two different surfaces AB and CD (see figure) starting at
A and C with linear speeds v1 and v 2 , respectively ,and
always remain in contact with the surfaces. If they reach
B and D with the same linear speed and v1 = 3 m / s ,

then v 2 in m/s is (g = 10 m/s2)  (2015)
2

mg v1=3m/s v2
(A) µ1 = 0 µ2 ≠ 0 and N2 tan θ = A C
2 30m 27m

mg
B D
(B) µ1 ≠ 0 µ2 =0 and N1 tan θ =
2
mg
(C) µ1 ≠ 0 µ2 ≠ 0 and N2 =
1 + µ1 µ2
Q.32 A ring of mass M and radius R is rotating with
mg angular speed ω about a fixed vertical axis passing
(D) µ1 = 0 µ2 ≠ 0 and N1 tan θ =
2 through its centre O with two point masses each of
M
Q.29 A uniform circular disc of mass 1.5 kg and radius 0.5 mass at rest at O. These masses can move radially
8
m is initially at rest on a horizontal frictionless surface. outwards along two massless rods fixed on the ring
Three forces of equal magnitude F = 0.5 N are applied
simultaneously along the three sides of an equilateral as shown in the figure. At some instant the angular
triangle XYZ with its vertices on the perimeter of the 8
disc (see figure). One second after applying the forces, speed of the system is ω and one of the masses is at
9
the angular speed of the disc in rad s-1 is  (2014) 3
a distance of R from O. At this instant the distance of
5
F the other mass from O is (2015)
X


O
Y E
Z

F
O
Q.30 A horizontal circular platform of radius 0.5 m
and mass 0.45 kg is free to rotate about its axis. Two
massless spring toy -guns, each carrying a steel ball of
mass 0.05 kg are attached to the platform at a distance 2 1 3 4
(A) R (B) R (C) R (D) R
0.25 m from the centre on its either sides along its 3 3 5 5
diameter (see figure).Each gun simultaneously fires the
balls horizontally and perpendicular to the diameter
in opposite directions. After leaving the platform, the Q.33 The densities of two solid spheres A and B of the
balls have horizontal speed of 9ms-1 with with respect r
to the ground. The rotational speed of the platform in same radii R vary with radial distance r as ρA (r ) =
k 
r
5 R 
rad s-1 after the balls leave the platform is (2014)
and ρB (r ) =k   , respectively, where k is a constant.
R 
The moments of inertia of the individual spheres about
P hysi cs | 7.79

 
axes passing through their centres are I A and IB ,
I n
(C) The force F is given by F= (ˆi + 2 ˆj) N
respectively. If B = , the value of n is  (2015)
I A 10 
(D) The torque τ = − ( 20 / 3) kˆ N m
Q.34 A uniform wooden stick of mass 1.6 kg and length
 rests in an inclined manner on a smooth, vertical
Q.36 Two thin circular discs of mass m and 4m, having
wall of height h ( <  ) such that a small portion of the
radii of a and 2a, respectively, are rigidly fixed by a
stick extends beyond the wall. The reaction force of the
wall on the stick is perpendicular to the stick. The stick massless, rigid rod of length  = 24 a through their
makes an angle of 30° with the wall and the bottom of centers. This assembly is laid on a firm and flat surface,
the stick is on a rough floor. The reaction of the wall on and set rolling without slipping on the surface so that
the stick is equal in magnitude to the reaction of the the angular speed about the axis of the rod is ω . The
floor on the stick. The ratio h /  and the frictional force angular momentum of the entire assembly about the
(
fat the bottom of the stick are g = 10 ms−2 (2016)) 
point ‘O’ is L (see the figure). Which of the following
statement(s) is(are) true?  (2016)
h 3 16 3
(A)
= = ,f N
 16 3 4m

h 3 16 3 m
(B)
= = ,f N
 16 3 z
 
h 3 3 8 3 2a
(C)
= 2 = ,f N 
 16 3 O
a

h 3 3 16 3
(D)
= = ,f N
 16 3

(A) The magnitude of angular momentum of the


 assembly about its center of mass is 17 ma2 ω / 2
Q.35 The position vector r of a particle of mass m is
 
given by the following equation r ( t ) = α t3 ˆi + β t2 ˆj, where (B) The magnitude of the z-component of L is 55 ma2 ω
= / 3 ms−3 , β 5 ms−2 and m = 0.1 kg . At t = 1 s,
α 10 = (C) The magnitude of angular momentum of center of
mass of the assembly about the point O is 81 ma2 ω
which of the following statement(s) is(are) true about
(D) The center of mass of the assembly rotates about
the particle?  (2016)
the z-axis with an angular speed of ω / 5

(A) The velocity v is given
=

v (10 ˆi + 10ˆj) ms −1


(B) The angular momentum L with respect to the origin

is given by L = − (5 / 3) kˆ Nms
7 . 8 0 | Rotational Mechanics

PlancEssential Questions
JEE Main/Boards JEE Advanced/Boards
Exercise 1 Exercise 1
Q.19 Q.22 Q.23 Q.5 Q.7 Q.8
Q.27 Q.28 Q.10 Q.11 Q.21
Q.24

Exercise 2 Exercise 2
Q.12 Q.25 Q.7 Q.9 Q.21
Q.22 Q.28 Q.41
Q.54
Previous Years’ Questions
Q.4 Q.7 Q.9 Previous Years’ Questions
Q.12 Q.1 Q.3

Answer Key

JEE Main/Boards
Q.11 KB> K A
Exercise 1 Q.18 8 g
Q.19 1.584 × 10⁷ g cm²
Q.1 kg m2, [M1L2T0], No
Q.20 1.25 kg m²;
Q.2 Torque Q.21 9.83 × 10³⁷ kg m²
Q.3 Inertia
Q.22 3.5 × 10⁴ g cm² ; 1.75 × 10⁴ g cm² ; 8.75 × 10⁴ g
Q.4 Theorem of parallel axes and theorem of cm² ; 10.5× 104 g cm²
perpendicular axes
5 2
2 Q.23 mL
Q.5 I = MR 2 , M = mass & R = radius 4
5
2 Q.24 0.01J
Q.6 I = MR 2 , M = mass & R = radius
3 Q.26 108 days
Q.8 No
Q.27 80π kg m2 s−1
Q.9 Hollow sphere
Q.10 IB> IA Q.28 360 r.p.m
P hysi cs | 7.81

Exercise 2

Single Correct Choice Type


Q.1 B Q.2 B Q.3 A Q.4 B Q.5 D Q.6 D
Q.7 D Q.8 B Q.9 B Q.10 C Q.11 A Q.12 D
Q.13 B Q.14 B Q.15 C Q.16 B Q.17 B Q.18 B
Q.19 B Q.20 A Q.21 D Q.22 C Q.23 C Q.24 B
Q.25 D Q.26 B Q.27 C

Previous Years’ Questions


Q.1. A Q.2 C Q.3 B Q.4 A Q.5 A Q.6 B
Q.7 C Q.8 C Q.9 C Q.10 B Q.11 A Q.12 C
Q.13 C Q.14 A Q.15 D Q.16 D Q.17 B Q.18 A
Q.19 C Q.20 B Q.21 A, C Q.22 D

JEE Advanced/Boards 3L2ρ


Q.12
8π2
Exercise 1
Q.13 1.65 N, 1.224 m
Q.1 2Mg
Q.14 L/6
Q.2 5 5 m/s
1.2g 1.2g  ˆ
Q.3 3 Q.15
(i)(a) (i)
(i)(a)
(a) (clockwise)  
(clockwise)  (b) −i  + 
(b) − 0.3g( 0.3g(i +  2 ˆj) 
2 j)      
c c
2.4g
Q.4 2F/M (ii) (a)
(ii)(a) (clockwise) (b) 0.5g
c
Q.5 2m/s
14gR
→ → Q.16 V =
Q.6 FA = (–133.64 î + 196 ˆj )N and FB = 133.64 î 3

2v 0 v0
3aρ0 5a 7ρ0 a3 12 Q.17 (i) v = (ii) t =
Q.7 (a) ; (b) ; (c) ; (d) ; 3 3µg
2 9 12 7ρ0 a2
1
w = 1 [3µ22mg22 t22 − 2µ mg t v 0 ](t < t0 ),
(e)
7 2 3
ρ ga w = 2 [3µ mg t − 2µ mg t v 0 ](t < t0 ),
4 0 2 1
w = − 1 mv 202 (t > t0 )
w = − 6 mv 0 (t > t0 )
6
Q.8 300 rad/sec, 150 rad/sec
Q.18 t = 2 5 sec, q = 4π/5 rad
Q.9 16 m/s²
37 37
Q.19 (a) ω / 3 , (b) m ω R, (c) mωR
27 3 3
Q.10 v = gR
7
Q.20 15 ft/sec
200 3
Q.11 (a) N; (b) 4 m/s Q.21 (a) 5 m/s2, (b) 0.3 < h < 1.5 m
7 7
7 . 8 2 | Rotational Mechanics

6aπ a
Q.22 6N, −0.6jˆ ± 0.6kˆ Q.23=
(i) t ;=
(ii) s 1 + (2π + 3)2
3v 0 3

Exercise 2

Single Correct Choice Type


Q.1 D Q.2 C Q.3 B Q.4 A Q.5 B Q.6 A
Q.7 D Q.8 B Q.9 D Q.10 C Q.11 A Q.12 A
Q.13 C Q.14 C

Multiple Correct Choice Type


Q.15. B, C Q.16 A, D Q.17 A, C, D Q.18 A, B, C Q.19 B, C Q.20 B, C
Q.21 B, C Q.22 B, C, D Q.23 C, D Q.24 A, B, C, D Q.25 B, C Q.26 A, B, D
Q.27 A, C, D Q.28 A, C, D Q.29 B, C, D Q.30 A, C Q.31 B, C Q.32 A, C
Q.33 A, B, C Q.34 A, B, C Q.35 A, B, C Q.36 B, C, D Q.37 A, B, C, D Q.38 B, C

Assertion Reasoning Type


Q.39 B Q.40 B Q.41 A Q.42 B Q.43 B Q.44 D
Q.45 B Q.46 D

Comprehension Type
Q.47 C Q.48 A Q.49 C Q.50 C Q.51 B Q.52 B
Q.53 A Q.54 A Q.55 D Q.56 B

Previous Years’ Questions


Q.1 A Q.2 A Q.3 B Q.4 A Q.5 A Q.6 D
Q.7 A Q.8 B Q.9 A Q.10 A Q.11 C Q.12 A
Q.13 B Q.14 D Q.15 D Q.16 D Q.17 C Q.18 B
Q.19 C Q.20 C Q.21 A Q.22 C, D Q.23 A Q.24 D
Q.25 A, B Q.26 D Q.27 8 Q.28 C, D Q.29 2 Q.30 4
Q.31 7 Q.32 D Q.33 6 Q.34 D Q.35 A, B, D Q.36 A, D
P hysi cs | 7.83

Solutions

JEE Main/Boards Sol 5: Moment of inertia of a solid sphere about its


diameter is
Exercise 1 I=
2
MR2
5
Sol 1: Moment of inertia of any body is given by where M = mass of the sphere
I = ∫ r 2dm where ‘dm’ is mass of a small element under R = Radius of the sphere
consideration and ‘r’ is the distance between the axis of
Derivation:
rotation and the element. Then,
S.I. units of moment of inertia will be m2 kg or kg.m2
R
DIMENSIONS – dm
[M0LT0]2[M1L0T0] = [M1L2T0] r
dr
It is not a vector quantity since direction is nowhere
considered.

Sol 2: Torque is the rotational analogue of force. Considering, the solid sphere as a large group of hollow
spheres whose radii range from 0 to R with a thickness
Sol 3: Moment of inertia is the rotational analogue of of dr. Integrating moment of inertia of these dements
mass of a body. gives the required value.
dm = ρ. 4pr2.dr (dm = ρ. dv) and I = ∫ dI
Sol 4: The theorem of parallel axes R
2 ∴
I = Icm + m.d 2 I= ∫ 3 .r
2
ρ. 4pr2dr ( I = ∫ r 2dm ) and
0
where Icm = Moment of Inertia about an axis passing 2
through center of mass and parallel to the considered (Ihollow sphere = MR2)
3
axis R
2 2  r5 
m = mass of the body ⇒ dI = (dm)r2 = . 4pρ  
3 3  5  0
d = distance between the axis (about which the value
of I is required) and the axis passing through center of  M 
R4 2
mass and parallel to the considered axis. = 4pρ . = MR  ρ = 4 π 3 
2 
5 5 R
The theorem of perpendicular axes  3 
Iz = Ix + Iy
z Sol 6: The moment of inertia of a hollow sphere about
2
an axis passing through its center is I = MR2
3
where M = mass of sphere
R = Radius of sphere
x
Derivation:

y Considering the hollow sphere, as a large group of


rings with thickness dr and radii ranging from 0 to R.
This theorem is applicable for planar bodies only and Integrating the moment of inertia of these rings we will
Ix and Iy should be about two perpendicular axes lying get the required moment of inertia.
in plane.
7 . 8 4 | Rotational Mechanics

∴ Ihollow sphere > Isolid sphere


r dm = . 2rRd
Sol 10: Given,
 
d R d Iz
m m

Ix

I= ∫ dI = ∫ r 2dm = ∫ r. ρ 2πrRdθ
Iy

[ Iring = MR ] 2
Circular discs A and B of same mass and same thickness
but different densities dA and dB (dA > dB)
⇒ dI = dmr2 ⇒I= ∫ 2πρR (R
3
sin3θ)dq
π MA = MB
3
I = 2prR4 ∫ sin θdθ
0 ⇒ πR 2A . dA tA = πRB2 . dB .tB
π 1/2
MR 2 (3sin θ – sin3θ) RA d 
⇒I=
2 ∫ 4
. dq ⇒ =  B 
0 RB  dA 

MR 2
π
 3cos θ cos3θ  MAR 2A
⇒I= (IA)x = = (IA)y
– +  4
2 0
4 12 
MAR 2A
MR 2
 3 1  3 1  2 ⇒ (IA)z = (IA)x + (IA)y =
⇒I= –  – +   ⇒ I = MR2 2
 –
2  4 12  4 12   3 [By perpendicular axes theorem]
also
Sol 7: Factors on which moment of inertia depend
MBRB2
→ Mass of the body (IB)z =
2
→ Mass distribution of the body 2
IA R  d
→ Size of the body =  A  = B ⇒ IA < IB since dB < dA
IB  RB  dA
→ Axis about which moment of inertia is required
Sol 11: Given.
Sol 8: No, it is not a constant
Moment of inertia of two rotating bodies A and B as
It depends on the axis about which moment of inertia
IA and IB (IA > IB)
is calculated
and
Iaxis
Since, K = Angular moment (LA and LB) are equal
M
⇒ LA = LB
Iaxis = moment of inertia about a given axis
then kinetic energies
M = mass of the body (constant)
(K.E.)A and (K.E.)B will be
Sol 9: Given 1 2 1  1 2 1 1 L2 
L A / I A and L2B / IB  K.E.= Iω = Lω= 
A solid sphere and hollow sphere have same mass and 2 2  2 2 2 I 
same radius
2 ⇒ (K.E.)A < (K.E.)B since IB < IA
Isolid sphere = MR2
5
2
Ihollow sphere = MR2 Sol 12: Moment of inertia is the measure of tendency
3
P hysi cs | 7.85

of a body to resist rotational motion if it was at rest or  2π 


1 1 2
 ∫ cos θ dθ = 0 
resist being stopped it was rotating. = I ω2 + MVcm
2 cm 2  0 
Radius of Gyration is the radius of the circle is which
a zero sized particle of mass M (which is equal to the 1 1
∴ K.E. = Icm ω2 + MVcm
2
mass of a body considered) such that the moment of 2 2
inertia of both the particle and the body about any axis
are equal
Sol 14: Laws of rotational motion
axis First law: Every body has tendency to be in rest or is
state of rotation unless acted upon by a torque

M Second law: The torque applied on a body is moment of


inertia times the angular acceleration of the body J – Iα
M K
MK2
Third law: Every action (torque) has an equal and
opposite reaction (torque) on the body which gave the
I I= MK2 action.

I
⇒K= Sol 15: Newton’s second law of linear motion is
M
F = ma
Sol 13: Kinetic energy of rolling motion where F = force acting on the body
m = mass of body
a = acceleration of body
w Vcm Now, consider ‘dm’ part of as body acted upon by a force
F and the body is rotating with an angular acceleration
of ‘α’
Consider a body rolling with angular velocity ω and
linear center of mass velocity vcm
Velocity of a particle at any point is given by dm
(V + rω cosθ) î + (rω sinθ) ˆj
cm F r

 Vcm+r cos 
 
 dT = r × F
r
= r. dm (rα)
r sin 

= r2 . dm . α
K.E. = ∫ d(K.E.)
∫ dT = ∫ r 2dm.α
∫ 2 dm ((Vcm + rω cos θ) )
1 2
= + (rω sin θ)2
⇒ Taxis = Iaixs aaxis
M M
1 2 2 1 2
=
2 ∫r ω dm +
2 ∫ Vcm dm + Sol 16: Principle of conservation of angular momentum.
0 0
In absence of a torque, the angular momentum of a
M
1 body is always conserved (constant)
2 ∫ 2Vcm r ω cos θ dm
0 dL
m T= =0
1 1 2 dt
= Icm ω2 + Vcm
2 2
+ Vcm ω ∫ r cos θ dm ⇒ L = constant ⇒ Iω = constant
0
R 2π
1 1 or I1ω1 = I2ω2
= Icm ω2 + MVcm
2
+ Vcm ω ∫ ∫ r cos θ .dθ .dr
2 2 0 0
7 . 8 6 | Rotational Mechanics

Example: By stretching hands a ballet decreases the angular Sol 21: Given,
speed of the body by increasing the moment of inertia
Radius of earth assuming it as a sphere as 6400 km
Example: A person sitting is a chair which can rotate mass of earth is 6 × 1024 kg
holds a rotating wheel in hands and when he flips the
Moment of inertia of the earth is
wheel, the person along with chair rotates conserving
angular momentum. 2 ∴ 2
I= MR2 ( I = MR2 for a sphere (solid))
5 5
Sol 17: Consider a circular ring of radius R and mass M 2
⇒I= × 6 × 1024 × (6400 × 103)2
5
⇒ I = 9.8304 × 1037 kgm2

R
dm Sol 22: Given,
A Uniform circular disc of mass 700 gms and diameter
2 2 20 cm
I= ∫ dI = ∫ R .dm = MR
(i) Moment of inertia about the transverse axis through
center of disc is
Sol 18: Given,
MR 2
I = 200 g cm2 I=
2
r = 5 cm 700 × 10 × 10
⇒ I= = 3.5 × 104 gcm2
We know that for a thin ring the moment of inertia 2
about an axis passing through center is Mr2 (ii) Moment of inertia about the diameter of disc is
T MR 2
⇒M= = 8 grams I=
2
r 4

[ Ix = Iy and Ix + Iy = Iz
Sol 19: Given,
(perpendicular axis theorem)]
diameter of disc = 40 cm
z
thickness of disc = 7 cm
density of disc = 9 gm cm–3
x
πD2
mass of the disc = ρ.V = ρ. .t
4
y
= 9 × π × 400 × 7 = 79168.13 grams
700 × (10 × 10)
moment of inertia of disc about a transverse axis ⇒ I= =1.75×104gcm2
through the center of the disc is 4

MR 2
I=
2
79168.13 × 20 × 20 R
⇒I= ⇒ I = 1.584 × 107 g cm2 (iii)
2

Sol 20: Given,


A uniform circular disc and A Uniform circular ring of
Moment of inertia about a tangent is plane is
same mass of 10 kg and diameter of 1 m
I = Id + MR2 (parllel axis theorem)
Moment of inertia of disc about a transverse axis
1 5MR 2
through center of the disc is I = MR2 ⇒I=
2 4
2
1 1 5
⇒ I = × 10 ×   = 1.25 kg m2 ⇒I= × (700) × (10 × 10) = 8.75 × 104 gcm2
2 2 4
P hysi cs | 7.87

A circular disc of mass 1 kg and radius 0.2 m rotating


(iv) about transverse axis passing through its center.
R Moment of inertia about the given axis as
MR 2 1
I= ⇒ I = × (0.2)2 = 0.02 kg–m2
2 2
30
Given, disc makes rotations per minute then,
Ic I π

Moment of inertia about a tangent perpendicular to 30 1


Angular velocity = 2π × × rad/s
the plane is 11 60
= 1 rad/s
I = IC + MR2
1 2
3 kinetic energy = Iw
⇒I= MR2 2
2
1
3 = × 0.02 × (1)2 = 0.01 joules
⇒ I= ×700 ×10 ×10 = 10.5 ×104 gcm2 2
2

Sol 25: Given,


Sol 23: x

C
D w V
1 60°
2
L L
A circular dice of mass M and radius r is set rolling on
table the kinetic energy of the disc is given by
60°
A B
1 2 1
L K.E = Iω + mv2(refer Q. 14)
2 2
Given, particles of masses m are placed at A, B and C 1 mr 2 2 1
⇒ K.E. = ω + mv2
and side of triangle ABC is L 2 2 2
Then, mv 2 mv 2
⇒ K.T. = +
4 2
AC AB L
CD = AC cos60° = = =
2 2 2 [For pure rolling v = rω]
moment of inertia of system is
3
I = I A + IB + IC ⇒ K.E. = mv2
4
2
L 
⇒ I = m(0)2 + m(L)2 + m  
2 Sol 26: Given,

5mL2 Sun rotates around itself once in 27 days angular


⇒I=
4 2π
velocity of sum = rad/days
27
Sol 24: Given, If it expands to truce its present diameter, then moment
of inertia becomes 4
2
∴I= MR2 (for sphere)
5

I2 R 22
=
I1 R12

⇒ I2 = 4I1
7 . 8 8 | Rotational Mechanics

I 1 ω1 = I 2 ω2 Exercise 2
(By principle of conservation of angular momentum)
Single Correct Choice Type
ω1 ω1
⇒ ω2 = = rad/days
4 108
Sol 1: (B)
Then, the new period of revolution
A

= = 108 days B MR 2
(2π / 108) I=
12
∴ π MR 2
T =  I=
 ω 12

Sol 27: Given, MR 2


MR 2 I=
I= 6
A 40 kg of flywheel is form of a uniform circular disc 1 2
meter in radius is making 120 r.p.m. C
1
Angular velocity = 120 × 2π × rad/s
60
= 4π rad/s sphere
Moment of inertia about transverse axis passing
through center is
MR 2 2MR 2
I= I=
2 5
40 × (1)2 B< C < A
⇒I= = 20 kg–m2
2
angular momentum = I.ω Sol 2: (B)
= 80 π kgm2/s = 251.33 kgm2/s

Sol 28: Given,


Angular velocity of the system = 120 r.p.m Ma2 I=Ma2
I=
Moment of inertia of the system initially = 6 kgm2 2
Moment of inertia of the system finally = 2 kgm2 I=
M(2a)2
12
I 1ω1 = I 2 ω2
M(2a)2
(By principle of conservation of angular momentum) I=
12
I1
⇒ ω2 = × ω1 = 360 r.p.m.
I2
2Ma2
∴ Final angular velocity = 360 r.p.m I=
3

D C

2a

A B

M
Mass of each rod =
4
P hysi cs | 7.89

Moment of inertia of one rod about the given axis is Sol 5: (D) Given,
 M(2a)2  Ma2 4Ma2 Ma2 I’ as a function of x of a rigid body
I=   + = =
 4 × 12  4 4×3 3 I = 2x2 – 12x + 27
 
Ma2 The value of moment of inertia is minimum at center
Total moment of inertia = 4I =
3 of mass point
To calculate min value of I, differentiate w.r.t. x and equate
Sol 3: (A) Given, it to 0

I1 is the moment of inertia about perpendicular bisector dI


=0
of rod dx
I1 ⇒ 4x – 12 = 0
⇒x=3
2
ML
I1 =
12 To check whether it is minimum,

d2 I
>0
Now rod is bent into semi-circular arc dx2

⇒4>0
∴ x = 3 is the x-coordinate of center of mass.

Sol 6: (D)

R 3m
Length of arc = L = pR A B
Moment of inertia of the arc = MR2
2N 4N
ML2
= = I2``
π2 To keep the body in equilibrium a 6 N acts at point x
from A
I2 > I1 (since p2 < 12)

x 6N
1 1
Sol 4: (B) So,=
2I
2
(M + M) R2 ⇒ 2I =
2
( 2M) R2 A B
P
2N 4N
Imaginary
R semi-disc Taking moment of torques about A
of same
specific list ∑ MA = 0 (In equilibrium)

⇒6×x–4×3=0
Mars=M
Radius=R ⇒ x = 2m

1 Sol 7: (D)
⇒ I = M R2 1 2
2 T1 T2
L/4
A B
L C
L/2
w
7 . 9 0 | Rotational Mechanics

Let T1 and T2 be tensions is the strings considering force Sol 10: (C) Given,
equilibrium
Two spheres of same mass M and radii R and 2R and
T1 + T2 = W also have equal rotational kinetic energies
Also Considering moment equilibrium about A 1 1 L2 L2
⇒ I1 ω12 = I2 ω22 ⇒ 1 = 2
2 2 I1 I2
∑ MA = 0 ∴
[ Iω= L (Angular momentum)]
3L L
⇒ T2 × –W× =0
4 2 L2 I2 R 22 R2
⇒ = = = =2
2W W L1 I1 R12 R1
⇒ T2 = ⇒ T1 =
3 3
Sol 11: (A) Angular momentum remains constant since,
Sol 8: (B) RA T no torque is acting on the skater.

A B While kinetic energy increases, since,


1
m K.E. = L.ω.
2
and as L = constant and ω increase K.E. increases
3
Sol 12: (D) Energy is not conserved in this case because
C W=mg the disc is fixed at its center and a force is acting on it
Center of mass of the triangular plate is at a distance when the child jumps.
 But Angular momentum can be conserved since No
of from AC.
3 torque is present in the boy-disc system
Considering force equilibrium ∴ Initial angular momentum = final angular momentum
RA + T = W = mg (I + mR2)ω = Iω’ + MRv
Considering torque or moment equilibrium about B (I + mR 2 )ω – mRv
⇒ ω’ =
 2  I
⇒ RA(  ) = W  
 3
2mg
⇒ RA = Sol 13: (B)
3

Sol 9: (B) Given, A constant torque is applied on a rod


which hinged the angular acceleration in rod will be 
/3
T
∝=
I
t
Tt
Angular velocity ω = ∫ α dt =
I
+ ω0
M2
0 Moment of inertia of the rod a bout pivot =
12
 Tt 
Power developed = T.ω = T.  + ω0  Since, the collision is elastic and the rod stops, velocity
 I 

If ω0 = 0 of the particle is v = .ω
3
T2 By principle of Angular Momentum,
P= (t)
I 
Pext Iω = m.v.
3
M2   3M
⇒ .ω = m. ω. ⇒ m =
Time 12 3 3 4
P hysi cs | 7.91

v 2  viˆ

Sol 14: (B)


C R
p v
2R
Sol 16: (B)
w v1  vjˆ R

The point P experiences centripetal acceleration


towards centre

∴ a = a î relative velocity
Angular velocity =
AB
the velocity of point P is
| viˆ – vjˆ | v
→ = =
v = v î + Rω ˆj 2R R


⇒ v = v î + v ˆj
Sol 17: (B)
T

[ v = Rω pure rolling]

⇒ v = v( î + ˆj )
→ →
a.v 1
cosθ = =
→ →
2
| a || v | mg
⇒ θ = 45°
mg – T T
Acceleration of yo-yo = =g–
m m
Sol 15: (C)
y
Sol 18: (B) In case of pure rolling v = Rω
3m/s (Bottom–most point has zero velocity)
8m

x
O
v
y
f
after 5 seconds w
15m
v
 f
8m  as ω is in clockwise direction, thread winds also friction
acts leftwards to increase w.
v cos

O

v cos θ v8 Sol 19: (B)


Angular velocity = =
 2
24 24 N1
= = rad/s w mg
82 + 152 289 f1

fa
Na

(a)
7 . 9 2 | Rotational Mechanics

6J
⇒ω=
m
f1=0
N1=0 B
mg w
L
.w
6 w B
fb
Nb vcm
vcm /2
1
f1 + Na = mg Nb = mg .w
2
N1 mg
+ Na = mg fb =
3 3
π m
fa ∴ After time t =
+ Na = mg [ N1 = fa] 12J
3
π m 6J π
The angle rotated by rod θ = ωt = . =
9mg  Na  3mg 12 J m 2
Na =  fa =  ⇒ fa =
10  3  10
2
 ω  2
fa 9 Velcoity of point P =   + (v cm )
=  6
fb 10
2 2
 J  J J
=   +  = 2
F m m m
Sol 20: (A)

Sol 22: (C) Solid cylinder rolls without slipping

⇒ vcm = Rω
F = Ma (By Newton’s second law), also T = Ta 1 2
(K.E.)rotational Iω
Ia ∴ a = 2
⇒ FR = ( α = for pvre rolling) (K.E.)translational 1
R R mv 2
2
⇒ I = MR2
(K.E.)r mR 2 ω2 1
This is satisfied for thin pipe ⇒ = =
(K.E.)t 2mv 2 2

Sol 21: (D)


T B Sol 23: (C) There will be no diff in velocity of centre of
mass
⇒ F = macm

C ∴ acm = same in both cases


/6
P

Sol 24: (B)


A
By conservation of linear momentum,
ˆj A hoop & solid cylinder of same mass
mvcm = J ⇒ vcm =
m ∴ Ihoop > Icylinder as mass is distributed away from center

By conservation of angular momentum, Iω = J. ∴ Since gain in potential energy in both case is same
2
P hysi cs | 7.93

Let P for hoop Previous Years’ Questions


2
1 1
P= mv2 + Ihoop  V  Sol 1: (A) A`B` ⊥ AB and C`D` ⊥ CD
2 2  
R 
2
From symmetry I AB = I A`B` and ICD = IC`D`
1 1
and P = mv2 + Icy  V  From theorem of perpendicular axes,
2 2 R 
 
A’
∴ Vhoop < Vcylinder C’ D

Sol 25: (D) (i) Upward acceleration = a = same for both


case A

B
1 4h
⇒ 2h = 0 × tQ + a × t2Q ⇒ tQ =
2 g
1 C D’
h = 0×tp + a × tp2 B’
2
2h IZZ = IAB + IA’B’ = ICD + IC’D’ = 2IAB = 2ICD
⇒ tp =
g
Alternate:
∴ tQ ≠ 2tp
The relation between I AB and ICD should be true for
(ii) The acceleration of both the balls is same = g sin q all value of θ

(iii) ∆KE = DPE At θ = 0, ICD = I AB

∴ ∆KEQ = 2mgH Similarly, at è = π /2 , ICD = I AB (by symmetry)

∆KEP = mgH Keeping these things in mind, only option (A) is correct.

∴ ∆KEQ = 2∆KEP
Sol 2: (C) Since, it is head on elastic collision between
two identical spheres, they will exchange their linear
Sol 26: (B) Moment of inertia decreases since mass is velocities i.e., A comes to rest and B starts moving
closer to axis while. with linear velocity v, As there is no friction anywhere,
torque on both the spheres about their center of mass
Angular momentum remains constant which implies
is zero and their angular velocity remains unchanged.
angular velocity increases and which intern implies
Therefore,
increase is kinetic energy.
∴ 1
( L = Iω and K.E = Lω) A=
2  v

A B A B
Sol 27: (C) Angular momentum is conserved on any Before collision After collision
point on the ground since the only force present passes
through that point making torque zero. v
Sol 3: (B) L = m r⊥
2

v 2 sin2 45° v2
Here, r⊥ = h = =
2g 4g

 v   v2  mv 3
∴L = m    =
 2   4g  4 2g
7 . 9 4 | Rotational Mechanics

Sol 4: (A) Let ω be the angular velocity of the rod. For work to be minimum, the moment of inertia (I)
Applying, angular impulse = change in angular should be minimum or
momentum about center of mass of the system dI
=0
L dx
J. = Ic ω
2
or 2(0.3) – 2 (0.7)(1.4 – x) = 0 or (0.3)x = (0.7)(1.4 – x)
L   ML2 
∴ (Mv)
=   (2)  ω (0.7)(1.4)
2 
 4  ⇒ x= = 0.98m
0.3 + 0.7
v
∴ ω=
L
Sol 8: (C) From the theorem
    
L0 = LCM + M (r × v)  … (i)
M M
We may write
J = Mv Angular momentum about O = Angular momentum
about CM + Angular momentum of CM about origin

∴ L0 = Iω + MRv
M
Sol 5: (A) Mass of the element dx is m = dx. 1 3
L = MR 2 ω + MR(Rω) = MR 2 ω
2 2
This element needs centripetal force for rotation.
y

y

v=R
F F+dF M  v
x=L x x
O B O
x dx (a) (b)
x=0 Note that is this case both the terms in
  
M  Eq. (i) i.e., LCM and M (r × v)
∴ dF = mx ω2 =  xω2dx 
L  Have the same direction. That is why we have used
L m L 2
Mω L L0 = Iω ~ MRv if they are in opposite direction as
∫0 dF = L .ω ∫0 xdx =
2
∴ F= shown in figure (b).
2
This is the force exerted by the liquid at the other end.
Sol 9: (C) At the critical condition, normal reaction N
will pass through point P. In this condition.
Sol 6: (B) mg sin θ component is always down the
τN = 0 = τfr (About P)
plane whether it is rolling up or rolling down. Therefore,
for no slipping, sense of angular acceleration should N
also be same in both the cases.
Therefore, force of friction f always act upwards. F

Sol 7: (C) Work done W= 1 Iω2 G


L
2
If x is the distance of mass 0.3 kg from the center of L
2
mass, we will have
f1
I = (0.3) x2 + (0.7)(1.4 – x2 )
mg
P hysi cs | 7.95

The block will topple when


Here, r 2 = a2 + [ R 2 − a2 − vt]2
L
τF > τmg or FL > (mg) Form conservation of angular momentum
2
mg ω0 I1 =
ω(t)I2
∴ F>
2
Substituting the values we can see that variation of ω
Therefore, the minimum force required to topple the (t) is non-linear.
block is
mg Sol 13: (C)∴ I1ω1 = I2ω2
F=
2
I1  Mr 2  M 
Sol 10: (B) Net external torque on the system is zero. ω2 = ω = ω =
2   ω
I2 2
 Mr + 2mr   M + 2m 
Therefore, angular momentum is conserved. Force
acting on the system are only conservative. Therefore,
total mechanical energy of the system is also conserved. a a2
Sol
= 14: (A) r 2 or r 2
=
2 2
Sol 11: (A) Mass of the whole disc = 4M a
Moment of inertia of the disc about the given axis M v M
a C r
1
= (4M)R 2 = 2MR 2

2 O O
∴ Moment of inertia of quarter section of the disc
Net torque about O is zero. Therefore, angular
1 1 momentum (L) about O will be conserved,
= (2MR 2 ) = MR 2
4 2 Or Li = L f
Note: These type of questions are often asked in a
objective. Students generally error in taking mass of the Mv  = I0 ω = (ICM + Mr 2 )ω
whole disc. They take if M instead of 4 M. 2
 Ma2   a2   2 2
=   + M =  ω Ma ω
Sol 12: (C) Since, there is no external torque, angular  6   2  3
   
momentum will remains conserved. The moment of
inertia will first decrease till the tortoise moves from 3v
ω=
A to Can then increase as it moves from C and D. 4a
Therefore, ω will initially increase and then decrease.
Let R be the radius of platform, m the mass of disc and Sol 15: (D) Mass of the ring M = ρL
M is the mass of platform.
Let R be the radius of the ring, then
Moment of inertia when the tortoise is at a
1
L=
2 π R or R =
MR 2 2π
I1 = mR 2+
2 Moment of inertia about an axis passing through O and
And moment of inertia when the tortoise is at B. parallel to XX` will be
1
I0 = MR 2
2
O Therefore, moment of inertia about XX` (from parallel
axis theorem) will be given by
r a
1 3
A D IXX' = MR 2 + MR 2 = MR 2
BC 2 2
vt Substituting values of m and R
2
MR  L2  3ρL3
I2 = mr 2+ 3
2 IXX' = (ρL)  =
2  4π2  8π2
 
7 . 9 6 | Rotational Mechanics

m2 r m1 r a2
Sol=
16: (D) r1 = ;r Moment of inertia about the required axis = I = ρa3 ,
m1 + m2 2 m1 + m2 6
nh M
(l1 + l)=
ω = n

where ρ =
4 3
πR
3
1 n  (m1 + m2 )
2 2
K.E.
=
2
( l1 + l2 ) ω=
2
 2R 
5
3M 1 32R5 4MR 2 4MR 2
2m1 m2 r 2 3M 1
=I =  = =
4 π R3 6  3 4 π R3 6 9 3 9 3 π 9 3 π
Sol 17: (B) From conservation of angular momentum
about any fix point on the surface  R
mr 2 ω
=0 2mr 2 ω Sol 21: (A,=
C) L0 mv
2
− kˆ ( ) [D to A]

ω0  R 
∴ω = =L0 mv  + a kˆ [C to D]
2  2 
ω0 r
∴ VCM =
2 Sol 22: (D) From normal reactions of roller, we can
conclude it moves towards left.
Sol 18: (A)

JEE Advanced/Boards

L

Exercise 1

Sol 1: Given,
V
A thin uniform rod of mass M and length L is hinged at
its upper end, and is released from rest in a horizontal

L changes in direction not in magnitude position.
O
L/3 COM
V
1000 L/2
Sol 19: =
(C) 5 e T −1 h1
L COM
V
⇒ e1000 6 ….(i)
=
T h2
V
1000
Again,
= I e T −1
Let angular velocity of the rod about hinge ‘O’ when it
1000 V is vertical be ‘ω’
dI 1000
= e T
dV T Moment of inertia of rod about o is
1000
2
1000 V L 
dI = e T dV I = Icom+ M   (parallel axis theorem)
T 2
Using (i) ML2 ML2 ML2
⇒I= + =
12 4 3
1000 60 60
∆ I= × 6 × 0.01= = = 0.2mA
T T 300 By using principle of conservation of energy
∆K.E = – DP.E
Sol 20: (B) For maximum possible volume of cube 1 2
⇒ Iω – 0 = – Mg(h2 – h1)com
2
2R = 3 a, a is side of the cube.
P hysi cs | 7.97

1 ML2 2 L 3g A is stationary
⇒ .ω = Mg ⇒ ω =
2 3 2 L Since, angular velocity of system would be same through
L VB 5
The tension in the rod at a point from hinge would ω= = rad/s
3 a a
be due to weight below that point and centrifugal force and
of that part.
VC = 2 a.ω = 5 2 m/s perpendicular to AC in vector
form.
VC = +5 î + (– 5) ˆj if co-ordinate system is along AB
r T and BC
Velocity of ‘C’ in original frame
dm 2L → → →
V c = V c + V a =+ 5iˆ – 5ˆj – 5ˆj ∴ (VA =– 5ˆj )
3

Mg ⇒ V c = 5iˆ – 10ˆj

2L 2M  M | Vc | = 52 + 102 = 5 5 m/s
m = ρ.A. =  ρ = 
3 3  A.L 

L Sol 3: Initial
2
Centrifugal force = ∫ rω .dm ω
L/3

L L
 r2 
= ω ∫ r.ρ.Adr = ω ρ.A  
2 2
/4 /4
L/3  2 L/3

3g M 1  2  8  4Mg
= .   [L ]   = final
L L 2
  9
  3
ω’
Tension at the point is T = mg + FC
2Mg 4Mg
= + = 2Mg
3 3 /2 /2

Sol 2: C Angular momentum of the system is conserved, since


no torque is applied
a Li = Lf
⇒ Ii ω = If ω1
A a B
 m 2 
2 
5 m/s ⇒  r + m  × 2  ω
 12 4 
 
Given,
10 m/s
 m 2 
2 
VA = 5m/s VB = 10 m/s =  r
+ m   × 2  ω1
 12 2 
In the frame of A
C  
0.03
⇒ × 30 = ω1
2 a 0.09
⇒ ω1 = 10 rad/s
A a B
By energy conservation,

5 m/s
7 . 9 8 | Rotational Mechanics

1 2 1 1 2
Sol 6: Given,
I ω = If(ω1)2 +  mv  × 2
2 1 2 2  A uniform wood door of mass m, height h and width w.
L 0.03 × 30 × (20) h 2h
⇒ (ω – ω1) = V2 ⇒ = V2 Location of hinges are and from the bottom of
2m 0.2 the door. 3 3
⇒ v = 3 m/s Let the hinges be named A and B.
∴ Velocity of ring along rod = 3m/s
w

Sol 4: Given,
(FA)
A (FA)x
A straight rod AB of mass M and length L, a horizontal
force F starts on A h/3 COM h
A F
mg
B
h/3 (FB)x
acm
Given, hinge A is screwed while B is not, So, the upward
component of force by hinge B is absent.
By equilibrium equations,
B acm
L
∑ Fx

2 = 0 ⇒ (FA)x = (FB)x
F = Macm (by newton’s second law)
∑ Fy = 0 ⇒ (F ) A y
= mg
T = Ia
L ML2
∑ MCOM =0
⇒ F. = ×a
2 12 (Moment about center of mass)
6F
⇒α=
ML ⇒ (FA)y  w  +(FA)x  h  + (FB)x  h  = 0
     
1 2 6 6
Acceleration of end B= acm î – x î
2
ω 3mgω
2F  F ⇒ mg . + (FB)x  h  = 0 ⇒ (FB)x = –
=– î  acm =  2   2h
M  M 3
→ →
2F and FA = (FA)x î + (FB) ˆj , FB = FB(– î )
∴ Magnitude of acceleration of end B =
M

3mgω →
3mgω
FA =– î + mg ˆj , FB = î
2h 2h
Given m = 20 kg, h = 2.2 m, ω = 1m
vcm=3m/s → →
Sol 5: 0.3 0.1 ⇒ FA = (–133.64 î + 196 ˆj )N and FB = 133.64 î
B String
w
Sol 7: Given, A thin rod of length ‘a’ with variable mass
A
Given, the wheel is rolling without slipping per unit length ρ = r0  1 + x  where x is distance from A.
 
∴  a
rAω = Vcm( VA = 0)
x
(pure rolling)   0 1 
x  a
The velocity of the string should be

VB = Vcm – rBω = Vcm  1 – rB  = 2m/s A B


  dx
 rA 
P hysi cs | 7.99

(a) Mass of the elemental part is dm = ρ.dx (c) Given, to find the moment of inertia about axis
perpendicular to rod and passing through A.
 x
⇒ dm = ρ  1 +  .dx
0
 a
dx
Mass of the rod
M a A B
 x
m= ∫ dm = ∫ ρ0  1 + a  .dx
0 0
a
 x2   3a 
⇒ m = ρ x +  m = ρ  
0
 2a  0
2 M
0
2 ∴
3aρ0
I= ∫ x dm ( dI = x2 dm)
∴ Mass of rod = 0
2
 x
(b) Center of mass is situated at distance of C from A but dm = ρ  1 +  .dx
where
0
 a 
M
a 
∫ x.dm 2 x3 
C= 0
⇒I= ∫ ρ0  x +  .dx
a 
M 0 
∫ dm  x3 x 4  7ρ0 a3
a
0 ⇒I= ρ  +  ⇒I=
M a
 x
0
 3 4a  0 12
Value of ∫ x.dm = ∫ x(ρ0 )  1 + a  dx
0 0
(d) We know that,
a
 x 2 x3   5a2 
= ρ  +  ⇒ ρ   Angular momentum L = I.ω
0  6 
 2 3a  0
0
 
7ρ0 a2
P.a = .ω
12
12
ω=
7ρ0 a2
a/2
(e) Given, an impulse of ‘P’ is applied at point B, then
Angular impulse about the axis will be
a/2
L = P.a
⇒ (Iω).ω = 2mga
P Pa.12.P
⇒ = 2 mga
7ρ0 a2
for minimum value of P, the angular velocity rod in the
final position should be zero 7 3  7 2 3
⇒ P2 = ρ ga2  2 P0 a  = ρ ga
by applying conservation of energy 6 0   4 0

K.E = – DP.E.
Sol 8: Given, two cylinders of mass 1 kg and 4 kg with
⇒ K.Ef – K.Ei = – mg(hf – hi)
radii 10 cm and 20 cm respectively.
1 2
0– Iω = – mg (a) also initial angular velocities as
2
5a2 ω1 = 100 rad/s and ω2 = 200 rad/s
5a
⇒C= 6 ⇒C= final angular velocities will be such that there is no slip
3a2 9 at point of contact
2
7 . 1 0 0 | Rotational Mechanics

r1 w21 Since, the body is in pure rolling

1
(Vcm = 3Rω) ⇒ (acm = 3Rα)
w2
w1
1 Solving (i) and (ii) we get,
6
acm = .g
5
r1 w11 Acceleration of the point where force is applied B
 3 8
a = acm î + R(α) î = acm î = g î
⇒ Vcontact = 0 ⇒ r1 ω11 – r2 ω12 =0 4 5

r1 ω11 | a | ≅ 16 m/s2
⇒ ω12 =
r2
Angular impulse on one cylinder due to other is Sol 10:
I1( ω11 – ω1) = (DP)(R1)
R+r
where DP = linear impulse while for the other sphere
v

r
R1 P R2 Given, A sphere of mass m and radius r and radius of
P loop as R + r the velocity of the sphere at the top most
point should be such that the centrifugal force balances
I2( ω12 –ω2) = (DP)(R2) the weight of sphere
1 mv 2f
I1 (ω1 – ω1 ) R1 ⇒ = mg
⇒ = R
I2 (ω1 – ω ) R2 ∴
2 2
( Center of mass makes circle of radius R)
⇒ 8( ω12 – ω2) = ω11 – ω1
⇒ vf = Rg
ω1 – 8ω2
⇒ ω11 = – 300 rad/s Since, the sphere is in pure rolling at every point of time,
5
ω1 vcm = rω
while ω12 = – = 150 rad/s
2 By principle of conservation of energy

Sol 9: K . E1 + P.E.i = K.E.f + P.E.f



1 1
Rf Mg ⇒ mv2 + Iω2 + mg(r)
2 2
3R a cm 1 1
= mv 2f + Iω2f + mg(2R + r)
2 2
I=MR2
1 1 2 1
⇒ mv2 +   mv2 = mv 2f +
f 2 2 5 2

Let, 1 2 2 2


    mv + mg(2R)
acm be acceleration of center of mass α be angular 2 5 5 f
acceleration 10
⇒ v2 = v 2f + g(2R)×
by Newton’s second law, 7

Mg – f = Macm  .... (i) 20Rg 27Rg


⇒v= Rg + =
and by considering torque 7 7

Mg × R + f × 3R = I . α .... (ii) 27Rg


∴v=
7
P hysi cs | 7.101

Sol 11: r.T2 = I2 .a2; Ma


r
⇒T=–
a2 2
a2 =
r2 2 Ma
– – mg sinα = ma
2
mg – T1 – T2 = ma1; T2
 M
a1 ⇒ – mg sin α =  m +  a
a1 = T1
 2
r1
(a) (T1 – T2).r = I1α1 also, mgsin α
1 ⇒a=–
r  M
a2 = 2a1 by constraint relations. m + 
 2
2g
⇒ a1 = Ma M ( + mgsin α )
7 So, T = – = ×
2x 2 2  M
2mg 200 m + 
⇒ T2 = = N  2
7 7
(b) And also velocity ‘v’ of the M.m.gsin α 2 × (1 / 2) × (9.8)(1 / 2) 9.8
T1 = = = = 1.65 N
body after traveling 1.2 m (2m + M) (2 × 0.5 + 2) 6

3 x x
(b) ω = 10 rad/s
v= 2× a× s = 4 m/s
7
mgsin α mg (0.5)(9.8)
⇒ α= a/R = – =– =–
 M R(2m + M) 0.2(1 + 2)
R m + 
Sol 12: The moment of inertia of a thin hoop about it’s  2 
diameter is
5
=–– . 9.8 rad/s2
1 6
M R2
2 =– 8.166 rad/s2
Here M = Lρ Now, ω2 = ω20 + 2αq
Also we have 2πR = L ⇒ 0 = (10)2 + 2(– 8.166) × q
1
⇒R = 100
2π ⇒θ= = 6.123 rad,
2 × 8.166
2
1 1  L  L3ρ So distance = θR = 6.123 × 0.2 = 1.224 m
So we have, I =M R 2 =L ρ   =
2 2  2π  8π2
Now using parallel axis theorem we have Sol 14:
2 2 v =0
L3ρ 3L ρ v0
 L 
I xx=
' Icm 2
+ M R= + L ρ  = m m
8π2
 2π  8π2
x Vcom

Sol 13:
A
T T×R
a
ω
ωL
 vA = Vcom – ; so for vA = 0
2
mg sin
ωL
⇒ Vcom =
(a) Now T – mg sina = ma  ... (i) 2
Now, mv0 = M.vcom (moment cons.)
MR 2 MRα
TxR = – Iα = – ×α=– ... (ii) mv 0
2 2  ⇒ Vcom =
M
Rα= a  ... (iii)
7 . 1 0 2 | Rotational Mechanics

and mv0x = Iω c


Torque about A, T = (mg)   = Ia
[Angular momentum conservation about O] 2
c
ML2 mg
So mv0x = ×ω ⇒α= 2 = 6g = 1.2g
12 5c c
5mc2
12mv 0 x 12
⇒ω=
ML2 5c  1.2g 
Acceleration of the center is a = ×  
4  c 
So Vcom = ωL/2’
= 0.3( 5g )
mv 0 12mv 0 x L or
⇒ = × ⇒ x = L/6
M ML 2 2 →
a = – 0.3( î + 2 ˆj )

Sol 15:
A B

C
2
mg

C
Before connection B is released
TA + TB = mg (By force equilibrium)
After connection B is released
and mg
TA is still , while TB = 0
TA = TB (for torque equilibrium) 2
Mg  mg 
TA = TB = g  mg – 
2 linear acceleration a = 2 
2  m 
⇒ Just after B is released  

TA is mg but TB = 0 angular acceleration = 0.5 g


⇒ FBD
TA Sol 16: By energy conservation,

A
C 5
2 4 R R/2
C
4 C
2 K.Ei + P.Ei = K.Ef + P.Ef
⇒ 0 + mgR
mg
1 m 2  1  m R 
=   (v )  1 +  +   (g)  
Moment of inertia about 24  2  4  2
2
m  2  C  
2
C 5  14gR
A= C +  + m .  ⇒v=
12   2  2 4 3
   
14gR
mc2  5  5 5 ∴ Velocity of the axis of cylinder =
⇒I=   + mc2 ⇒ I = mc2 3
12  4  16 12
P hysi cs | 7.103

Sol 17: FBD of disc is


v0 v

Angular momentum about any point on ground is 


conserved
f
⇒ mv0R = Iω + mvR
mR 2 ω MR 2 2f
mv0R = + mvR f.R = Iα ⇒ f. R = α⇒α=
2 2 MR
2 ∴ FBD of ant, in frame of disc
⇒v= v0 [ v = Rω]
3
a1
Work done by frictional for time to

1 1 1  m(R)
W=– mv 20 +  mv 2 + Iω2 
2  2 2 
mv 20 3 4 
f
W=– +  m. v 20  mRα
2 4 9  ⇒ mRα + f = ma1 ⇒ mRα + = a1
2
1 5Rα
W=– mv 20 ⇒ a1 = ∴ M = 3m
6 2
Also for t > to No frictional force exists Given, after time T, the ant reaches same point
1 1  5Rα  2 2
⇒W=– mv 20 for t ≥ to ⇒   T = 2π .R ⇒ T = seconds
6 2 2  5
Also ma = – mmg ⇒ a = – mg 1 2 4π
Also the angle moved by disc = aT = radians
and v = v0 – mgt 2 5

v0
to = Sol 19:
3µg
w
T f.R 2µg
α= = =
I MR 2 R R
m
2
Work done by friction for t < to = K.Ef – K.Ei m v=2wR
1 1  2µgt    1
2
2
⇒ W =  m(v 0 – µgt)2 + I   –  mv 0 
2 2  R   2 
 
1 1  Angular momentum of the system is conserved
⇒ W =  mµ2g2 t2 + m(2µ2g2 t2 ) – mv 0 µgt 
2 2  Li = Lf

1 mR 2 ω  mR 2 
⇒W= (3mm2g2t2 – 2mv0mgt) ⇒ =  + mR 2  ω1
2 2  2 
 
ω
⇒ ω1 =
Sol 18: 3
300cm
Impulse in the direction of velocity
20cm 100cm
 = m (vf – vi) = – mv = – 2mωR
7 . 1 0 4 | Rotational Mechanics

Impulse perpendicular to direction of velocity Sol 21:


= m(vf – mi) = m(Rω1)
mRω
=
3 F
mRω
Net Impulse = – 2mωR ˆj + î
3 h ma 0.9 m

37
| ∆p | = mRω
3
0.6 m
Impulse on particle due to disc =
F = 100 N
Impulse on hinge due to disc F = ma ⇒ a = 5 m/s2

37 The cabinet will tip when
⇒ | ∆P |disc due to hinge = mRω
3 F. h > mg(0.3) + ma(0.9)
20 × 10 × 0.3 20 × 5 × 0.9
Sol 20: h> +
30 inches 100 100
⇒ h > 1.5 m

View and also when


from the 2ft/s2 F.h > m = (0.9) – mg (0.3)
⇒ h > 0.3 m
car

∴ 0.3 < h < 1.5 m is the range of values of h for which


 cabinet will not tip.

Sol 22: In the frame of truck,



am/s2
2ft/s2
2cos ft/s2

2cos θ
Angular acceleration of door = ft/s2
w f f
 
w is the width of the door 2
z
We know that,

dω dω dθ
=α⇒ . =α y
dt dθ dt
x
⇒ ω.dω = α . dq
π /2 2M(a) – 2f = 2Ma1 (force equation)
ω2 4

2
–0=
w ∫ cos θ .dq (2f)R = (2T).α (Moment equation)
0
a1 = Rα (pure rolling)
ω2 4
⇒ = [1]
2 w Iα Ma1
f= =
⇒ω=2 R 2

Velocity of the outer edge = (ω)(w) = 8w 3a1 2a


⇒a= ⇒ a1 = = 6 m/s2
2 3
P hysi cs | 7.105

f=
Ma1
= 6N
Exercise 2
2
Single Correct Choice Type
Frictional torque magnitude about rod is
f.R = 0.6 Nm
Sol 1: (D) A
O
B
Friction torque about O is
I2
I1
 0.2 
= ± 0.6   ( k̂ ) – 0.6(0.1) ˆj = – 0.6 ( ˆj ± k̂ )
P
 2 
O’
D C
Sol 23: A a

We know the moment of inertia of a square plate along


Ma2
OO' as
12
v cm
Ma2
1 3a
.
3 2 ∴ I2 =
12
mv0 B
Also IZ = IX + Iy
C

Conserving linear momentum, we get (Perpendicular axis theorem)


(3m)(vcm) = mv0 ⇒ IZ = 2IX
v0 ∴
( IX = Iy fn square)
⇒ vcm =
3
Ma2
Conserving angular momentum about COM we get ⇒ IZ =
6
a (IZ is about transverse axis through p) to find IAPC or I1,
mv0 = = I.ω
2 3 we take x and y axis as diagonals of square and apply
mv 0 a  a 
2 perpendicular axis theorem again
⇒ = 3m   .ω z
2 3  3
y
v0
⇒ω=
2 3a
π
Time taken to complete one revolution =
ω x

3πa
= Iz Ma2
v0 ⇒ IX = Iy = ⇒ I1 = IX =
2 12
Displacement of point B will be I1
=1
 2a 3 ˆ 2a 1 ˆ  I2
vcm t î +  × i+ . j
 3 2 3 2 

Sol 2: (C) Given, moment of inertia of rectangular plate
 2π  a ˆ about transverse axis through P as I then the moment
⇒  + 1  (a) î + j
 3  3 I
of inertia of PQR about P will be greater that since
2
mass is distributed away from P unlike in PSR. Since, I
depends on distance ‘r’, the farther the mass, the more
the moment of inertia. The moment of inertia of PQR
7 . 1 0 6 | Rotational Mechanics

I then about any axis as shown below will be


will be less than about R since mass is distributed
closes to R 2 I
A B

P Q

D C
R I1
Consider, this axis as x-axis and y-axis perpendicular to
a line to it
Sol 3: (B) Given, A triangle ABC such AB = BC = a and
∠ACB = 90° of mass M (I1)x-axis = (I1)y-axis |By symmetry|

A
I1x
I1y
O 
a


C B
a

O is the midpoint
Iz = I1x + I1y = 2 I1x
Consider a counterpart with same mass such a square
is formed Iz
I1x =
2
A C1
Ma2
We know that Iz is independent θ since Iz =
O 6

⇒ I1x is also independent of q


C B
⇒ I1x (θ) = I1x (0) = I
we know the moment of inertia of a square about
Ma2
transverse axis through center as Sol 5: (B) Given, see-saw is out of balance.
6
⇒ Centre of mass is not at the center of see-saw.
But, here m = 2M (total mass)
Ma2 wH
⇒ Isquare =
3 L/2
Since both triangles are symmetric about axis through O
x
wG
they have equal moment of inertia about axis through O. COM
Isquare Ma2 ws
⇒ Irequired = =
2 6
Let ‘x’ be the distance of COM from center. By moment
equilibrium at center
Sol 4: (A) Given, I is moment of inertia of a uniform
L (wH – wG ) L
square plate about axis parallel to two of its sides and [wH – wG] = wS . x ⇒ x =
passing through center 2 wS 2
I Now if the girl and body move to half of the original
A B wH

L/2
x
wG
D C COM
ws
P hysi cs | 7.107

L Moment of inertia of rod about point ICOM


moment due to heavy body and girl is (wH – wG)
4 Md2
(opposite in direction to wS.x) =
12
while Angular momentum is conserved as no torque is acting
L on the system. (while energy is not since a force acts on
wS . x = (wH – wG)
2 rod at point)
L ∴ Initial angular momentum = final angular momentum
∴ wS . x > (wH – wG)
4 d
I 1ω1 + m v 1 = I 2ω2
The side the girl is sitting on will once again tilt 2
downward
Md2 dv  Md2 Md2 
⇒ (0) + m =  +  ω1
3 2  12 4 
 
Sol 6: (A) F
md 3
⇒= v md2 ω1
2 4
F 2v
⇒ ω1 =
3d
The force on the hinge is same as the force on the
thread this can be found by using force equilibrium
conditions. Sol 9: (D)
l
Since, there is a torque always about hinge on pulley,
angular velocity increases. P

Sol 7: (D)
P


w
 By the principle of conservation of energy
O
torque acting on the pole due to weight about point K.Ei + P.Ei = K.Ef + P.E.f
O is 1 1

⇒ 0 + mg(l) = mv2 + Iω2 + 0
→ L 2 2
T = r × F = W cosq
2 1 2 2 1 2 2 2
⇒ mgl = mr w + . mr ω
T = Ia 2 2 5
L
W cos θ
3mgcos θ 3gcos θ 10 gl
⇒α= 2 = = ⇒ω=
ML 2 2mL 2L 7 r2
3
3gcos θ Angular momentum of the sphere about P is
Acceleration of point P=L.α =
2 L = Iω + m.l.v
2 2
⇒L= mr ω + m. l. r ω
Sol 8: (B) 5

10 2 
⇒ L = m. gl .  r + l
7  5 
6m d

m v
7 . 1 0 8 | Rotational Mechanics

Sol 10: (C) when α = 45°


vA = – v and y =
2
A
dv A 2v 2 . 2
⇒ =–
y dt 
vA
Angular acceleration =
30° v
C B
x 45°
x2 + y2 = l2 aA
aA
aA
L = length of ladder = constant 2
2
dx dy
⇒ x. + y. =0 a=0
dt dt
⇒ x. v + y.vA = 0 aA

α= 2
x
⇒ vA = – .v = – 3v (L)
y
2v 2
O α=
L2
L/2

3v L ( L sin30°) OC= L
Sol 12: (A)
3 60° 2 2
L /2 Initial Final
ω0
30°
v

Angular velocity of rod


v0
ω=0 v=0
| – 3vjˆ + viˆ | 2v
= =
L L
f f
L
Velocity of center = .ω = v Frictional forces acts to reduce the velocity of bottom
2
most point.
dv
Sol 11: (A) If =0 f = – macm (By Newton’s second law)
dt
T = Ia
x2 + y2 = l2 MR 2 MRα
⇒ f.R = .a⇒f=
dx dy 2 2
⇒ x. + y. =0
dt dt After time t = t, angular velocity and lineal velocity
⇒ x. v + y vA = 0 becomes zero.

dx dv dy dv ⇒ 0 –v0 = – acmt and 0 – ω0 = – at


⇒ .v + x. + . v + y. A = 0
dt dt dt A dt
f
dv A v0 Macm
⇒ v2 + v 2A + y. ⇒ = =
dt w0 α 2f
MR
dv A 0 – (v 2 + v 2A )
⇒ = v0 R v 1
dt y ⇒ = ⇒ 0 =
ω0 2 Rω0 2
P hysi cs | 7.109

Sol 13: (C) Conserving linear momentum Sol 14: (C) When F1 is applied, the body moves right
and angular acceleration is developed accordingly by
2mv – mv = 2m × vcm
friction
v
vcm = when F3 is applied, the angular acceleration developed it
2
the body move left.
Initial angular momentum
When F2 is applied the body can move either left of
b b 3vbm right depending on angle of inclination.
= m × 2v × + mv × =
2 2 2
Final angular momentum Multiple Correct Choice Type
 b 2
b
2
mb2 ω
= m   + m    × ω = Sol 15: (B, C) x
 2  2   2

COM
3mvb mb2 ω A B
⇒ =
2 2
FBD of rod
3v
∴ ω2 =
b x
For skater at x = b/2

RA w RB

wb RA + RB = w (force equilibrium)

2 RB.d = w.x (torque equilibrium)

wx w(d – x)
⇒ RB = and RA =
d d
ωb
vx = v + cosq
2
Sol 16: (A, D) mg cos
ωb
vy = – sinq
2
 mgsin
θ = ωt
mg cos
v
∴ vx at t =

2
mg
3v  3vt  Sliding condition = mg sinθ > mmgcosθ
vx = v + cos  
2  b 
⇒ tanθ > m
3V  3vt  h a
∴ x = ∫ v x dt = ∫ v + cos   Toppling condition = mgsinθ. > mgcosθ.
2  b  2 2
a
⇒ tanθ >
3V  3vt  h
= vt + × b × sin  
2 × 3V  b  a
If µ >
h
b  3vt 
= vt + sin   a
2  b  tanθ > is met earlier than tanθ > m
h
ωb ∴ Topples before sliding
y= ∫ v y dt = ∫ – 2
sin (ωt)
a
If µ <
+ωb b  3vt  h
= cos (ωt) = cos  
2×ω 2  b 
It will slide before toppling
7 . 1 1 0 | Rotational Mechanics

Sol 17: (A, C, D) Kinetic energy of system before collision is equal to


a=g kinetic energy of system just after collision since, the
mg h collision is elastic

d mR
Sol 20: (B, C)
Angular momentum = mvd = mgtd m
dL
Torque of gravitational force = = mgd
dt
Kinetic energy of the body
Moment of inertia = m(d2 + h2)
1 1 1
1 = mv2 + mv2 + Iω2
where h = H0 – gt2 2 2 2
2
v gt  1 2 4 ∴
Angular velocity = = =  1 + .  mv2 = mv2 ( v = Rω)
d d  2 3 3

→ → → ∴ 2
Sol 18: (A, B, C) T = A × L ( I= mR2 only hollow sphere
3
→ ∴
Non-viscous liquid )
dL → →
⇒ = A×L
dt Angular momentum about any point on ground
→ 2 8
dL → = 2mRv + mR2ω = mRv
∴ ⊥ L 3 3
dt
→ →
Components of L on A remain unchanged because Sol 21: (B, C) A
→ R
if L component changes the L.H.S changes while w
R.H.S remains unchanged which is a contradiction. If  
magnitude of L changes with time, thin L.H.S and R.H.S
vary differently with time which is a contradiction. B v


L
Suppose L = (x.t) Since the cylinder does not slip

|L| At point B velocity = 0
→ →
→ → → → 
dL xL → →
A× L ⇒ − V + V cm + Rω = 0
Then = while A × L = xt
dt → → →
|L| |L| ⇒ V cm = – (v – Rw) î

At point A, velocity = 0
Sol 19: (B, C) A
⇒ v = Rω

⇒ V cm = 0

x
Sol 22: (B, C, D) To the right of B, angular acceleration
will disappear but linear acceleration will increase since
no friction is present angular velocity attained by disc
m
after time T is
Linear momentum is not conserved because of hinge
force angular momentum about A is conserved since ω = aT
torque at A is zero. 1
and 2π = aT2
2
P hysi cs | 7.111

Time to complete one rotation (c)


2π = ωt F/2
Ix F
2π 2π.T T
⇒t= = = Ix
αT 2π.2 2 F/2

Sol 23: (C, D) Given,


180° < Qf – Qi > 360°
(d) While D is possible
π rad < Qf – Qi < 2π rad
F
Sol 24: (A, B, C, D)
1
2 F F

3
Sol 26: (A, B, D)

v
4 h w
I
A f = w = mg
I = I1 + I3 (Perpendicular axes theorem)
L = mvh (Angular momentum)
Also I1 = I3 (by symmetry)
µmg
I acceleration = – = – mg
⇒ = I1 = I3 m
2
Vt = v – mgt
I = I2 + I4 (perpendicular axis theorem)
Lt = m(v – mgt)h
Also I2 = I4 (by symmetry)
dL t
I = T = – mmgh
⇒ = I2 = I4 dt
2
Sol 27: (A, C, D) If Re spreads or curls up his hands,
Sol 25: (B, C) Option A is incorrect, since the statement moment of inertia changes, accordingly angular
indicates a force body system as below. velocity changes too.
If I ω = Constant, it cant keep
1 2
F1 F2 I ω the same, rotational kinetic energy would also
2
change.

Which is not in equilibrium while B, C are possible Sol 28: (A, C, D)

F3 F1
u
w

u = Rω (for pure rolling)


F2
→ → →
(b) F1 + F2 + F3 = 0 The velocity of bottom most point is u – Rω = 0
7 . 1 1 2 | Rotational Mechanics

the velocity of topmost point is u + Rω = 2u Sol 32: (A, C)


∴ 0 ≤ v ≤ 2u
a1

fpseudo=ma
Rw 30°
120° f

P
u In frame of plank,
Fpseudo – f = ma1
Velocity of P is
f
a1 = a – where a is acceleration of plank
→  u 3u ˆ  m
V = u î +  – ˆi + j
 2 2  F–f
 a=
M

u 3u ˆ
⇒ V = î + j Total K.E. of system = work done by force F
2 2 ∴
( no other external forces is doing work)
⇒V=u
Work done on sphere = work done by friction + work
If CR is horizontal done by pseudo force = change in K.E.

V = uiˆ + ujˆ
Sol 33: (A, B, C)
V= 2u A
2v

Sol 29: (B, C, D) Angular moment about O is not


constant because a component of weight causes v
torque at point ‘O’.
Angular moment about C is zero since weight is parallel
axis B

About O, VA = V + Rω = 2V
→ → → VB = V – Rω = 0
L = m( r × v ) gives angular momentum in direction
perpendicular to length of thread and velocity. The 1 3mRV
L about B = mvR + mR2ω = clockwise
vertical component never changes direction. 2 2
1 mRV
L about A = – mvR + mR2ω = anti-clockwise
Sol 30: (A, C) 2 2

Sol 34: (A, B, C)


w
v
N

>v P

r v
o o
30 30
A cylinder rolling down with incline may or may not
w
<v
attain pure rolling. It depends on length of the incline

Sol 31: (B, C) Friction on cylinder under pure rolling B


depends on the external forces
P hysi cs | 7.113

N Sol 38: (B, C) By holding a pole horizontally, the


moment of inertia is increased leading to slower
>v angular acceleration due to undesired torques.
C
v Also, adjusts center of gravity to be vertically over rope
to eliminate torque.
<v R
R M R Assertion Reasoning Type

VP = (V + rω cosθ) î + (rω sinθ) ˆj
→ Sol 39: (B) A cyclist always bends inwards to reduce the
| VP |2 = V2 + r2ω2 + 2rω cosqV centrifugal force.
Also, he lowers the center of gravity.
= (R2 + r2 + 2r R cosθ)ω2
→ But the reason does not explain the assertion
| VP |2 = M2P2ω2

⇒ | VP | = ( MP ).ω
Using the above relation we draw a circle with radius R Li
to get point with velocities R.ω = v
θi
Sol 35: (A, B, C) As the ring enters frictional force Sol 40: (B)
x θf
(limiting) acts on sphere increases angular acceleration
in clockwise direction and slows down the linear motion.

Sol 36: (B, C, D) Rolling motion starts when the Lf


point of contact has zero velocity. Conserving angular
momentum about point of ground gives Statement-I is true because the initial angular moment
mvR = Iω + mv’R about any point on xy plane is Li=mvisinqix and final
∴ angular momentum Lf = mvfxsin qf
⇒ mvR = mR2ω1 + mR2ω1 ( v1 = Rω1)
for elastic collision vf = vi, qi = qf
v
⇒ ω1 = ⇒ Li = Lf
2R
v Statement-II is also correct since the disc is in equilibrium
v1 =
2 But II is explanation of I
Time taken to achieve pure rolling is
v v Sol 41: (A)
v – u = at ⇒ – v = –mgt ⇒ t =
2 2µg
Q
P
Sol 37: (A, B, C, D) Distance moved by ring
W
v 2 – u2 3v 20 r1
= = r2
2a 8µg
3v 20
Work done by friction = – mmg. In frame of O
8µg
3mv 20 ωP = ωQ = ω
= –
8 vP = rPωP vQ = rQωQ
1 v 20 mv 20 In frame of P
Gain in rotational K.E. = mR2. =
2 4R 2 8
v0 = – rPωP vQ = rQωQ –rPωP
 3mv 20 1  mv 20 v0
Loss in K.E. = –  – + mv 20  = w0 = – = – ωP = ω
 8 8  4 rP
 
7 . 1 1 4 | Rotational Mechanics

(rQ – rP )(ω) Taking an elemental point dy


w1Q = =ω
(rQ – wP )

∴ Statement-I is true and statement-II is correct


explanation of I I dy
y
Sol 42: (B) Statement-I true by parallel axis theorem.
Statement-II is true but doesn’t explain parallel axis x
theorem.
dm = ρ.(2x)dy

Sol 43: (B) See above = ρ.(2y)dy [ y = x]

M 4M
Sol 44: (D) fy ρ= =
1 2 2

fx 4
dm(2x)2
dI2 = + dm(y)2
12
L
f
 m2 
 for rod, parallel axis theorem
 12 
 /2  
y2 ∴
⇒ ∫ dI2 = ∫  + y 2  (2y)ρ dy ( y = x)
Fx = F (force equilibrium in x-direction )  3 
0  
∴ Assertion is false  /2  /2
8ρ 3 8ρ  y 4 
While reason is true since ⇒ ∫ dI1 = I2 =
3 ∫ y .dy = .  
3  4 
0 0
F
T = Iα = F.x ⇒ α = .x  4 
I 2ρ ML2
=   =
3  16  6
Sol 45: (B) Statement-I is true, which is the condition
for pure rolling
Sol 48: (A) dIx = (dm)(y2)
Statement-II is also correct by the definition of center of
 /2
mass but II is not correct explanation of I. 3
⇒ ∫ dIx = ∫ 2ρy dy
0
Sol 46: (D) Statement-I is false, because a body can roll if  /2
 y4  8M  4 M2
we throw it with property determined linear and angular ⇒ Ix = 2ρ   = . =
velocities.  4  0 2 64 8
Statement-II is true by definition of pure rolling
Sol 49: (C) Moment of inertia about base is
Comprehension Type 2
L 
dI = dm  – y 
Sol 47: (C) 2 
y L /2 2
 L 
45° 45°
⇒ ∫ dI = ∫  – y  (2ρy) dy
2 
0

M L/2
L 
⇒ I = 2ρ ∫ y 2  – y  . dy
0 2 
45°
x
P hysi cs | 7.115

 L y3 y 4 
L /2 So 55: (D)
⇒ I = 2ρ  . – 
 2 3 4 
0 F

1 1   4 – 3  ML2 C
⇒ I = 2ρL4  48 – 64  = 8ML2  64 × 3  =
    24

Frictional force in this depends on distance between


Sol 50: (C) Iz = Ix + Iy (perpendicular axis theorem) application of force and center

ML2 ML2 ML2


⇒ Iy = – = f
6 8 24 Sol 56: (B) F

→ → →
Sol 51: (B) d L = dm ( r × v )
pseudo
force = Fs
dm 1
= ma f
y
 By basic FBD’s we can understand that friction acts in
r  forward direction
v x
∴ Option B is correct

L
z And the torque is acting horizontally, since the
horizontal component of angular momentum is only
dr changing.

 Previous Years’ Questions


r 
L
 Sol 1: (A) W =∆U =Uf − Ui =U∞ − UP
v
= −UP = −VP (as m = 1)
−mVP =
(z direction)
Potential at point P will be obtained by integration as
∴ Angular momentum is down at 20° to horizontal given below.

Sol 52: (B) There is a torque since angular momentum


is always changing direction. 2 2
4R 16R +r
Sol 53: (A)

r
dr
F

f
Let dM be the mass small ring as shown
Friction reduces linear acceleration and increases
angular velocity M 2Mrdr
= dM = (2πr)dr
π(4R)2 − π(3R)2 7R 2
Sol 54: (A) Same as previous
G dM
dVP = −
16R 2 + r 2
7 . 1 1 6 | Rotational Mechanics

4R I zz = IAB + IA`B` = ICD + IC`D` = 2I AB = 2ICD


2GM r 2GM
= − ∫
7R 2 3R 16R 2 + r 2
dr = −
7R
(4 2 − 5)
I AB = ICD

2GM
∴W =+ (4 2 − 5) Alternate The relation between I AB and ICD should
7R be true for all values of Nθ
At θ = 0, ICD = I AB
2 1 2
Sol 2: (A) =MR 2 Mr + Mr 2 Similarly, at θ = π /2, ICD = I AB
5 2
(By symmetry)
2 3
Or MR 2 = Mr 2 Keeping these things in mind, only option (a) is correct.
5 2
2
∴r = R Sol 6: (D) In case of pure rolling,
15
mgsin θ
f= (Upwards)
Sol 3: (B) Condition of sliding is mR 2
1+
ms sin θ > µ mg cos θ or tan θ > µ I
or tan θ > 3  … (i) ∴ f ∝ sin θ
Condition of toppling is Therefore, as θ decreases force of friction will also
decrease.
N
f Sol 7: (A) On smooth part BC, due to zero torque,
angular velocity and hence the rotational kinetic
O
mg energy remains constant. While moving from B to C
sin
mg cos   translational kinetic energy converts into gravitational
potential energy.


Torque of mg sin θ about O>torque of mg about


Sol 8: (B) From conservation of angular momentum
 15   10  (I ω = constant), angular velocity will remains half. As
∴ (mg sin θ )   > (mg cos θ )  
 2   2  1 2
K= Iω
2 2
or tan θ >  … (ii)
3 The rotational kinetic energy will become half. Hence,
With increase in value of θ , condition of sliding is the correct option is (b).
satisfied first.
Sol 9: (A) Let ω be the angular velocity of the rod.
Sol 4: (A) I Applying angular impulse = change in angular
remaining= Iwhole − Iremoved momentum about center of mass of the system
 2 2
1 2 1  R  1  2R   J.
L
= IC ω
or I (9M)(R) − m   + m    ….(i) 2
9 2  3  2  3  
   ML2 
L  v
2 ∴ (Mv)
=   (2)   ω ∴ ω =
2  
Here,=
m
9M R 
× π = M  4  L
πR 2
3
Substituting in Eq. (i), we have I = 4MR 2 Sol 10: (A) In case of pure rolling bottom most point is
the instantaneous center of zero velocity.

Sol 5: (A) A’B’ ⊥ AB and C’D’ ⊥ CD Velocity of any point on the disc,, where r is the distance
of point from O.
From symmetry IAB = IA`B` and ICD = IC`D` From
theorem of perpendicular axes, rQ > rC > rP
P hysi cs | 7.117

Paragraph 2

C Q Sol 15: (D) a= Rα

 
P 2kx − f  fR 
∴ =R  
M  1 MR 2 
O   2 
Solving this equation, we get
∴ v Q > v C> v P
a

Paragraph 1

1 1
Sol 11: (C) I(2ω)2 =kx12  …(i) 2kx
2 2
1 1
(2I)(ω)2 =kx22 ….. (ii)
2 2  f
x1 2kx
From Eqs. (i) and (ii), we have = 2 f=
x2 3
2kx 4kx
Sol 12: (A) Let ω' be the common velocity. Then from ∴ | Fnet=| 2kx − f= 2kx − =
3 3
conservation of angular momentum, we have
This is opposite to displacement.
(I + 2I)ω'= I(2ω) + 2I(ω)
4kx
∴ Fnet = −
4 3
ω' = ω
3
From the equation,  4kx 
Sol 16: (D) Fnet = −  x
Angular impulse = change in angular momentum, for  3 
any of the disc, we have F  4k 
∴ a = net = −  2
 x = −ω x
 4  2Iω M  3M 
τ.t = I(2ω) − I  ω  =
3  3
4k
2Iω ∴ ω=
∴ τ= 3M
3t
Sol 17: (C) In case of pure rolling mechanical energy
Sol 13: (B) Loss of kinetic energy = Ki − K f will remains conserved.
2 2
1 1  1 4  1 1 11  v  1 2 
=  I(2ω)2 + (2I)(ω)2  − (3I)  ω  = Iω2 ∴ Mv 20 +  MR 2   0  =
2  kxmax 
2 2  2 3  3 2 22  R  2 

2
 3v 2  3M
1 2 1 v 1 ∴ xmax = v
Sol 14: (D) mv + I   =
mg   ∴ I =mR 2 4k 0
2 2 R   4g  2
 
2kx
∴ Body is disc. As f =
3
2kx 2k 3M
µMg = max =
∴ Fmax = v
3 3 4k 0

3M
∴ v 0 = µg
k
7 . 1 1 8 | Rotational Mechanics

Sol 18: (B) Angular momentum about rotational axis  m 2R 2 


IP=  ( ) 2  m R
+ m ( 2R ) − 
2
m 
+ 5R 2 
  2   4 (2) 4 
  

 mR 2 5mR 2 
v ⇒ 2mR 2 + 4mR 2  −  + 
x    8 4 

x=vt
11 37
⇒ 6mR 2 − mR 2 ⇒ mR 2
I + m vt 2  ω
L( t ) =
 ( )  8 8
IP 37 8
dL t = × ≈3
= 2mv 2 t ω IO 8 13
dt

Torque
= τ (2mv ω) t
2
Sol 21: (A)

Sol 19: (C)

P  

P Q  = t
 = t

 
R sin    R sin 
L‘P LP
 
R R
R cos  R cos 
  
L0= r0 × p 2 ωR sin ( ω t )
So, v r =
 At t T=
= / 2, v r 0
L0 is always directed along the axis & its magnitude is
constant. So two half cycles will take place.

Sol 20: (C) Sol 22: (C, D)

O P
2R
t R sin t
Let mass of original disc = m
m m
The mass of disc removed = × πR 2 =
π 4R( ) 2 4 Rt
According to problem particle is to land on disc.
So M.O.I of remaining section about axis passing
If one consider a time ‘t’ then x component of disc is
m ( 2R )
2
 m R2 m 
through “O” IO = − + R2  Rωt
2  4 ( 2 ) 4  R sin ωt < R ω t

 mR 2 + 2mR 2   3 13 This particle ‘P’ land on unshaded region. For “Q”


⇒ 2mR 2 −  2
 ⇒ 2 −  mR ⇒ mR 2 x-component is very small and y-component equal to
 8   8 8
P it will also land in unshaded region.
MOI of remaining section about “P” Now repeat same thing when right part is shaded then
correct answer is “C” or “D”
P hysi cs | 7.119

Sol 23: (A) In both the cases, the instantaneous axis  50 0.4 2 
50 ( 0.4 ) ( )
2
will be along z-axis i.e. along vertical direction. + 4 ( 6.25 ) ( 0.2 ) ω
 2
× 10
=
2  2 
 
Sol 24: (D) w.r.t. centre of mass only pure rotation of
disc will be seen. So in both the cases, angular speed ω =8rad / sec
about instantaneous axis will be " ω " .

Sol 28: (C, D) Condition of translational equilibrium


Sol 25: (A, B)
N1 = µ2 N2
VP = Rωˆi +
ω ˆ
2
( ) (
− j × R cos 30° ˆi + R sin 30° kˆ ) N2 + µ1 N1 =Mg

ω ω 11 ˆ 3 mg
= 3Rω ˆi + 3 Rkˆ − Riˆ= Rω i + Rω kˆ Solving N2 =
4 4 4 4 1 + µ1 µ2
µ2 mg
N1 =
Sol 26: (D) 1 + µ1 µ2

N Applying torque equation about corner (left) point on


f the floor

mg cos θ N1  sin θ + µ1 N1  cos θ
=
mg 2
1 − µ1 µ2
Solving tan θ =
Translation motion: 2 µ2
mg sin θ − f = macm  …(i)
Sol 29: (2)
Rotational motion
R Icmα 
f= …(ii) F

Rolling without slipping


αR =acm  …(iii)
From (ii) & (iii) o
R 30
Icm acm
f=
R2
Rsin30o
Put this in (i)
Icm acm F
mg sin θ − = macm
2
R
mg sin θ
acm = F
 Icm 
 2 + m 
R  τ = Iα
As IP > IQ 3FR sin30° = I α

MR 2
Sol 27: (8) Conservation of angular momentum about I=
2
vertical axis of disc
α =2
ω = ω0 + α t
ω =2 rad / s

Sol 30: (4) Since net torque about centre of rotation


is zero, so we can apply conservation of angular
momentum of the system about center of disc
7 . 1 2 0 | Rotational Mechanics

Li = L f 3
N = mg
2
0 = I ω+ 2mv (r / 2 ) ; comparing magnitude

 0.45 × 0.5 × 0.5  0.5 2


∴ =ω 0.05 × 9 × ×2 N= mg
 2  2 3

∴ω =4 mg 16 16 3
fr
= = =
3 3 3
Sol 31: (7) Kinetic energy of a pure rolling disc having
Torque balance (about A)
1 1  mR 2  v 2 3
velocity of centre of mass v =+mv 2   = mv 2 h L
2 2  2  R 2 4 N× mg × sin30°
=
cos 30° 2
So,
3 2 2h L
(30 ) 34 m ( v 2 ) + mg ( 27 )=
m ( 3 ) + mg= mg × = mg ×
2 2
∴ v2 7 m / s 3 4
4 3

h 3 3
Sol 32: (D) Using conservation of angular momentum =
L 16
mR 2 ω
 8 ω   m 9R 2 8 ω   m 2 8 ω 
=  mR 2 × + × × + ×x ×  Sol 35: (A, B, D)
 9   8 25 9   8 9 

 d r (t)
4R 
⇒x =  dv
v= = 3 α t2 ˆi + 2 β t ˆj ,a = = 6 α t ˆi + 2 β ˆj
5 dt dt

2
Sol 33: I = ∫ ρ 4 πr 2 r 2 dr
At t = 1,=

v (10 ˆi + 10 ˆj) ms −1

3 
a 20 ˆi + 10 ˆj ms−2
=
∫ (r ) (r
)(r ) dr
2 2
IA ∝
 10 ˆ ˆ
=r i +5 j m
IB ∝ ∫ (r )(r )(r ) dr
5 2 2 3
    5 ˆ
IB 6 L0 =
r × mv =
− k N m s
∴ =  3 
I A 10
 
dv
(
F m = 2 ˆi + ˆj N
=
dt
)
Sol 34: (D) Force balance
      20 ˆ 
τ = r × F = r × ma =  − k N m
 3 
o
N sin 30

N cos 30o
ωa
Sol 36: (A, D) ωz = cos θ = ω /5
30 o

B
30o c
N

mg A
fr

N + N sin 30° =mg

You might also like